Anda di halaman 1dari 101

Item: 1 of 46

11 f M a rk

Q. Id : 1231 [549201]

<a
P re v io u s

E>
Next

II
Lab V a lu e s

Bp*-
N o te s C a lc u la to r

Experimental drug X is being tested for the treatment of stage IV solid tumor malignancies. Part of the drug
company's evaluation process is to analyze survival data after 3 months of treatment. The results are given in
the table below.

Alive
Dead
Treated with Treated with
drugX placebo
20 38
60 38
Which of the following best represents the number needed to harm for drug X?
O A. 2
O B. 3
O C. 4
O D. 7
OE.9

Block Tim e Rem aining: oo :


T u to r

31

Feedback Suspend End Block

Item: 1 of 46

11 f M a rk

Q. Id : 1231 [549201]

<a
P re v io u s

E>
Next

II
Lab V a lu e s

Bp*-
N o te s C a lc u la to r

Experimental drug X is being tested for the treatment of stage IV solid tumor malignancies. Part of the drug
company's evaluation process is to analyze survival data after 3 months of treatment. The results are given in
the table below.

Alive
Dead
Treated with Treated with
drugX placebo
20 38
60 38
Which of the following best represents the number needed to harm for drug X?
Explanation: User Id: 549201
Unfortunately, not all interventions end up helping patients. The number needed to harm (NNH) represents the

number of people that must be treated for one adverse event to occur. It is calculated in a manner similar to
the number needed to treat (NNT) but using the attributable risk instead of the absolute risk reduction:
NNH = 1 / Attributable risk
To determine the attributable risk, first calculate the adverse event rates in the treatment and placebo groups.
Ari\/pr^p p\/pnt ratp in trpptm pnt nrm in = fid / fif) = n 7

Block Tim e Rem aining: oo


T u to r

: 55

Ft

Feedback Suspend End Block

Item: 1 of 46

11 f M a rk

Q. Id : 1231 [549201]

<a
P re v io u s

E>
Next

II
Lab V a lu e s

Bp?-N o te s C a lc u la to r

O D. 7 [5%]
O E . 9 [3%]
Explanation: User Id: 549201
Unfortunately, not all interventions end up helping patients. The number needed to harm (NNH) represents the
number of people that must be treated for one adverse event to occur. It is calculated in a manner similar to
the number needed to treat (NNT) but using the attributable risk instead of the absolute risk reduction:
NNH = 1 / Attributable risk
To determine the attributable risk, first calculate the adverse event rates in the treatment and placebo groups.
Adverse event rate in treatment group = 60 / 80 = 0.75
Adverse event rate in placebo group = 38 / 76 = 0.50
The attributable risk can then be calculated by subtracting the event rate in the control group from the event
rate in the treatment group:
Attributable risk = Event rate!re3rnert - Event ra te ^ ^
Attributable risk = 0.75 - 0.50 = 0.25
The absolute risk increase attributable to the treatment is 0.25. That is, patients treated with drug X have a
25% absolute increased risk of being dead at 3 months over patients on placebo. The NNH then is simply the
multiplicative inverse of the attributable risk:
NNH = 1 / Attributable risk
NNH = 1/0.25 = 4
The result indicates that for every 4 patients treated with drug X, 1 will experience an adverse event (in this
case, death). This drug will likely not get very far in the approval process!
Educational objective:
Number needed to harm = 1 / Attributable risk
Time Spent: 72 seconds Copyright USMLEWorld,LLC. Last updated: [3/21/2014] V

Block Tim e Rem aining: o i : 02


T u to r

Feedback Suspend End Block

Item: 2 of 46
Q. Id : 1207 [549201]

<a
P re v io u s

E>
Next

II
Lab V a lu e s

11 f M a rk

Bp*-
N o te s C a lc u la to r

A study evaluated the role of ACE inhibitors in the prevention of coronary events in diabetic patients. During
five years of follow-up, 120 diabetic patients out of 400 who had taken an ACE inhibitor developed an acute
coronary event. Over the same time, 100 diabetic patients out of 300 who had not taken an ACE inhibitor
experienced coronary events. What is the relative risk of developing a coronary event in diabetic patients
taking ACE inhibitors compared to diabetic patients who are not taking ACE inhibitors?

O
O
O
O
O

A. 120x200/100x280
B. 120x280/100x200
C. 120x100/280x200
D. (120/220)/(280/480)
E. (120/400)/( 100/300)

Block Tim e Rem aining: o i :


T u to r

07

Feedback Suspend End Block

Item: 2 of 46

11 f M a rk

Q. Id : 1207 [549201]

<a
P re v io u s

E>
Next

A study evaluated the role of ACE inhibitors in the prevention of coronary events in diabetic patients. During
five years of follow-up, 120 diabetic patients out of 400 who had taken an ACE inhibitor developed an acute
coronary event. Over the same time, 100 diabetic patients out of 300 who had not taken an ACE inhibitor
experienced coronary events. What is the relative risk of developing a coronary event in diabetic patients
taking ACE inhibitors compared to diabetic patients who are not taking ACE inhibitors?
Lab V a lu e s N o te s C a lc u la to r
A

O
O
O
O

A. 120x200/100x280 [10%]
B. 120x280/100x200 [4%]
C. 120x100/280x200 [5%]

D. (120/220)/(28Q/480) [6%]
* E. (120/400)/( 100/300) [74%]
Explanation: User Id: 549201
It is important to know how to calculate some basic measures of effect when given raw data. The first step is
to organize the data into a 2x2 table as shown below.
Coronary event No coronary event
ACE inhibitor 120 (a) 280 (b) 400
No ACE inhibitor 100 (c) 200 (d) 300
220 480 700
Relative risk is calculated for cohort studies using the following formula:
RR = [a/(a+b)] / [c/(c+d)] = (120/400)/( 100/300) = 0.90.
Odds ratio is calculated using the following formula: OR = ad/bc (Choice A).
Educational Objective:
Questions about relative risk are commonly asked. Make sure you can construct 2*2 table (Exposure in rows
and Outcome in columns) and calculate relative risk.
V

Block Tim e Rem aining: o i : 15


T u to r

Feedback Suspend End Block

Item: 3 of 46
Q. Id : 8519 [549201]

<a
P re v io u s

E>
Next

11 f M a rk

II
Lab V a lu e s

Bp*-
N o te s C a lc u la to r

A study is conducted assessing the relationship between smoking status and the forced expiratory flow
between 25% and 75% of vital capacity (FEF2-_7.%) in 4 groups. Group A consists of 200 nonsmokers, group B
consists of 200 light smokers (1-7 cigarettes per day), group C consists of 200 moderate smokers (8-22
cigarettes per day), and group D consists of 200 heavy smokers (23+ cigarettes per day). F E F ^ * is
measured in all participants. Which of the following is the most appropriate statistical method to compare the
FEF^.*. results among all 4 groups?

O
O
O
O
O
O

A. Analysis of variance
B. Chi-square test
C. Meta-analysis
D. Multiple linear regression
E Pearson correlation coefficient
F. Two-sample t-test

Block Tim e Rem aining: o i : 22


T u to r

Feedback Suspend End Block

Item: 3 of 46

11 f M a rk

Q. Id : 8519 [549201]

<a
P re v io u s

E>
Next

II
Lab V a lu e s

Bp?-N o te s C a lc u la to r

A study is conducted assessing the relationship between smoking status and the forced expiratory flow
between 25% and 75% of vital capacity (FEF2-_7.%) in 4 groups. Group A consists of 200 nonsmokers, group B
consists of 200 light smokers (1-7 cigarettes per day), group C consists of 200 moderate smokers (8-22
cigarettes per day), and group D consists of 200 heavy smokers (23+ cigarettes per day). F E F ^ * is
measured in all participants. Which of the following is the most appropriate statistical method to compare the
FEF^.*. results among all 4 groups?
* A. Analysis of variance [51 %]
O B. Chi-square test [24%]
O C. Meta-analysis [9%]
O D. Multiple linear regression [6%]
O E. Pearson correlation coefficient [5%]
O F. Two-sample t-test [3%]
Explanation: User Id: 549201

Analysis of variance (ANOVA)


Null hypothesis
All samples drawn from
the same population
(p i =

m2

= M3)

Alternate hypothesis
Samples drawn from
different populations
(Ml *M2*M3)
12 m456789
10

11
12
13
14
15
16
17
18
19

20
21
22
23
24
25
26
27
28
29
30
31
32
33
34
35
36
37
38
39
40
41
42

t'l

Item: 3 of 46

H^Mark

Q. Id : 8519 [549201]

<O
P re v io u s N e x t

Analysis of variance (ANOVA)


Null hypothesis
All samples drawn from
the same population
(Ml = M2 = m3)
Alternate hypothesis
Samples drawn from
different populations
(p i M2

m3)

Small apparent difference between the means of the samples


null hypothesis not rejected
Large apparent differences between the means of the samples
null hypothesis rejected
Block Tim e Rem aining: 01 : 41
T u to r

IIwm
Lab V a lu e s N o te s C a lc u la to r
AV

WJi o
Feedback Suspend End Block
11 L Mark
MX

Item: 3 of 46
Q. Id : 8519 [549201]

<a

P re v io u s

E>
Next

II
Lab V a lu e s

Bp?-N o te s C a lc u la to r

Large apparent differences between the means of the samples


null hypothesis rejected
U SM L EWorld, LLC

Analysis of variance (AN OVA) is used to determine whether there are any significant differences between the
means of 2 or more independent groups. ANOVA compares the means between the groups relative to the
variability within groups and determines whether any of those means are significantly different from one
another. Specifically, it tests the null hypothesis that all groups are simply random samples of the same
population. The null hypothesis is rejected and the alternative hypothesis is accepted when there are at least
2 group means that are significantly different from one another.
(Choice B) Chi-square tests can be used to check for an association between 2 categorical variables. For
example, a chi-square test could be used to determine if the distribution of gender and smoking status is
random or if there is a difference between the sexes regarding smoking status.
(Choice C) Meta-analysis involves the pooling of data from several studies to perform an analysis having
greater statistical power than the individual studies themselves. For example, individual studies assessing the
effects of aspirin on certain cardiovascular events may be inconclusive. However, analysis of data compiled
from multiple clinical trials may reveal a significant benefit.
(Choice D) Multiple linear regression is a method used to model the linear relationship between a dependent
variable and 2 or more independent variables. For example, this test could be used to quantify the effects of
alcohol use, tobacco smoking, and charred food consumption on the incidence of gastric cancer.
(Choice E) The Pearson correlation coefficient is a measure of the strength and direction of a linear
relationship between 2 variables. For example, a study may report a correlation coefficient describing the
association between hemoglobinA10 levels and average blood glucose levels.

Block Tim e Rem aining: o i : 47


T u to r

Feedback Suspend End Block

Item: 3 of 46

11 F Mark

Q. Id : 8519 [549201] Lab V a lu e s

USMLEWorld, LLC

Ml M2 M3
Analysis of variance (AN OVA) is used to determine whether there are any significant differences between the
means of 2 or more independent groups. ANOVA compares the means between the groups relative to the
variability within groups and determines whether any of those means are significantly different from one
another. Specifically, it tests the null hypothesis that all groups are simply random samples of the same
population. The null hypothesis is rejected and the alternative hypothesis is accepted when there are at least
2 group means that are significantly different from one another.
(Choice B) Chi-square tests can be used to check for an association between 2 categorical variables. For
example, a chi-square test could be used to determine if the distribution of gender and smoking status is
random or if there is a difference between the sexes regarding smoking status.
(Choice C) Meta-analysis involves the pooling of data from several studies to perform an analysis having
greater statistical power than the individual studies themselves. For example, individual studies assessing the
effects of aspirin on certain cardiovascular events may be inconclusive. However, analysis of data compiled
from multiple clinical trials may reveal a significant benefit.
(Choice D) Multiple linear regression is a method used to model the linear relationship between a dependent
variable and 2 or more independent variables. For example, this test could be used to quantify the effects of
alcohol use, tobacco smoking, and charred food consumption on the incidence of gastric cancer.
(Choice E) The Pearson correlation coefficient is a measure of the strength and direction of a linear
relationship between 2 variables. For example, a study may report a correlation coefficient describing the
association between hemoglobinA10 levels and average blood glucose levels.
(Choice F) A two-sample t-test can be used when 2 group means need to be compared. This test could
have been used for the example given in the question if the study participants were divided only into smoking
and nonsmoking groups.
Educational objective:
A t-test is used to compare the difference between the means of 2 groups. Analysis of variance (ANOVA)

compares the difference between the means of 2 or more groups.


Time Spent: 15 seconds Copyright USMLEWorld,LLC. Last updated: [7/22/2014]

Block Tim e Rem aining: o i : 49


T u to r

Feedback Suspend End Block

Item: 4 of 46

11 f M a rk

Q. Id : 1178 [549201]

<a
P re v io u s

E>
Next

II
Lab V a lu e s

Bp*-
N o te s C a lc u la to r

A new test to diagnose urinary tract infections in women is being evaluated. The comparison gold standard is
positive urine dipstick plus urine culture. Results of the study are given below.
Positive urine dipstick Negative urine dipstick
and culture and culture
Test positive 60 20
Test negative 140 180
What is the new test's specificity?

o
o
o
o
o

A. 10%
B. 30%
C. 56%
D. 70%
E. 90%

Block Tim e Rem aining: o i :


T u to r

53

Feedback Suspend End Block

Item: 4 of 46

11 f M a rk

Q. Id : 1178 [549201]

<a
P re v io u s

E>
Next

A new test to diagnose urinary tract infections in women is being evaluated. The comparison gold standard is
positive urine dipstick plus urine culture. Results of the study are given below.
Positive urine dipstick Negative urine dipstick
and culture and culture
Test positive 60 20
Test negative 140 180
What is the new test's specificity?
Lab V a lu e s N o te s C a lc u la to r
A

Explanation: User Id: 549201


The specificity of a test refers to the number of true negatives divided by all people without the disease. It
should be high in confirmatory tests in order to decrease the number of false positives. Using the generic
two-by-two table below specificity may be represented as d/(b+d)
Disease positive Disease negative
Test positive a
True positive
b

False positive
Test negative c
False negative
d
True negative
In this case the specificity equals 180/(180 + 20), or 90%. The false positive ratio is 1-specificity. It is not
affected by disease prevalence.
Educational Objective:
The specificity of a test is given by d/(b+d). It should be high in confirmatory tests in order to decrease false

Block Tim e Rem aining: 02 : 00


T u to r

Feedback Suspend End Block

Item: 4 of 46 1

f^ M a rk < 1 O i l m
Q. Id : 1178 [549201] P re v io u s N e x t Lab V a lu e s N o te s C a lc u la to r

positive urine dipstick plus urine culture. Results of the study are given below.
Positive urine dipstick Negative urine dipstick
and culture and culture
Test positive 60 20
Test negative 140 180
What is the new test's specificity?
Explanation: User Id: 549201
The specificity of a test refers to the number of true negatives divided by all people without the disease. It
should be high in confirmatory tests in order to decrease the number of false positives. Using the generic
two-by-two table below specificity may be represented as d/(b+d)
Disease positive Disease negative
Test positive a
True positive
b
False positive
Test negative c
False negative
d
True negative
In this case the specificity equals 180/(180 + 20), or 90%. The false positive ratio is 1-specificity. It is not
affected by disease prevalence.
Educational Objective:
The specificity of a test is given by d/(b+d). It should be high in confirmatory tests in order to decrease false
positives.
Time Spent: 10 seconds Copyright USMLEWorld,LLC. Last updated: [8/22/2014]

Block Tim e Rem aining: 02 : 03


T u to r

Feedback Suspend End Block

Item: 5 of 46

11 f M a rk

Q. Id : 1183 [549201]

<a
P re v io u s

E>
Next

II
Lab V a lu e s

Bp*-
N o te s C a lc u la to r

A patient in the ICU has twenty serial measurements of pulmonary capillary wedge pressure taken over two
hours. The maximal recorded value is 12 mmHg and the minimal recorded value is 10 mmHg. If the next
measurement is 26 mmHg, which of the following is most likely to remain unchanged?
O A. Mean
O B. Mode
O C. Standard deviation
O D. Variance

O E. Range

Block Tim e Rem aining: 02 : 07


T u to r

Feedback Suspend End Block

Item: 5 of 46

11 f M a rk

Q. Id : 1183 [549201]

<a
P re v io u s

E>
Next

II
Lab V a lu e s

Bp?-N o te s C a lc u la to r

A patient in the ICU has twenty serial measurements of pulmonary capillary wedge pressure taken over two
hours. The maximal recorded value is 12 mmHg and the minimal recorded value is 10 mmHg. If the next
measurement is 26 mmHg, which of the following is most likely to remain unchanged?

A. Mean [6%]
* B. Mode [76%]

O
O
O

C. Standard deviation [8%]


D. Variance [6%]

E. Range [3%]
Explanation: User Id: 549201
An outlier is defined as an extreme and unusual value observed in a dataset. It may be the result of a
recording error, a measurement error, or a natural phenomenon. In this case, the value of 26 mmHg is the
outlier since the other values lie between 10-12mmHg. An outlier can affect the measures of central tendency
as well as the measures of dispersion. For example, the mean is quite sensitive to outliers (Choice A).
(Choice B) The mode, defined as the most commonly observed data point, is much more resistant to outliers.
(Choices C and D) The standard deviation and variance are sensitive to outliers because they are the
measures of dispersion within the dataset. Outliers increase the dispersion of datasets.
(Choice E) The range is equal to the maximal value minus the minimal value and would clearly be affected by
outliers.
Educational Objective:
An outlier is defined as an extreme and unusual observed in a dataset. The mean, standard deviation,
variance, and range are sensitive to outliers. The mode is more resistant to outliers.
Time Spent: 8 seconds Copyright USMLEWorld,LLC. Last updated: [8/22/2014]

Block Tim e Rem aining: 02 : 11


T u to r

Feedback Suspend End Block

Item: 6 of 46

11 f M a rk

Q. Id : 1233 [549201]

<a
P re v io u s

E>
Next

II
Lab V a lu e s

Bp*-
N o te s C a lc u la to r

A 56-year-old female comes to the emergency department because of chest pain and shortness of breath.
You use test X to rule out the possibility of a pulmonary embolus. Her result is negative. Test X has a

specificity of 80% and a sensitivity of 90% when tested in 100 subjects with pulmonary embolus and 100
subjects without pulmonary embolus. Assume that this patient's pretest probability for having a pulmonary
embolus is equivalent to the disease prevalence in the study population. What is the probability that this
patient truly does not have a pulmonary embolus?

o
o
o
o
o

A. 18%
B. 53%
C. 66%
D. 82%
E. 89%

Block Tim e Rem aining: 02 :14


T u to r

Feedback Suspend End Block

Item: 6 of 46

11 f M a rk

Q. Id : 1233 [549201]

<a
P re v io u s

E>
Next

II
Lab V a lu e s

Bp?-N o te s C a lc u la to r

Item: 6 of 46

11 f M a rk

Q. Id : 1233 [549201]

O C. 66% |b%]
O D. 82% [26%]
E. 89% [56%]

<O
P re v io u s N e x t Lab V a lu e s N o te s C a lc u la to r
A

Explanation: User Id: 549201


This question asks for the negative predictive value (NPV) of test X. In order to determine the NPV, it helps to
create a two-by-two table. The question tells us that there are 100 subjects with pulmonary embolus (PE) and
100 subjects without PE, and to assume that the patient's pre-test probability (essentially the disease
prevalence modified by the physician's clinical judgment) is equal to the disease prevalence in the test
population. Knowing that specificity = d/[b+d] (true negatives/total disease negatives) and sensitivity = a/[a+c]
(true positives/total disease positives), we can construct the following two-by-two table:

l=>
Test X
+ PE -P E
Test X
positive
90 20 110
Test X
negative
10 80 90
100 100
Generic
Disease
positive
Disease
neqative

Test
positive
a
True
positive
b
False
positive
a+b
Test
negative
c
False
negative
d
True
negative
c+d
a+c b+d
The NPV is the probability that a patient truly does not have a disease if she tests negative for the disease.
Thus, NPV = d/[c+d] (true negatives/total negative tests). In this study, the NPV = 80/(10+80), or -89%.
Unlike sensitivity and specificity, NPV varies based upon disease prevalence and is inversely proportional to
the prevalence of a disease. For example, NPV decreases as the prevalence of a disease increases.
Educational objective:
Negative predictive value (NPV) represents the probability of not having a disease given a negative test result.
Unlike sensitivity and specificity, NPV varies based upon disease prevalence and is inversely proportional to
the prevalence of a disease.
Time Spent: 11 seconds Copyright USMLEWorld,LLC. Last updated: [3/30/2014]

Item: 7 of 46

11 f M a rk

Q. Id : 1206 [549201]

<a
P re v io u s

E>
Next

The figure below shows the incidence and mortality for the three most common cancers affecting women
living in the United States during 2011. The x-axis lists the specific cancers, and the y-axis shows the number
of patients affected.
250,000
200,000

150,000
100,000

50,000
0
Cancer 1 Cancer 2 Cancer 3
Incidence
Deaths
Which of the following lists of cancers best corresponds to cancers 1, 2, and 3 respectively?
Lab V a lu e s N o te s C a lc u la to r
A

O
O
O
O
O

A. Breast, lung, colon


B. Lung, breast, colon
C. Breast, ovary, lung
D. Lung, breast, cervix
E. Breast, colon, lung

Block Tim e Rem aining: 02 : 34


T u to r

Feedback Suspend

O
End B lo ck

Item: 7 of 46
Q. Id : 1206 [549201]

<a
P re v io u s

E>
Next

11 f M a rk

The figure below shows the incidence and mortality for the three most common cancers affecting women
living in the United States during 2011. The x-axis lists the specific cancers, and the y-axis shows the number
of patients affected.
250,000
200,000

150,000
100,000

50,000
0
Cancer 1 Cancer 2 Cancer 3
Incidence
Deaths
Which of the following lists of cancers best corresponds to cancers 1, 2, and 3 respectively?
Lab V a lu e s N o te s C a lc u la to r
A

* A. Breast, lung, colon [83%]

O
O
O
O

B. Lung, breast, colon [9%]


C. Breast, ovary, lung [3%]
D. Lung, breast, cervix [3%]
E. Breast, colon, lung [2%]

Block Tim e Rem aining: 02 : 42


T u to r

Feedback Suspend End Block

Item: 7 of 46

11 f M a rk

Q. Id : 1206 [549201]

<a
P re v io u s

E>
Next

II
Lab V a lu e s

Bp?-N o te s C a lc u la to r

Explanation: User Id: 549201


According to the American Cancer Society's Cancer Facts & Figures 2011, the most common cancers
among women living in the United States during 2011 were breast cancer, lung cancer, and colon cancer. In
order to arrange the cancer statistics in the correct order, a basic knowledge of cancer incidence and their
corresponding mortality is needed. Aside from skin cancers, breast cancer is the most common cancer
among women in the United States, but its mortality rate is moderately low. In fact, 230,480 breast cancers
were diagnosed in the United States during 2011, but only 39,520 breast cancer deaths were
reported. Despite the fact that the incidence of lung cancer is much lower than that for breast cancer,
mortality from lung cancer is higher. Lung cancer has emerged as the leading cause of cancer mortality in
women.
250,000
200,000

150,000
100,000

50,000
Breast Lung Colon
(Choices C and D) The incidences of ovarian and cervical cancers are much lower than that of the three
cancers presented on the slide.
V

Block Tim e Rem aining: 02 : 4 7 I


T u to r

Feedback Suspend End Block

230,48 0
Incidence
106,070

1
Item: 7 of 46

39,5201

71'340 69,36024430 Deaths

11 f M a rk

Q. Id : 1206 [549201]

200,000

Incidence
Deaths
(Choices C and D) The incidences of ovarian and cervical cancers are much lower than that of the three
cancers presented on the slide.
(Choice E) The incidence of colon cancer is somewhat close to that of lung, but the associated mortality is
relatively much lower.
Educational objective:
According to 2011 statistics, the most common cancers (aside from skin cancer) in women in order of
incidence were breast, lung, and colon cancer. In terms of mortality, lung cancer claimed the most lives,
followed by breast and then colon cancer.
References:
1. Breast cancer statistics, 2011.
Time Spent: 19 seconds Copyright USMLEWorld,LLC. Last updated: [7/21/2014]

Block Tim e Rem aining: 02 : 50


T u to r

Feedback Suspend End Block

Item: 8 of 46

11 f M a rk

Q. Id : 1273 [549201]

<a
P re v io u s

E>
Next

II
Lab V a lu e s

Bp*-
N o te s C a lc u la to r

A geneticist is interested in the potential causes of a congenital abnormality. She selects the mothers of
children with and without the abnormality randomly from a national birth registry. When interviewing the
mothers of children with the abnormality, she discovers that several of them used acetaminophen during
pregnancy. Mothers of children who do not have the abnormality report that they did not take acetaminophen
as frequently. This type of investigation is most susceptible to which of the following types of bias?

O
O
O
O
O
O

A. Referral bias
B. Detection bias
C. Lead-time bias
D. Allocation bias
E. Recall bias
F. Sampling bias

Block Tim e Rem aining: 02 : 54


T u to r

Feedback Suspend End Block

Item: 8 of 46

11 L Mark

Q. Id : 1273 [549201]

<a
P re v io u s

E>
Next

A geneticist is interested in the potential causes of a congenital abnormality. She selects the mothers of
children with and without the abnormality randomly from a national birth registry. When interviewing the
mothers of children with the abnormality, she discovers that several of them used acetaminophen during
pregnancy. Mothers of children who do not have the abnormality report that they did not take acetaminophen
as frequently. This type of investigation is most susceptible to which of the following types of bias?
Lab V a lu e s N o te s C a lc u la to r
A

O
O
O
O

A. Referral bias [1 %]
B. Detection bias [6%]
C. Lead-time bias [2%]

D. Allocation bias [2%]


* E. Recall bias [77%]

F. Sampling bias [11%]


Explanation: User Id: 549201
Recall bias results from inaccurate recall of past exposure by people in the study and applies mostly to
retrospective studies such as case-control studies. People who have suffered an adverse event (such as
having a child with congenital abnormalities) are more likely to recall previous risk factors than are people who
have not experienced a poor outcome. The risk is known at the time of enrollment in prospective studies, so
recall bias is eliminated.
(Choice A) Referral (admission rate) bias occurs when the case and control populations differ due to
admission or referral practices. For instance, a study involving cancer risk factors performed at a hospital
specializing in cancer research may enroll cases referred from all over the nation. However, hospitalized
control subjects without cancer may come from only the local area.
(Choice B) Detection bias refers to the fact that a risk factor itself may lead to extensive diagnostic
investigation and increase the probability that a disease is identified. For instance, patients who smoke may
undergo increased imaging surveillance due to their smoking status, which would detect more cases of
cancer in general.
(Choice C) Lead-time bias occurs when a screening test diagnoses a disease earlier than it would have

Block Tim e Rem aining:


T u to r

03 : 07

qp o

Feedback Suspend End Block

Item: 8 of 46

11 F M a rk

Q. Id : 1273 [549201]

<a
P re v io u s

E>
Next

II
Lab V a lu e s

Bp?-N o te s C a lc u la to r

Recall bias results from inaccurate recall of past exposure by people in the study and applies mostly to
retrospective studies such as case-control studies. People who have suffered an adverse event (such as
having a child with congenital abnormalities) are more likely to recall previous risk factors than are people who
have not experienced a poor outcome. The risk is known at the time of enrollment in prospective studies, so
recall bias is eliminated.
(Choice A) Referral (admission rate) bias occurs when the case and control populations differ due to
admission or referral practices. For instance, a study involving cancer risk factors performed at a hospital

specializing in cancer research may enroll cases referred from all over the nation. However, hospitalized
control subjects without cancer may come from only the local area.
(Choice B) Detection bias refers to the fact that a risk factor itself may lead to extensive diagnostic
investigation and increase the probability that a disease is identified. For instance, patients who smoke may
undergo increased imaging surveillance due to their smoking status, which would detect more cases of
cancer in general.
(Choice C) Lead-time bias occurs when a screening test diagnoses a disease earlier than it would have
appeared by natural history so that the time from diagnosis until death is prolonged. Affected studies may not
accurately reflect an improvement in survival or alteration of the natural history.
(Choice D) Allocation bias can result from the way that treatment and control groups are assembled. It may
occur if subjects are assigned to the study groups of a clinical trial in a non-random fashion. For instance, in
a study comparing oral NSAIDs and intraarticular corticosteroid injections for the treatment of osteoarthritis,
obese patients may be preferentially assigned to the corticosteroid group.
(Choice F) Sampling bias occurs due to a non-random sampling of a population. It can lead to a study
population having characteristics that differ from the target population. A common example is that severely ill
patients are the most likely to enroll in cancer trials, leading to results that are not applicable to patients with
less advanced cancers.
Educational objective:
Recall bias results from inaccurate recall of past exposure by people in the study and applies mostly to
retrospective studies such as case-control studies. People who have suffered an adverse event are more
likely to recall risk factors than those without adverse experiences. Like all sources of bias, recall bias is a
threat to the validity of a study.
Time Spent: 22 seconds Copyright USMLEWorld,LLC. Last updated: [4/19/2014]

Block Tim e Rem aining:


T u to r

03

:1 1

qp o

Feedback Suspend End Block

Item: 9 of 46

11 f M a rk

Q. Id : 1190 [549201]

<a
P re v io u s

E>
Next

II
Lab V a lu e s

Bp*-
N o te s C a lc u la to r

A 34-year-old female who recently emigrated from Japan presents to your office with a palpable thyroid node.
You proceed with node aspiration that reveals no malignant cells. As you are explaining the test result, the
patient asks, "What are the chances that I really do not have cancer?" Which of the following epidemiologic
parameters best answers this patient's question?
O A. Sensitivity
O B. Specificity
O C. Positive predictive value
O D. Negative predictive value
O E. Validity

Block Tim e Rem aining:


T u to r

03 : 1 4

Feedback Suspend End Block

Item: 9 of 46

11 f Mark

Q. Id : 1190 [549201]

<a
P re v io u s

E>
Next

A 34-year-old female who recently emigrated from Japan presents to your office with a palpable thyroid node.
You proceed with node aspiration that reveals no malignant cells. As you are explaining the test result, the
patient asks, "What are the chances that I really do not have cancer?" Which of the following epidemiologic
parameters best answers this patient's question?

Lab V a lu e s N o te s C a lc u la to r
A

O
O
O

A. Sensitivity [8%]
B. Specificity [15%]

C. Positive predictive value [6%]


* D. Negative predictive value [69%]

E. Validity [1%]
Explanation: User Id: 549201
The negative predictive value (NPV) is defined as the probability of being free of a disease if the test result is
negative. It is important to remember that the NPV will vary with the pretest probability of a disease. A patient
with a high probability of having a disease will have a low NPV if a negative test, whereas a patient with a low
probability of having a disease will have a high NPV with a negative test. Specific examples are given below:
1. Thyroid cancer and FNA test results: A patient with a high pre-test probability for having thyroid cancer
(e g., young age, radiation exposure) has a low NPV.
2. HIV and ELISA test results: A patient who belongs to a high-risk group (e.g., multiple sexual partners,
admits to not using condoms, IV drug user) has a high pre-test probability; consequently, this patient
will have a low NPV. On the other hand, a patient who belongs to a low-risk group (e.g., one
monogamous sexual partner, consistent condom use, no history of IV drug use) has a low pre-test
probability; consequently, this patient will have a high NPV.
*Note: The prevalence of a disease is directly related to the pre-test probability of having the disease and,
thus, also affects the NPV.
(Choices A and B) The sensitivity and specificity of a test are fixed values which do not vary with the pretest
probability of a disease. Most researchers agree that the ideal diagnostic test should have high sensitivity and
specificity. In this case, FNA also has a high sensitivity and specificity; however, the statistical parameter

Block Tim e Rem aining:


T u to r

03

: 21

Feedback Suspend End Block

Item: 9 of 46

11 L Mark

Q. Id : 1190 [549201]

<a
P re v io u s

E>
Next

II
Lab V a lu e s

Bp?-N o te s C a lc u la to r

Explanation: User Id: 549201


The negative predictive value (NPV) is defined as the probability of being free of a disease if the test result is
negative. It is important to remember that the NPV will vary with the pretest probability of a disease. A patient
with a high probability of having a disease will have a low NPV if a negative test, whereas a patient with a low
probability of having a disease will have a high NPV with a negative test. Specific examples are given below:
1. Thyroid cancer and FNA test results: A patient with a high pre-test probability for having thyroid cancer
(e.g., young age, radiation exposure) has a low NPV.
2. HIV and ELISA test results: A patient who belongs to a high-risk group (e.g., multiple sexual partners,
admits to not using condoms, IV drug user) has a high pre-test probability; consequently, this patient
will have a low NPV. On the other hand, a patient who belongs to a low-risk group (e.g., one
monogamous sexual partner, consistent condom use, no history of IV drug use) has a low pre-test
probability; consequently, this patient will have a high NPV.
*Note: The prevalence of a disease is directly related to the pre-test probability of having the disease and,
thus, also affects the NPV.
(Choices A and B) The sensitivity and specificity of a test are fixed values which do not vary with the pretest
probability of a disease. Most researchers agree that the ideal diagnostic test should have high sensitivity and
specificity. In this case, FNA also has a high sensitivity and specificity; however, the statistical parameter
being described by the patient and physician was the high negative predictive value.
(Choice C) The positive predictive value follows the same concept but applies if the test result is positive.

(Choice

E) Validity represents the appropriateness of the test (i.e., the test measures what it is supposed to
measure). It does not depend on the pretest probability of the disease.

Educational Objective:
NPV is the probability of being free of a disease if the test result is negative. Remember that the NPV will vary
with the pretest probability of a disease. A patient with a high probability of having a disease will have a low
NPV with a negative test but a patient with a low probability of having a disease will have a high NPV with a
negative test.
Time Spent: 12 seconds Copyright USMLEWorld,LLC. Last updated: [8/22/2014] V

Block Tim e Rem aining:


T u to r

03 : 24

Feedback Suspend End Block

Item: 10 of 46

11 f M a rk

Q. Id : 1285 [549201]

<a
P re v io u s

E>
Next

II
Lab V a lu e s

Bp?-N o te s C a lc u la to r

A screening test using a new serum marker is developed for diagnosing ovarian cancer Healthy and
diseased population curves along the screening dimension, along with the proposed serum marker cutoff
value, are given below.
Compared to the solid curves, the dashed curves are associated with:

O
O
O
O
O

A. Higher sensitivity and higher specificity


B. Higher sensitivity and lower specificity
C. Higher sensitivity and same specificity
D. Lower sensitivity and higher specificity
E. Lower sensitivity and lower specificity

Block Tim e Rem aining: 03 : 29 O


T u to r

Feedback Suspend End Block

Item: 10 of 46

11 f M a rk

Q. Id : 1285 [549201]

<a
P re v io u s

E>
Next

II
Lab V a lu e s

Bp?-N o te s C a lc u la to r

A screening test using a new serum marker is developed for diagnosing ovarian cancer Healthy and
diseased population curves along the screening dimension, along with the proposed serum marker cutoff
value, are given below.
Compared to the solid curves, the dashed curves are associated with:
* A. Higher sensitivity and higher specificity [61 %]
O B. Higher sensitivity and lower specificity [14%]
O C. Higher sensitivity and same specificity [3%]
O D. Lower sensitivity and higher specificity [15%]
O E. Lower sensitivity and lower specificity [6%]
Explanation: User Id: 549201

--------Healthy

Block Tim e Rem aining:


T u to r

03 : 35

Feedback Suspend End Block

Item: 10 of 46

11 ^Mark

Q. Id : 1285 [549201]

<a
P re v io u s

E>
Next

User Id: 549201


Lab V a lu e s N o te s C a lc u la to r
A

FN = false negative, FP = false positive, TN = true negative. TP = true positive


The degree of overlap between the healthy and diseased population curves limits the maximum combined
sensitivity and specificity of a test. In this example, the cutoff value is optimally placed to maximize both
sensitivity and specificity. Sensitivity represents the ability of a test to identify those with a given disease. It is
calculated as TP divided by the number of people with the disease (TP + FN). Specificity represents the
ability of a test to exclude those from having a given disease. It is calculated as TN divided by the number of
people without the disease (TN + FP). In this specific instance, decreased overlap between the healthy and
diseased population curves decreases both the number of FP and FN, and thus allow for a test with both
higher sensitivity and specificity. Therefore, the dashed curves are associated with higher sensitivity and
specificity.
Depending upon the disease or condition being tested for, sensitivity or specificity may be preferred and the
cutoff value adjusted accordingly. In the above example, moving the cutoff value to the right (higher value)
would increase specificity at the expense of sensitivity, while moving the cutoff to the left (lower value) would
increase sensitivity at the expense of specificity. A cutoff value just outside the overlapping portion would
maximize the sensitivity OR specificity at 100%. However, the corresponding specificity/sensitivity of such a
maximized test is dependent on the degree of overlap between the healthy and diseased population curves.
Educational Objective:
The degree of overlap between the healthy and diseased population curves limits the maximum combined

Block Tim e Rem aining:


T u to r

03 : 39

mo

Feedback Suspend End Block

Item: 10 o f 46

11 f Mark

Q. Id : 1285 [549201]

<a
P re v io u s

E>
Next

II
Lab V a lu e s

Bp?-N o te s C a lc u la to r

FN = false negative, FP = false positive, TN = true negative, TP = true positive


The degree of overlap between the healthy and diseased population curves limits the maximum combined
sensitivity and specificity of a test. In this example, the cutoff value is optimally placed to maximize both
sensitivity and specificity. Sensitivity represents the ability of a test to identify those with a given disease. It is
calculated as TP divided by the number of people with the disease (TP + FN). Specificity represents the
ability of a test to exclude those from having a given disease. It is calculated as TN divided by the number of
people without the disease (TN + FP). In this specific instance, decreased overlap between the healthy and
diseased population curves decreases both the number of FP and FN, and thus allow for a test with both
higher sensitivity and specificity. Therefore, the dashed curves are associated with higher sensitivity and
specificity.
Depending upon the disease or condition being tested for, sensitivity or specificity may be preferred and the
cutoff value adjusted accordingly. In the above example, moving the cutoff value to the right (higher value)
would increase specificity at the expense of sensitivity, while moving the cutoff to the left (lower value) would
increase sensitivity at the expense of specificity. A cutoff value just outside the overlapping portion would
maximize the sensitivity OR specificity at 100%. However, the corresponding specificity/sensitivity of such a

maximized test is dependent on the degree of overlap between the healthy and diseased population curves.
Educational Objective:
The degree of overlap between the healthy and diseased population curves limits the maximum combined
sensitivity and specificity of a test (the area under its ROC curve). The degree to which sensitivity or
specificity is affected depends upon the chosen cutoff value.
Time Spent: 15 seconds Copyright USMLEWorld,LLC. Last updated: [8/7/2014]

Block Tim e Rem aining:


T u to r

03 : 43

mo

Feedback Suspend End Block

Item: 11 o f 46

11 f M a rk

Q. Id : 1169 [549201]

<a
P re v io u s

E>
Next

II
Lab V a lu e s

Bp*-
N o te s C a lc u la to r

A new biomarker has been shown to allow for the early detection of invasive gastric cancer. It has a
sensitivity of 89% and a specificity of 85% when compared to endoscopy with multiple biopsies. The test is
used in two groups: a population in the United States, where 5 out of 100,000 people have gastric cancer; and
a population in China, where 100 out of 100,000 people suffer from the disease. Which of the following is the
most accurate statement concerning this new test?

O
O
O
O
O

A. Sensitivity of the test is higher in the American population


B. Specificity of the test is higher in the Chinese population
C. Positive predictive value of the test is higher in the Chinese population
D. Negative predictive value of the test is lower in the American population
E. The test is not reliable in the American population

Block Tim e Rem aining:


T u to r

03

46

Feedback Suspend End Block

Item: 11 o f 46

11 f Mark

Q. Id : 1169 [549201]

<a
P re v io u s

E>
Next

A new biomarker has been shown to allow for the early detection of invasive gastric cancer. It has a
sensitivity of 89% and a specificity of 85% when compared to endoscopy with multiple biopsies. The test is
used in two groups: a population in the United States, where 5 out of 100,000 people have gastric cancer; and
a population in China, where 100 out of 100,000 people suffer from the disease. Which of the following is the
most accurate statement concerning this new test?
Lab V a lu e s N o te s C a lc u la to r
A

O
O

A. Sensitivity of the test is higher in the American population [4%]

O
O

D. Negative predictive value of the test is lower in the American population [5%]

B. Specificity of the test is higher in the Chinese population [6%]


*/ () C. Positive predictive value of the test is higher in the Chinese population [80%]
E. The test is not reliable in the American population [5%]
Explanation: User Id: 549201
Traditionally, the positive predictive value has been defined as the proportion of people with positive test
results who actually have the disease: (True Positives) / (True + False Positives). It is easier to understand
the concept in terms of probability: if a patient has a positive test result, what is the likelihood that he actually
has the disease? Positive predictive value depends on the prevalence of a specific disease in the population
being tested, which is a very important concept to remember. The more common the disease is in the
population, the more likely a patient with a positive test result actually is diseased (a "true positive"). If the

disease is relatively common - as gastric cancer is in China - the probability is reasonably high that a patient
who tests positive actually has the disease. If the disease is relatively uncommon - as gastric cancer is in the
United States - the probability is far lower that a patient who tests positive actually has the disease.
(Choices A and B) The sensitivity and specificity of a test do not depend on the prevalence of the disease in
the population. These characteristics are instead inherent to the test itself.
(Choice D) Like positive predictive value, the negative predictive value depends on the prevalence of the
disease in the population. Instead, however, negative predictive value has an inverse association with the
prevalence. As the prevalence of the disease increases, the negative predictive value decreases because the
probability of a true negative result for a patient who tests positive is high in a population with low prevalence of
the disease (as seen with gastric cancer in the United States).
V

Block Tim e Rem aining:


T u to r

03 : 51

Wfk o

Feedback Suspend End Block

Item: 11 o f 46

11 ^ M a r k

Q. Id : 1169 [549201]

<a
P re v io u s

E>
Next

II
Lab V a lu e s

Bp?-N o te s C a lc u la to r

B. Specificity of the test is higher in the Chinese population [6%]


* C. Positive predictive value of the test is higher in the Chinese population [80%]

O
O

D. Negative predictive value of the test is lower in the American population [5%]

E. The test is not reliable in the American population [5%]


Explanation: User Id: 549201
Traditionally, the positive predictive value has been defined as the proportion of people with positive test
results who actually have the disease: (True Positives) / (True + False Positives). It is easier to understand
the concept in terms of probability: if a patient has a positive test result, what is the likelihood that he actually
has the disease? Positive predictive value depends on the prevalence of a specific disease in the population
being tested, which is a very important concept to remember The more common the disease is in the
population, the more likely a patient with a positive test result actually is diseased (a "true positive"). If the
disease is relatively common - as gastric cancer is in China - the probability is reasonably high that a patient
who tests positive actually has the disease. If the disease is relatively uncommon - as gastric cancer is in the
United States - the probability is far lower that a patient who tests positive actually has the disease.
(Choices A and B) The sensitivity and specificity of a test do not depend on the prevalence of the disease in
the population. These characteristics are instead inherent to the test itself.
(Choice D) Like positive predictive value, the negative predictive value depends on the prevalence of the
disease in the population. Instead, however, negative predictive value has an inverse association with the
prevalence. As the prevalence of the disease increases, the negative predictive value decreases because the
probability of a true negative result for a patient who tests positive is high in a population with low prevalence of
the disease (as seen with gastric cancer in the United States).
(Choice E) Reliability is defined as the provision of identical measurements on different occasions separated
by a short interval of time (ie, how reproducible the measurement is). No information is provided to suggest
that this gastric cancer biomarker is not reliable.
Educational Objective:
The positive and negative predictive values of a test depend on the disease prevalence in the population. The
sensitivity and specificity of a test do not depend on the prevalence of the disease in the population.
Time Spent: 8 seconds Copyright USMLEWorld,LLC. Last updated: [8/22/2014]

Block Tim e Rem aining:


T u to r

Feedback Suspend End Block

Item: 12 o f 46
Q. Id : 1192 [549201]

11 T M a rk

03 :

54

qp o

<a
P re v io u s

E>
Next

II
Lab V a lu e s

Bp*-
N o te s C a lc u la to r

A new serological test has been developed for the detection of active pulmonary tuberculosis and is compared
to the gold standard of sputum mycobacterial culture. 1000 subjects are randomly selected for testing from a
population with a high prevalence of tuberculosis. Results of the study are given below.
Sputum culture results
Serological test results Positive Negative Total
Positive 130 60 190
Negative 50 760 810
Total 180 820 1000
Which of the following is the positive predictive value of the test under study?

O
O
O
O
O

A. 130/180
B. 130/190
C. 50/810
D. 760/810
E. 60/190

Block Tim e Rem aining:


T u to r

03

57

Feedback Suspend End Block

Item: 12 o f 46

11 ^Mark

Q. Id : 1192 [549201]

<a
P re v io u s

E>
Next

A new serological test has been developed for the detection of active pulmonary tuberculosis and is compared
to the gold standard of sputum mycobacterial culture. 1000 subjects are randomly selected for testing from a
population with a high prevalence of tuberculosis. Results of the study are given below.
Sputum culture results
Serological test results Positive Negative Total
Positive 130 60 190
Negative 50 760 810
Total 180 820 1000
Lab V a lu e s N o te s C a lc u la to r
A

Which of the following is the positive predictive value of the test under study?
O A. 130/180 [18%]
* B. 130/190 [78%]
O C. 50/810 [1%]
O D. 760/810 [1%]
O E. 60/190 [1%]
Explanation: User Id: 549201
Predictive values are of prime importance to physicians. In clinical practice, patients will more often present
with a positive or negative test than with a defined diseased or disease-free state. Therefore, one requires
sufficient knowledge of the predictive values of a test.
Positive predictive value is defined as the proportion of subjects with a positive test who actually have the
disease. Negative predictive value is defined as the proportion of subjects with a negative test who are truly
free of disease. Consider the following 2 x 2 table:

Item: 12 o f 46
Q. Id : 1192 [549201]

O C. 50/810 [1%]
O D. 760/810 [1%]
O E. 60/190 [1%]

11 f M a rk

<l>
P re v io u s N e x t Lab V a lu e s N o te s C a lc u la to r
A

Explanation: User Id: 549201


Predictive values are of prime importance to physicians. In clinical practice, patients will more often present
with a positive or negative test than with a defined diseased or disease-free state. Therefore, one requires
sufficient knowledge of the predictive values of a test.
Positive predictive value is defined as the proportion of subjects with a positive test who actually have the
disease. Negative predictive value is defined as the proportion of subjects with a negative test who are truly
free of disease. Consider the following 2 x 2 table:________________________________
Disease
Test results Present Absent Total
Positive A
True positive (TP)
B
False positive (FP) A+B
Negative C
False Negative (FN)
D
True Negative (TN) C+D
Total A+C B+D A+B+C+D
Positive predictive value = TP/TP+FP (A/A+B)
Negative predictive value = TN/TN+FN (D/C+D)
In this case, the PPV is 130 / 190.
(Choice A) Sensitivity and specificity are useful for assessing the validity of a test. 130/180 is the sensitivity
of the test.
(Choice D) 760/810 is the negative predictive value of the test.
Educational Objective:
Positive predictive value is defined as the proportion of subjects with a positive test that actually have the
disease.
Time Spent: 10 seconds Copyright USMLEWorld,LLC. Last updated: [8/22/2014]

Block Tim e Rem aining:


T u to r

04

: 06

Feedback Suspend End Block

Item: 13 o f 46

11 f M a rk

Q. Id : 1302 [549201]

<a
P re v io u s

E>
Next

II
Lab V a lu e s

Bp*-
N o te s C a lc u la to r

A researcher is interested in how often primary care physicians take the sexual histories of their patients.
Patients are asked to fill out a questionnaire immediately upon exiting a doctor's office. Once the physicians
are aware of the study, which of the following is a potential problem?
O A. Berkson's bias
O B. Pygmalion effect
O C. Hawthorne effect
O D. Lead-time bias
O E. Recall bias

Block Tim e Rem aining:


T u to r

Feedback Suspend End Block

Item: 13 o f 46
Q. Id : 1302 [549201]

<a

11 L Mark

04

09

P re v io u s

E>
Next

A researcher is interested in how often primary care physicians take the sexual histories of their patients.
Patients are asked to fill out a questionnaire immediately upon exiting a doctor's office. Once the physicians
are aware of the study, which of the following is a potential problem?
Lab V a lu e s N o te s C a lc u la to r
A

O A. Berkson's bias [10%]


O B. Pygmalion effect [20%]
* C. Hawthorne effect [58%]
O D. Lead-time bias [5%]
O E. Recall bias [6%]
Explanation: User Id: 549201
The Hawthorne effect is defined as the tendency of a study population to affect an outcome due to the
knowledge of being studied. This awareness leads to a change in behavior while under observation, thereby
seriously affecting the validity of the study. The Hawthorne effect is commonly seen in studies concerning
behavioral outcomes or outcomes that can be influenced by behavioral changes. In order to minimize the
Hawthorne effect, studied subjects can be kept unaware that they are being studied. This can occasionally
pose ethical problems.
(Choice A) Berkson's bias refers to selection bias created by selecting hospitalized patients as the control
group.
(Choice B) Pygmalion effect describes researcher's beliefs in the efficacy of treatment that can potentially
affect the outcome.
(Choice D) Lead-time bias refers to apparent prolongation of survival after applying a screening test without
any real effect on prognosis.
(Choice E) Recall bias results from inaccurate recall of past exposures by patients. It is unlikely in this case
since the patients fill out a form immediately upon leaving the doctor's office.
Educational Objective:
The Hawthorne effect is the tendency of a study population to affect an outcome due to the knowledge of v

Block Tim e Rem aining: 04 : 3 1


T u to r

Feedback Suspend End Block

Item: 13 of 46

il

1 f M a rk < 1 O
m
Q. Id : 1302 [549201] P re v io u s N e x t Lab V a lu e s N o te s C a lc u la to r

Patients are asked to fill out a questionnaire immediately upon exiting a doctor's office. Once the physicians
are aware of the study, which of the following is a potential problem?

O
O

A. Berkson's bias [10%]

O
O

D. Lead-time bias [5%]

B. Pygmalion effect [20%]


* C. Hawthorne effect [58%]
E. Recall bias [6%]
Explanation: User Id: 549201
The Hawthorne effect is defined as the tendency of a study population to affect an outcome due to the
knowledge of being studied. This awareness leads to a change in behavior while under observation, thereby
seriously affecting the validity of the study. The Hawthorne effect is commonly seen in studies concerning
behavioral outcomes or outcomes that can be influenced by behavioral changes. In order to minimize the
Hawthorne effect, studied subjects can be kept unaware that they are being studied. This can occasionally
pose ethical problems.
(Choice A) Berkson's bias refers to selection bias created by selecting hospitalized patients as the control
group.
(Choice B) Pygmalion effect describes researcher's beliefs in the efficacy of treatment that can potentially
affect the outcome.
(Choice D) Lead-time bias refers to apparent prolongation of survival after applying a screening test without
any real effect on prognosis.
(Choice E) Recall bias results from inaccurate recall of past exposures by patients. It is unlikely in this case
since the patients fill out a form immediately upon leaving the doctor's office.
Educational Objective:
The Hawthorne effect is the tendency of a study population to affect an outcome due to the knowledge of
being studied.
Time Spent: 37 seconds Copyright USMLEWorld,LLC. Last updated: [8/22/2014]

Block Tim e Rem aining: 04

:38

qp o

T u to r

F e e d b a c k Suspend End B lo ck

Item: 14 o f 46

11 f M a rk

Q. Id : 1191 [549201]

<a
P re v io u s

E>
Next

II
Lab V a lu e s

Bp*-
N o te s C a lc u la to r

A serum biomarker level is being used for early detection of pancreatic carcinoma. Test results in 200 healthy
volunteers and 190 patients with biopsy-proven pancreatic cancer are given in the figure below.
Setting the cutoff value of the serum biomarker level at point X instead of point Y would most likely result in:

O
O
O
O
O

A. Lower sensitivity
B. Lower number of true positives
C. Lower number of false positives
D. Lower positive predictive value
E. Higher number of false negatives

Block Tim e Rem aining: 04


T u to r

: 56

Feedback Suspend End Block

Item: 14 o f 46

11 f M a rk

Q. Id : 1191 [549201]

<a
P re v io u s

E>
Next

II
Lab V a lu e s

Bp?-N o te s C a lc u la to r

A serum biomarker level is being used for early detection of pancreatic carcinoma. Test results in 200 healthy
volunteers and 190 patients with biopsy-proven pancreatic cancer are given in the figure below.
Setting the cutoff value of the serum biomarker level at point X instead of point Y would most likely result in:

O
O
O

A. Lower sensitivity [11 %]


B. Lower number of true positives [10%]

C. Lower number of false positives [19%]


() D. Lower positive predictive value [43%]

E. Higher number of false negatives [16%]


Explanation: User Id: 549201
It is important to first recall the definitions of the parameters of diagnostic tests in order to correctly answer
biostatistics questions.
True positive (TP) - represents the people who have the disease and obtained positive test results.
False negative (FN) - represents the people who have the disease but obtained negative test results (meaning
the test failed to identify these people as sick!).

Block Tim e Rem aining:

05

: 02

T u to r

Feedback Suspend End Block

Item: 14 o f 46

11 L Mark

Q. Id : 1191 [549201]

<a
P re v io u s

E>
Next

It is important to first recall the definitions of the parameters of diagnostic tests in order to correctly answer
biostatistics questions.
True positive (TP) - represents the people who have the disease and obtained positive test results.
False negative (FN) - represents the people who have the disease but obtained negative test results (meaning
the test failed to identify these people as sick!).
Positive predictive value (PPV) - represents a test's ability to correctly identify those with the disease from all
those who had positive results. It is the fraction of those truly with the disease among those with positive
results. PPV is dependent on the prevalence of the disease.
Sensitivity - represents the ability of a test to rule out those with the disease. The sensitivity of a test is very
important for screening purposes. A test with high sensitivity is one which identifies most patients with the
disease. For a high sensitivity test most sick patients will have a positive test result. Sensitivity is not
dependent on prevalence.
Specificity - represents the ability of a test to exclude those without the disease. A very specific test is one
which has a low false positive rate. In a high specificity test most healthy patients will have a negative test
result. Specificity is not dependent on prevalence.
The relationships of these parameters are demonstrated in the equations below:
PPV = TP/ (TP + FP)
Sensitivity = TP/ (TP +FN)
Specificity = TN/ (TN + FP)
A lower cutoff point for the serum biomarker level will increase the number of positive results and,
consequently, increase the sensitivity of the test (Choice A). The true positives will also increase (Choice
B); however, the false positives will have a relatively larger increase (Choice C). This results in a decrease in
the PPV and the FN (Choice E).
Educational Objective:
Lowering the cut-off point will increase the sensitivity of a test. The true positives will also increase but the
false positives will have a relatively larger increase. This results in a decrease in the PPV and the FN.
Lab V a lu e s N o te s C a lc u la to r
A

Time Spent: 16 seconds Copyright USMLEWorld,LLC. Last updated: [8/22/2014]

Block Tim e Rem aining:


T u to r

05 : 06

Feedback Suspend End Block

Item: 15 o f 46

11 f M a rk

Q. Id : 1170 [549201]

<a
P re v io u s

E>
Next

II
Lab V a lu e s

Bp*-
N o te s C a lc u la to r

A new biomarker has been shown to allow for the early detection of non-small cell lung carcinoma. A study of
this new test demonstrates that its use prolongs survival of lung cancer patients by three months when
compared to the survival of those subjects diagnosed by conventional methods. The researchers conclude
that the use of this new biomarker improves prognosis in patients with non-small cell lung cancer. Which of
the following is a potential problem with this conclusion?

O
O
O
O
O

A. Observer bias
B. Measurement bias
C. Rare disease assumption
D. Confounding
E. Lead-time bias

Block Tim e Rem aining:


T u to r

05

09

Feedback Suspend End Block

Item: 15 o f 46

11 ^ M a r k

Q. Id : 1170 [549201]

<a
P re v io u s

E>
Next

II
Lab V a lu e s

Bp*--N o te s C a lc u la to r

A new biomarker has been shown to allow for the early detection of non-small cell lung carcinoma. A study of
this new test demonstrates that its use prolongs survival of lung cancer patients by three months when
compared to the survival of those subjects diagnosed by conventional methods. The researchers conclude
that the use of this new biomarker improves prognosis in patients with non-small cell lung cancer. Which of
the following is a potential problem with this conclusion?

O
O
O
O

A. Observer bias [6%]


B. Measurement bias [7%]
C. Rare disease assumption [3%]

D. Confounding [8%]
* E. Lead-time bias [74%]
Explanation: User Id: 549201
The prospect of lead-time bias should always be considered when evaluating any screening test. Lead-time
bias is defined as an artificial increase in survival time among tested patients who actually have an unchanged
prognosis. Patients screened with more sensitive tests appear to live longer only because the disease was
detected earlier than it would have been if diagnosed clinically. The overall length of time from disease onset
to death actually remains the same in both groups.
(Choices A and B) Measurement bias and observer bias refer to the misclassification of outcome and/or
exposure (eg, labeling diseased as non-diseased and vice versa) and are related to poor study design. The
scenario described does not mention how the study was designed.
(Choice D) Although the results of the study could be potentially confounded, there is no information on how
potential confounders were treated during the design or analysis stage of the study.
(Choice C) Rare disease assumption refers to the practice of approximating the odds ratio and relative risk
when studying rare diseases.
Educational Objective:
The typical example of lead-time bias is an apparent increase in survival in patients diagnosed with a new test
who actually have an unchanged prognosis. Think of lead-time bias when vou see "a new screening test" for

Block Tim e Rem aining: 05 :15


T u to r

Wfk

Feedback Suspend End Block

Item: 15 of 46

il

1 f M a rk < 1 O
m
Q. Id : 1170 [549201] P re v io u s N e x t Lab V a lu e s N o te s C a lc u la to r

tms new test demonstrates tnat its use prolongs survival or lung cancer patients Dy tnree montns wnen
compared to the survival of those subjects diagnosed by conventional methods. The researchers conclude
that the use of this new biomarker improves prognosis in patients with non-small cell lung cancer Which of
the following is a potential problem with this conclusion?

O
O
O
O

A. Observer bias [6%]


B. Measurement bias [7%]
C. Rare disease assumption [3%]

D. Confounding [8%]
* E. Lead-time bias [74%]
Explanation: User Id: 549201
The prospect of lead-time bias should always be considered when evaluating any screening test. Lead-time

bias is defined as an artificial increase in survival time among tested patients who actually have an unchanged
prognosis. Patients screened with more sensitive tests appear to live longer only because the disease was
detected earlier than it would have been if diagnosed clinically. The overall length of time from disease onset
to death actually remains the same in both groups.
(Choices A and B) Measurement bias and observer bias refer to the misclassification of outcome and/or
exposure (eg, labeling diseased as non-diseased and vice versa) and are related to poor study design. The
scenario described does not mention how the study was designed.
(Choice D) Although the results of the study could be potentially confounded, there is no information on how

potential confounders wer


reated during the design or analysis stage of the study.
(Choice C) Rare disease assumption refers to the practice of approximating the odds ratio
and relative risk
when studying rare diseases.
Educational Objective:
The typical example of lead-time bias is an apparent increase in survival in patients diagnosed with a new test
who actually have an unchanged prognosis. Think of lead-time bias when you see "a new screening test" for
poor prognosis diseases like lung or pancreatic cancer!
Time Spent: 8 seconds Copyright USMLEWorld;LLC. Last updated: [8/22/2014]

Block Tim e Rem aining: 05 :18


T u to r

qp o

Feedback Suspend End Block

Item: 16 o f 46

11 f M a rk

Q. Id : 1172 [549201]

<a
P re v io u s

E>
Next

II
Lab V a lu e s

Bp*-
N o te s C a lc u la to r

A large study of serum folate levels in women aged 16-45 years reveals that this parameter is normally
distributed, with a mean of 5.0 ng/ml and a standard deviation of 0.5 ng/ml. According to the study results,
95% of serum folate observations in these patients will lie between the following limits:

o A. 4.0 and 6.0 ng/ml


o B. 4.0 and 5.5 ng/ml
o c. 4.5 and 5.5 ng/ml
o D. 3.5 and 6.0 ng/ml
o E. 3.5 and 6.5 ng/ml
Block Tim e Rem aining: 05 : 21
T u to r

Feedback Suspend End Block

Item: 16 o f 46
Q. Id : 1172 [549201]

<a
P re v io u s

E>
Next

11 f Mark

II
Lab V a lu e s

Bp?-N o te s C a lc u la to r

A large study of serum folate levels in women aged 16-45 years reveals that this parameter is normally
distributed, with a mean of 5.0 ng/ml and a standard deviation of 0.5 ng/ml. According to the study results,
95% of serum folate observations in these patients will lie between the following limits:
* () A. 4.0 and 6.0 ng/ml
o'*
CNJ

o
o
o
o

B. 4.0 and 5.5 ng/ml

[2%]

C. 4.5 and 5.5 ng/ml [18%]


D. 3.5 and 6.0 ng/ml

[2%]

E. 3.5 and 6.5 ng/ml [5%]


Explanation: User Id: 549201
Normal distribution is defined as a symmetrical, bell-shaped distribution curve. What makes this particular
distribution curve so useful is the ease of predicting what proportion of observations will lie within certain
distances from the mean. Specifically, the standard deviation represents the extent of expected variance from
the mean. 95% of all observations lie within two standard deviations of the mean.
(Choice C) 68% of all observations lie within one standard deviation of the mean.
(Choices B and D) These limits are asymmetric and are therefore inconsistent with a normal distribution
curve around the mean.
(Choice E) 99.7% of all observations lie within three standard deviations of the mean.
Educational Objective:
In a normal (bell-shaped) distribution curve, 68% of observations lie within one standard deviation of the mean,
95% of observations lie within two standard deviations of the mean, and 99.7% of observations lie within three
standard deviations of the mean.
Time Spent: 13 seconds Copyright USMLEWorld,LLC. Last updated: [8/22/2014]

Block Tim e Rem aining:


T u to r

05

29

Feedback Suspend End Block

Item: 17 o f 46

11 f M a rk

Q. Id : 8422 [549201]

<a
P re v io u s

E>
Next

II
Lab V a lu e s

Bp*-
N o te s C a lc u la to r

A study is performed comparing the effect of tramadol to placebo for painful polyneuropathy. Fifty patients are
randomized to 1 of 2 sequences: tramadol followed by placebo or placebo followed by tramadol. Each
treatment is delivered for 4 weeks with an interim 1-week washout period. After each treatment period,
patients use a 10-point numeric scale to rate pain, paresthesia, and tenderness. Which of the following best
describes this study design?

O
O
O
O
O
O

A. Case-control study
B. Case series study
C. Crossover study
D. Cross-sectional study
E. Prospective cohort study
F. Retrospective cohort study

Block Tim e Rem aining:


T u to r

05

31

Feedback Suspend End Block

Item: 17 o f 46

11 f M a rk

Q. Id : 8422 [549201]

<a
P re v io u s

E>
Next

II
Lab V a lu e s

Bp*-
N o te s C a lc u la to r

A study is performed comparing the effect of tramadol to placebo for painful polyneuropathy. Fifty patients are
randomized to 1 of 2 sequences: tramadol followed by placebo or placebo followed by tramadol. Each
treatment is delivered for 4 weeks with an interim 1-week washout period. After each treatment period,
patients use a 10-point numeric scale to rate pain, paresthesia, and tenderness. Which of the following best
describes this study design?

O
O

A. Case-control study [14%]

O
O
O

D. Cross-sectional study [8%]

B. Case series study [7%]


* C. Crossover study [51 %]
E. Prospective cohort study [19%]

F. Retrospective cohort study [2%]


Explanation:

Crossover study design


User Id: 549201
Wash out period
Placebo I Placebo
(ftlKMI FWnrlri 11C

Block Tim e Rem aining:


T u to r

05 : 39

Feedback Suspend End Block

Item: 17 o f 46

11 F M a rk

Q. Id : 8422 [549201]

<a
P re v io u s

UlUUpD
Active treatm ent
<D USMLEWodcL LLC
Next

Active treatm ent


Lab V a lu e s N o te s C a lc u la to r
A

In a crossover study, subjects are randomly allocated to a sequence of 2 or more treatments given
consecutively. The simplest model is the AB/BA type of study in which subjects allocated to the AB study arm
receive treatment A followed by treatment B, and vice versa in the BA arm. Crossover trials allow the patients
to serve as their own controls. The principal drawback of crossover trials is that the effects of one treatment
may carry over" and alter the response to subsequent treatments. To limit this disadvantage, a washout (no
treatment) period is often added between consecutive treatments. The washout period is designed to be long
enough to allow the effects of prior treatment to wear off.
(Choice A) A case-control study is designed by selecting patients with a particular disease (cases) and
without that disease (controls) and then determining their previous exposure status.
(Choice B) A case series is a descriptive study that tracks patients with a known condition (eg, a particular
exposure, risk factor, or disease) to document natural history or response to treatment. Unlike a case-control
study, a case series is a qualifying study that cannot quantify statistical significance.
(Choice D) A cross-sectional study is also known as a prevalence study. It is characterized by the
simultaneous measurement of exposure and outcome. It is a snapshot study design that frequently uses

surveys. These studies are relatively inexpensive and easy to perform.


(Choice E & F) Prospective cohort studies are organized by selecting a group of individuals (ie, cohort),
determining their exposure status, and then following them over time for development of the disease of
interest. Sometimes the exposure status is determined retrospectively and patients are tracked from the point
of exposure onward, typically using medical records.
Educational objective:
In a crossover study, subjects are randomly allocated to a sequence of 2 or more treatments given
consecutively. A washout (no treatment) period is often added between treatment intervals to limit the
confounding effects of prior treatment.
Time Spent: 9 seconds Copyright USMLEWorld,LLC. Last updated: [8/11/2014] V

Block Tim e Rem aining:

05 : 43

Tutor
Feedback Suspend End Block

Item: 18 o f 46

11 f M a rk

Q. Id : 1208 [549201]

<a
P re v io u s

E>
Next

II
Lab V a lu e s

Bp*-
N o te s C a lc u la to r

A suburban hospital's yearly surgical infection statistics are shown below.


Surgical infection Fatal % of fatal Non-fatal % of non-fatal
Cl. difficile 2 3 14 6
S. aureus, methicillin-sensitive 10 13 25 11
S. aureus, methicillin-resistant 40 53 70 32
E. coli 10 13 50 23
Cl. perfringens 1 1 1 < 1
S. epidermidis 2 3 30 14
Others 10 13 30 14
Overall 75 100 220 100
What is the case-fatality rate for methicillin-resistant S. aureus surgical infections in this hospital?

O
O
O
O
O

A. 40/70
B. 40/110
C. 40/75
D. 53/100
E. 70/220

Block Tim e Rem aining:


T u to r

Feedback Suspend End Block

Item: 18 o f 46
Q. Id : 1208 [549201]

<a
P re v io u s

E>
Next

II
Lab V a lu e s

Bp?-N o te s C a lc u la to r

11 ^ M a r k

05

46

A suburban hospital's yearly surgical infection statistics are shown below.


Surgical infection Fatal % of fatal Non-fatal % of non-fatal
Cl. difficile 2 3 14 6
S. aureus, methicillin-sensitive 10 13 25 11
S. aureus, methicillin-resistant 40 53 70 32
E. coli 10 13 50 23
Cl. perfringens 1 1 1 < 1
S. epidermidis 2 3 30 14
Others 10 13 30 14
Overall 75 100 220 100
What is the case-fatality rate for methicillin-resistant S. aureus surgical infections in this hospital?

A. 40/70 [7%]

() B. 40/1

o
o
o

10[71 %]

C. 40/75 [14%]
D. 53/100 [7%]

E. 70/220 [1%]
Explanation: User Id: 549201
You may encounter USMLE scenarios where you are asked to interpret epidemiological graphs or tables and
perform some basic calculations. Case-fatality rate is calculated by dividing the number of fatal cases by the
total number of people with the disease. In this scenario, 40 fatal and 70 non-fatal cases of MRSA infection
are reported. Therefore, case-fatality rate is 40/(40+70) = 40/110.
(Choices C and D) These choices describe the proportion of MRSA deaths out of all surgical infection deaths
(40/75).
Educational Objective:
Case-fatality rate is calculated by dividing the number of fatal cases by the total number of people with the
disease.

Block Tim e Rem aining: 05 : 51


T u to r

Feedback Suspend End Block

Item: 19 o f 46

11 f M a rk

Q. Id : 1270 [549201]

<a
P re v io u s

E>
Next

II
Lab V a lu e s

Bp*-
N o te s C a lc u la to r

A state with a population of 4,000,000 contains 20,000 people who have disease A, a fatal neurodegenerative
condition. There are 7000 new cases of the disease a year and 1000 deaths attributable to disease A. There
are 40,000 deaths per year from all causes. What is the incidence of the disease?

o
o
o
o
o

A. 1 0 0 0 /4 . 000,000
B. 6000 / 4 000,000
c . 7 0 0 0 /3 980,000
D. 20,000 / 4,000,000
E. 40,000 / 4,000,000

Block Tim e Rem aining:


T u to r

Feedback Suspend End Block

05

56

Item: 19 o f 46

11 f M a rk

Q. Id : 1270 [549201]

<a
P re v io u s

E>
Next

II
Lab V a lu e s

Bp?-N o te s C a lc u la to r

A state with a population of 4,000,000 contains 20,000 people who have disease A, a fatal neurodegenerative
condition. There are 7000 new cases of the disease a year and 1000 deaths attributable to disease A. There
are 40,000 deaths per year from all causes. What is the incidence of the disease?

o
o

A. 1000/4,000,000 [2%]
B. 6000 / 4,000,000 [6%]

c.

* ()

o
o

7000/3,980,000 [84%]

D. 20,000/4,000,000(6%]

E. 40,000 / 4,000,000 [1 %]
Explanation: User Id: 549201
The incidence of a disease is the number of new cases of a disease per year divided by the total population at
risk. In this case it is 7000 / (4,000,000 - 20,000)
(Choice A) The disease-specific mortality is the number deaths attributable to a disease per year divided by
the total population. It gives 1000/ 4,000,000.
(Choice B) The rate of increase of a disease is the number of new cases per year minus the number of
death (or cures) per year, divided by the total population. This yields (7000-1000) / 4,000,000.
(Choice D) The prevalence of a disease is the number of persons with a disease divided by the total
population at a specific point in time. In this case it is 20,000 / 4,000,000
(Choice E) The mortality rate is the number of deaths per year divided by the total population. It yields 40,000
/ 4,000,000.
Educational Objective:
The incidence of a disease is the number of new cases of a disease per year divided by the total population at
risk.
Time Spent: 9 seconds Copyright USMLEWorld,LLC. Last updated: [8/22/2014]

Block Tim e Rem aining: 06 : 02


T u to r

Feedback Suspend End Block

Item: 20 o f 46

11 f M a rk

Q. Id : 1171 [549201]

<a
P re v io u s

E>
Next

II
Lab V a lu e s

Bp*-
N o te s C a lc u la to r

An experimental drug that selectively binds malignant lymphocytes has been shown to significantly prolong
survival in patients with stage 3 and stage 4 chronic lymphocytic leukemia (CLL). If this new drug were widely
used, what changes in the incidence and prevalence of CLL would be expected?
O A. Incidence will decrease, prevalence will decrease
O B. Incidence will not change, prevalence will increase
O C. Incidence will increase, prevalence will not change

O D. Incidence will not change, prevalence will not change


O E. Incidence will decrease, prevalence will increase

Block Tim e Rem aining: 06 :


T u to r
AV

05

Feedback Suspend End Block

Item: 20 of 46 1 fMark < 1 O i l iR l


Q. Id : 1171 [549201] P re v io u s N e x t Lab V a lu e s N o te s C a lc u la to r
A

An experimental drug that selectively binds malignant lymphocytes has been shown to significantly prolong
survival in patients with stage 3 and stage 4 chronic lymphocytic leukemia (CLL). If this new drug were widely
used, what changes in the incidence and prevalence of CLL would be expected?

A. Incidence will decrease, prevalence will decrease [3%]


* () B. Incidence will not change, prevalence will increase [86%]

O
O
O

C. Incidence will increase, prevalence will not change [2%]


D. Incidence will not change, prevalence will not change [6%]

E. Incidence will decrease, prevalence will increase [3%]


Explanation: User Id: 549201
Prevalence and incidence are two basic measures of disease occurrence, and it is important to understand
the difference between them. Incidence is the measure of new cases diagnosed in a given period of time.
Prevalence is the measure of the total number of cases at a particular point in time. Incidence answers the
question: how many new cases of the disease were diagnosed in a population during a particular period of
time? Prevalence answers the question: how many cases of the disease, both new and old, exist in a
population at a particular point in time? The relationship between prevalence and incidence can be expressed
as the following equation:
Prevalence = (Incidence) x (Time)
Incidence of a disease is not changed by any kind of treatment, because the disease has already developed
when the treatment is started. Any treatment that prolongs survival but does not cure the disease will
increase prevalence due to an increase in the number of afflicted (but still alive) individuals over time.
(Choices A, C, and E) Incidence of a disease is not affected by treatment.
(Choice D) Prevalence will increase if the length of survival time increases.
Educational Objective:
Incidence is the measure of new cases diagnosed in a given period of time. Prevalence is the measure of the

Block Tim e Rem aining: 06 : 1 0


T u to r

Wfk

Feedback Suspend End Block

Item: 20 o f 46

11 f Mark

Q. Id : 1171 [549201] Lab V a lu e s

used, what changes in the incidence and prevalence of CLL would be expected?

A. Incidence will decrease, prevalence will decrease [3%]


() B. Incidence will not change, prevalence will increase [86%]

O
O
O

C. Incidence will increase, prevalence will not change [2%]


D. Incidence will not change, prevalence will not change [6%]

E. Incidence will decrease, prevalence will increase [3%]


Explanation: User Id: 549201
Prevalence and incidence are two basic measures of disease occurrence, and it is important to understand
the difference between them. Incidence is the measure of new cases diagnosed in a given period of time.
Prevalence is the measure of the total number of cases at a particular point in time. Incidence answers the
question: how many new cases of the disease were diagnosed in a population during a particular period of
time? Prevalence answers the question: how many cases of the disease, both new and old, exist in a
population at a particular point in time? The relationship between prevalence and incidence can be expressed
as the following equation:
Prevalence = (Incidence) x (Time)
Incidence of a disease is not changed by any kind of treatment, because the disease has already developed
when the treatment is started. Any treatment that prolongs survival but does not cure the disease will
increase prevalence due to an increase in the number of afflicted (but still alive) individuals over time.
(Choices A, C, and E) Incidence of a disease is not affected by treatment.
(Choice D) Prevalence will increase if the length of survival time increases.
Educational Objective:
Incidence is the measure of new cases diagnosed in a given period of time. Prevalence is the measure of the

total cases at a particular point in time. Any treatment that prolongs survival but does not cure the disease will
increase prevalence due to an increase in the number of afflicted (but still alive) individuals over time.
Time Spent: 9 seconds Copyright USMLEWorld,LLC. Last updated: [8/22/2014]

Block Tim e Rem aining: 06 :


T u to r

13

Feedback Suspend End Block

Item: 21 o f 46

11 f M a rk

Q. Id : 8456 [549201]

<a
P re v io u s

E>
Next

II
Lab V a lu e s

Bp*-
N o te s C a lc u la to r

A primary care physician who has just completed residency is reviewing employment offers from a number of
local clinics and hospitals. He learns that there are different payment methods that health insurance plans
use to pay physicians. Which of the following payment methods is most strongly associated with physicians
providing more preventive care and health counseling services?
O A. Capitation
O B. Discounted fee-for-service
O C. Fee-for-service
O D. Salary

Block Tim e Rem aining: 06 : 16


T u to r

Feedback Suspend End Block

Item: 21 o f 46

11 F M a rk

Q. Id : 8456 [549201]

<a
P re v io u s

E>
Next

A primary care physician who has just completed residency is reviewing employment offers from a number of
local clinics and hospitals. He learns that there are different payment methods that health insurance plans
use to pay physicians. Which of the following payment methods is most strongly associated with physicians
providing more preventive care and health counseling services?
Lab V a lu e s N o te s C a lc u la to r
A

* () A. Capitation [43%]

O
O
O

B. Discounted fee-for-service [9%]


C. Fee-for-service [23%]

D. Salary [25%]
Explanation: User Id: 549201
There are essentially 4 basic payment methods that serve as the foundation for most contracts between
health insurance plans and physicians:
Capitation: Physicians are paid a fixed amount per enrollee, not per service. Physicians paid by
capitation have incentives to contain costs due to the fixed budget allocated to them. If many
enrollees seek care or there are enrollees needing extensive care, physicians' costs can become
greater than their payments. Thus, physicians are motivated to provide more preventive care (eg,
Pap smears and mammograms) to catch illnesses early so that patients stay healthier and need
fewer tests and procedures as they age.
Fee-for-service (FFS): Physicians are paid a fixed amount for every service and diagnostic test
that they provide. They face little financial risk and are enticed to increase the number of services
they provide on each visit, as well as the number of visits per patient. There is no incentive to avoid
costly tests and procedures (Choice C).
Discounted fee-for-service: Discounted FFS works similarly to FFS except that physicians are
reimbursed a discounted amount. Thus, physicians paid under this model may be more

conservative when ordering tests and providing services compared to those paid by FFS,
especially if expensive tests or services are greatly discounted (Choice B)
Salary: Physicians who are salaried are paid a fixed amount, and their pay is not tied to the

Block Tim e Rem aining: 06 : 21


T u to r

Feedback Suspend End Block

Item: 21 of 46 1 F M a rk O i l iR l
Q. Id : 8456 [549201] P re v io u s
in nun mi ^im min mu in ii 111in11

ii ii

N e x t Lab V a lu e s N o te s C a lc u la to r

greater m an meir p aym ents. i n u s, p n y siu a ris are rriuuvaieu

iu

pruviue rriure preverm ve ca re ^ey,

Pap smears and mammograms) to catch illnesses early so that patients stay healthier and need
fewer tests and procedures as they age.
Fee-for-service (FFS): Physicians are paid a fixed amount for every service and diagnostic test
that they provide. They face little financial risk and are enticed to increase the number of services
they provide on each visit, as well as the number of visits per patient. There is no incentive to avoid
costly tests and procedures (Choice C).
Discounted fee-for-service. Discounted FFS works similarly to FFS except that physicians are
reimbursed a discounted amount. Thus, physicians paid under this model may be more
conservative when ordering tests and providing services compared to those paid by FFS,
especially if expensive tests or services are greatly discounted (Choice B)
Salary: Physicians who are salaried are paid a fixed amount, and their pay is not tied to the
number of enrollees or services rendered. Unless their contracts include withholds or bonuses,
salaried physicians face no financial risk. Thus, they have no financial incentive to change their
treatment patterns, either in services provided or number of follow-up visits (Choice D).
Capitation is often used in health maintenance organization insurance plans. FFS and discounted FFS are
commonly used in preferred provider organization insurance plans.
Educational objective:
There are essentially 4 basic methods that health insurance plans use to reimburse physicians: capitation,
fee-for-service (FFS), discounted FFS, and salary. Physicians paid under FFS face little financial risk and
have incentives to increase services, tests, and patient visits. Physicians paid under capitation face the
greatest financial risk and have incentives to provide more preventive care and better health counseling.
References:
1. Capitation, salary, fee-for-service and mixed systems of payment: effects on the
behaviour of primary care physicians.
2. Capitation payment, length of visit, and preventive services: evidence from a national
sample of outpatient physicians.
Time Spent: 9 seconds Copyright USMLEWorld,LLC. Last updated: [3/8/2014] V

Block Tim e Rem aining: 06 : 25


T u to r

Feedback Suspend End Block

Item: 22 o f 46

11 f M a rk

Q. Id : 1013 [549201]

<a
P re v io u s

E>
Next

II
Lab V a lu e s

Bp*-
N o te s C a lc u la to r

A 50-year-old man comes to the physician for a routine check-up. He has a 20-pack-year smoking history
and consumes alcohol occasionally. He eats a balanced diet and consumes fried foods and red meat several
times a week. He is not very active and spends much of his free time watching television and sitting at his
computer. His family history is significant for hypertension in his mother and bladder cancer in his father. His
blood pressure is 160/90 mm Hg and heart rate is 70/min. Laboratory testing is significant for a fasting blood
glucose level of 155 mg/dL. Which of the following interventions is likely to have the greatest effect on

reducing this patient's mortality risk?

O
O
O
O
O
O
O

A. Annual fecal occult blood test


B. Annual prostate-specific antigen test
C. Daily aspirin
D. Dietary modification
E. Regular exercise
F. Smoking cessation
G. Tight glycemic control

Block Tim e Rem aining: 06 :


T u to r

29

Feedback Suspend End Block

Item: 22 o f 46

11 f M a rk

Q. Id : 1013 [549201]

<a
P re v io u s

E>
Next

II
Lab V a lu e s

Bp*-
N o te s C a lc u la to r

A 50-year-old man comes to the physician for a routine check-up. He has a 20-pack-year smoking history
and consumes alcohol occasionally. He eats a balanced diet and consumes fried foods and red meat several
times a week. He is not very active and spends much of his free time watching television and sitting at his
computer. His family history is significant for hypertension in his mother and bladder cancer in his father. His
blood pressure is 160/90 mm Hg and heart rate is 70/min. Laboratory testing is significant for a fasting blood
glucose level of 155 mg/dL. Which of the following interventions is likely to have the greatest effect on
reducing this patient's mortality risk?

O
O
O
O
O

A. Annual fecal occult blood test [0%]


B. Annual prostate-specific antigen test [0%]
C. Daily aspirin [Q%]
D. Dietary modification [0%]

E. Regular exercise [0%]


* F. Smoking cessation [93%]

G. Tight glycemic control [7%]


Explanation: User Id: 549201
15

Mortality reduction
Aspirin therapy
Lipid reduction
I BP control
Smoking cessation
Residual risk
Block Tim e Rem aining: 06 :
T u to r

Feedback Suspend End Block

Item: 22 o f 46
Q. Id : 1013 [549201]

<a
P re v io u s

E>
Next

11 f M a rk

39

Mortality reduction
Aspirin therapy
Lipid reduction
I BP control
Smoking cessation
Residual risk
Lab V a lu e s N o te s C a lc u la to r
A

USMLEWorld, LLC

One of the objectives for the national Healthy People 2020 initiative is to reduce the prevalence of tobacco
smoking in adults from 20% in 2008 to less than 12% by 2020. Tobacco use, especially smoking, is the
single most preventable cause of death and disease in the United States. In addition to its numerous direct
adverse effects, tobacco smoking also substantially increases the risk for macrovascular (eg, myocardial
infarction [Ml], stroke) and microvascular (eg, retinopathy, nephropathy) complications of diabetes mellitus.
These complications significantly increase mortality, primarily by increasing the risk of Ml. The risk of
Ml-associated mortality begins to decrease immediately upon smoking cessation. However, it takes several
tobacco-free years before the risk returns to baseline, especially in patients with a long history of smoking. In
addition, smokers with diabetes have a higher likelihood of developing hypercholesterolemia, poor glycemic
control, and end-stage renal disease. It appears that smoking not only worsens the complications of diabetes
but also increases the likelihood of developing diabetes in the first place, thus highlighting the importance of
prevention and early cessation.
(Choice A) Fecal occult blood testing screens for colorectal cancer and improves mortality though earlier
detection. Even so, the mortality benefit is much lower than that achieved by smoking cessation.
V

Block Tim e Rem aining: 06 : 44 P


T u to r

Feedback Suspend End Block

Item: 22 of 46 1

I^ M a rk < 1 O i l iR l
Q. Id : 1013 [549201] P re v io u s N e x t Lab V a lu e s N o te s C a lc u la to r

infarction [Ml], stroke) and microvascular (eg, retinopathy, nephropathy) complications of diabetes mellitus. a
These complications significantly increase mortality, primarily by increasing the risk of Ml. The risk of
Ml-associated mortality begins to decrease immediately upon smoking cessation. However, it takes several
tobacco-free years before the risk returns to baseline, especially in patients with a long history of smoking. In
addition, smokers with diabetes have a higher likelihood of developing hypercholesterolemia, poor glycemic
control, and end-stage renal disease. It appears that smoking not only worsens the complications of diabetes
but also increases the likelihood of developing diabetes in the first place, thus highlighting the importance of
prevention and early cessation.
(Choice A) Fecal occult blood testing screens for colorectal cancer and improves mortality though earlier
detection. Even so, the mortality benefit is much lower than that achieved by smoking cessation.
(Choice B) Prostate-specific antigen is increased in some patients with prostate cancer. However, annual
prostate-specific antigen testing has not been shown to improve mortality from prostate cancer.
(Choice C) Aspirin use significantly reduces the risk of death from coronary heart disease in diabetic
patients, at levels similar to smoking cessation. However, when all-cause mortality is compared, smoking
cessation has a greater effect on reducing mortality than daily aspirin therapy.
(Choices D, E, and G) Dietary modification and exercise reduce the risk of cardiovascular disease (to a
lesser extent than smoking cessation) and help improve glycemic control in diabetic individuals. Although tight
glycemic control has been shown to reduce the risk of microvascular complications from diabetes, it does not
significantly improve cardiovascular or all-cause mortality.
Educational objective:
Smoking cessation is by far the most effective preventive intervention in almost all patients, and this is
especially true in those with diabetes.
References:
1. How can we best prolong life? Benefits of coronary risk factor reduction in non-diabetic
and diabetic subjects.
2. Effect of tig h t blood glucose control versus conventional control in patients with type 2
diabetes mellitus: a systematic review with meta-analysis of randomized controlled trials.
Time Spent: 10 seconds Copyright USMLEWorld,LLC. Last updated: [3/27/2014] V

Block Tim e Rem aining: 06 : 4 7


T u to r

Feedback Suspend End Block

Item: 23 o f 46

11 f M a rk

Q. Id : 1230 [549201]

<a
P re v io u s

E>
Next

II
Lab V a lu e s

Bp*-
N o te s C a lc u la to r

A new test to diagnose urinary tract infections (UTIs) is being evaluated. The sensitivity of the test is 70% and
the specificity is 90%. In the study there are 100 patients who actually have UTIs and 200 who actually do not,
as determined by the diagnostic gold standard. How many false positives are in this study?

Block Tim e Rem aining: 06 : 51


T u to r

Feedback Suspend End Block

Item: 23 o f 46

11 f M a rk

Q. Id : 1230 [549201]

<a
P re v io u s

E>
Next

II
Lab V a lu e s

Bp*-
N o te s C a lc u la to r

A new test to diagnose urinary tract infections (UTIs) is being evaluated. The sensitivity of the test is 70% and
the specificity is 90%. In the study there are 100 patients who actually have UTIs and 200 who actually do not,
as determined by the diagnostic gold standard. How many false positives are in this study?
* () A. 20 [66%] o B. 30 [20%] o c. 70 [7%] o D. 120 [3%] o E. 180 [5%]
Explanation: User Id: 549201
False positives describe people who test positive for a disease but do not actually have it. In the table below
they are represented by square b.
Disease positive Disease negative

a
True positive
b
False positive a+b
Test negative c
False negative
d
True negative c+d
a+cb+d
Test positive

S ensitivity = a/(a+c)
Specificity = d/(b+d)

Block Tim e Rem aining: 07 : oi


T u to r

Feedback Suspend End Block

I F Mark

<1

P re v io u s

The number of false positives can be calculated using the test's specificity and total number of patients
without UTIs. The specificity of a test refers to the number of true negatives divided by the total number of
patients without the disease (d / [b + d]). In this case, the specificity is 90% and the total number of patients

without UTIs (b + d) equals 200. Using this information, the number of true negatives (d) can be calculated:
Specificity = d / (b + d)
0.9 = d / 200
d = 180
Lab V a lu e s

Since the total number of patients without the disease (b + d) equals 200 and the number of true negatives (d)
equals 180, the number of false positives (b) is 20.
For the sake of thoroughness, the two-by-two table can be completed as follows. Sensitivity refers to the
number of true positives divided by all people with the disease (a / [a + c]). Repeating the same exercise as
above gives the completed table:
Sensitivity = a / (a + c)
0.7 = a/100
a = 70
c = 100 - 7 0 = 30
+ UTI - UTI
Test positive 70 20 90
Test negative 30 180 210
100 200
The number of true negatives and false positives can be more quickly calculated using the following equations:
True negatives = (Specificity) * (Number of patients actually without the disease)
False positives = (1 - Specificity) * (Number of patients actually without the disease)
Educational objective:
Specificity is the number of true negatives divided by the total number of subjects actually without the disease.
True negatives = (Specificity) * (Number of patients actually without the disease)
False positives = (1 - Specificity) * (Number of patients actually without the disease)

Block Tim e Rem aining:


r 's t m r m s w M ' I I O M I C t A f n r U I I O

MMO/OfH /II

07:06
Tutor

Item: 24 o f 46

11 f M a rk

Q. Id : 1185 [549201]

<a
P re v io u s

E>
Next

II
Lab V a lu e s

Bp*-
N o te s C a lc u la to r

Middle-aged men who have a first-degree relative with Alzheimer's disease have their blood carotene
concentration measured. They are followed over twenty years for the development of Alzheimer's disease.
Results are given below.
Carotene concentration Low Normal
Diseased 20 40 60
Non-diseased 20 120 140
40 160 200
What is the 20-year risk of developing Alzheimer's disease in subjects with low carotene level?
O A. 0.5
O B. 0.3
O C. 0.25
O D. 0.2
o E. 0.14

Block Tim e Rem aining: 07 :10


T u to r

Feedback Suspend End Block

Item: 24 o f 46
Q. Id : 1185 [549201]

<a

11 f M a rk

P re v io u s

E>
Next

II
Lab V a lu e s

Bp*-
N o te s C a lc u la to r

Middle-aged men who have a first-degree relative with Alzheimer's disease have their blood carotene
concentration measured. They are followed over twenty years for the development of Alzheimer's disease.
Results are given below.
Carotene concentration Low Normal
Diseased 20 40 60
Non-diseased 20 120 140
40 160 200
What is the 20-year risk of developing Alzheimer's disease in subjects with low carotene level?
* () A. 0.5 [62%] o B. 0.3 [16%] o c. 0.25 [10%] o D. 0.2 [9%] o E. 0.14 [2%]
Explanation: User Id: 549201
Risk is a measure of incidence of a disease. It shows the probability of getting a disease over a certain period
of time (20 years in this scenario). To calculate the risk, divide the number of diseased subjects by the overall
number of subjects in the corresponding group. In this example, to get the 20-year risk of Alzheimer's disease
in subjects with low carotene levels (exposed), divide the number of exposed diseased subjects (20) by the
total number of exposed people (40).
20/40 = 0.5
The calculated risk value is 0.5, which is interpreted as a 50% probability of developing Alzheimer's syndrome
in 20 years.
(Choice B) The prevalence of exposure in diseased people is 20/60 (0.3, 30%).

Block Tim e Rem aining: 07 :16


T u to r

Feedback Suspend End Block

Item: 24 o f 46

11 f M a rk

Q. Id : 1185 [549201]

<a
P re v io u s

E>
Next

II
Lab V a lu e s

Bp?-Notes C a lc u la to r

* () A. 0.5 [62%]
O'
o
un
C\J
o o D. 0.2 [9%]
6s

B. 0.3 [16%]

o c.

nP

E.

CM.

d
Explanation: User Id: 549201
Risk is a measure of incidence of a disease. It shows the probability of getting a disease over a certain period
of time (20 years in this scenario). To calculate the risk, divide the number of diseased subjects by the overall
number of subjects in the corresponding group. In this example, to get the 20-year risk of Alzheimer's disease
in subjects with low carotene levels (exposed), divide the number of exposed diseased subjects (20) by the
total number of exposed people (40).
20/40 = 0.5
The calculated risk value is 0.5, which is interpreted as a 50% probability of developing Alzheimer's syndrome

in 20 years.
(Choice B) The prevalence of exposure in diseased people is 20/60 (0.3, 30%).
(Choice C) The 20-year risk of getting Alzheimer's disease in people with normal carotene levels is 40/160
(0.25, 25%).
(Choice D) The prevalence of exposure in the entire cohort (exposed and unexposed combined) is 40/200

(0.2, 20%).
(Choice E) The prevalence of exposure in non-diseased people is 20/140 (0.1 4, 14%).
Educational Objective:
Risk is the probability of getting a disease over a certain period of time. To calculate the risk, divide the
number of diseased subjects by the total number of subjects in the corresponding group (i.e., all the people at
risk).
Time Spent: 12 seconds Copyright USMLEWorld,LLC. Last updated: [8/22/2014]

Block Tim e Rem aining:


T u to r

07 : 20

Feedback Suspend End Block

Item: 25 o f 46

11 f M a rk

Q. Id : 1277 [549201]

<a
P re v io u s

E>
Next

II
Lab V a lu e s

Bp*-
N o te s C a lc u la to r

Women who have just delivered babies with neural tube defects are asked about their use of acetaminophen
during the first three months of pregnancy. Women who delivered apparently healthy babies are also asked
about their use of acetaminophen during the first three months of pregnancy. Which of the following
measures of association are the investigators most likely to report?
O A. Relative risk
O B. Median survival
O C. Exposure odds ratio
O D. Relative rate
O E. Prevalence odds ratio

Block Tim e Rem aining: 07


T u to r

: 23

Feedback Suspend End Block

Item: 25 o f 46

11 f M ark

Q. Id : 1277 [549201]

<a
Previous

E>
Next

Women who have just delivered babies with neural tube defects are asked about their use of acetaminophen
during the first three months of pregnancy. Women who delivered apparently healthy babies are also asked
about their use of acetaminophen during the first three months of pregnancy. Which of the following
measures of association are the investigators most likely to report?
Lab Values Notes Calculator
A

O
O

A. Relative risk [34%]

O
O

D. Relative rate [3%]

B. Median survival [1%]


* C. Exposure odds ratio [56%]
E. Prevalence odds ratio [5%]
Explanation: User Id: 549201
This scenario describes a typical case-control study design. People with the disease of interest (cases) and
people without this disease (controls) are asked about previous exposure to the variable being studied

(acetaminophen use). The main measure of association is the exposure odds ratio, which is expressed as
the following equation:
(Odds of exposure of people with the disease [cases])
(Odds of exposure of people without the disease [controls])
(Choices A and D) Incidence measures (eg. relative risk or relative rate) cannot be directly measured in
case-control studies because the people being studied are those who already have the disease. Relative risk
and relative rate are calculated in cohort studies, in which people are followed over time for developing the
disease.
(Choice B) Median survival is calculated in cohort studies or clinical trials, and is usually used to compare the
median survival times in two or more groups of patients (eg, those receiving a new treatment and those
receiving a placebo).
(Choice E) Prevalence odds ratio is calculated in cross-sectional studies to compare the prevalence of a
disease in different populations,_____________________________________________________________________

Block Tim e Rem aining: 07 : 28


T u to r

Feedback Suspend End Block

Item: 25 o f 46

11 f M ark

Q. Id : 1277 [549201]

O
O

A. Relative risk [34%]

O
O

D. Relative rate [3%]

B. Median survival [1%]


* C. Exposure odds ratio [56%]
E. Prevalence odds ratio [5%]
<>Previous Next Lab Values Notes Calculator
A

Explanation: User Id: 549201


This scenario describes a typical case-control study design. People with the disease of interest (cases) and
people without this disease (controls) are asked about previous exposure to the variable being studied
(acetaminophen use). The main measure of association is the exposure odds ratio, which is expressed as
the following equation:
(Odds of exposure of people with the disease [cases])
(Odds of exposure of people without the disease [controls])
(Choices A and D) Incidence measures (eg, relative risk or relative rate) cannot be directly measured in
case-control studies because the people being studied are those who already have the disease. Relative risk
and relative rate are calculated in cohort studies, in which people are followed over time for developing the
disease.
(Choice B) Median survival is calculated in cohort studies or clinical trials, and is usually used to compare the
median survival times in two or more groups of patients (eg, those receiving a new treatment and those
receiving a placebo).
(Choice E) Prevalence odds ratio is calculated in cross-sectional studies to compare the prevalence of a
disease in different populations.
Educational Objective:
A case-control study is used to compare the exposure of people with the disease (cases) to the exposure of
people without the disease (controls). The main measure of association is the exposure odds ratio.
Time Spent: 9 seconds Copyright USMLEWorld,LLC. Last updated: [8/22/2014]

Block Tim e Rem aining: 07


T u to r

Feedback Suspend End Block

Item: 26 o f 46
Q. Id : 1176 [549201]

<a
P re v io u s

E>
Next

II
Lab V a lu e s

11 f M a rk

: 31

Bp*-
N o te s C a lc u la to r

You are designing a study to evaluate the efficacy of a new drug, KM28. Your study will compare KM28 plus
standard care versus standard care alone in decreasing the incidence of recurrent breast cancer The FDA
will approve the new drug if KM28 plus standard care decreases the rate of breast cancer recurrence by 40%
compared to standard therapy alone. The recurrence rate on standard therapy is found to be 8%. In order for
KM28 to be approved, what is the maximal incidence of recurrent disease acceptable for women treated with
KM28 plus standard therapy?

o
o
o
o
o

A. 5.2%
B. 4.8%
C. 3.6%
D. 3.2%
E. 2.8%

Block Tim e Rem aining: 07 : 34


T u to r

Feedback Suspend End Block

Item: 26 o f 46

11 f M ark

Q. Id : 1176 [549201]

<a
Previous

E>
Next

You are designing a study to evaluate the efficacy of a new drug, KM28. Your study will compare KM28 plus
standard care versus standard care alone in decreasing the incidence of recurrent breast cancer The FDA
will approve the new drug if KM28 plus standard care decreases the rate of breast cancer recurrence by 40%
compared to standard therapy alone. The recurrence rate on standard therapy is found to be 8%. In order for
KM28 to be approved, what is the maximal incidence of recurrent disease acceptable for women treated with
KM28 plus standard therapy?
Lab Values Notes Calculator
A

Explanation: User Id: 549201


This question asks you to recognize that the 40% mentioned in the prompt refers to relative risk reduction
(RRR). RRR is defined as the percent reduction in absolute risk between the treatment and control groups.
The formula for RRR is:
RRR = [Absolute Risk{oonBor. - Absolute Risk(rntlirn1]l / Absolute Risk;ccn;ro,:
We are provided values for Absolute Risk{control) and RRR in the question stem, so the next step is to solve for
Absolute Risk,,reaWftt;. The formula can thus be rearranged to:
Absolute R is k ,^ ^ = Absolute Risk(cortrol) - [RRR * Absolute Risk{ccrtroI;]
In this case the RRR is 0.4 and the Absolute Risk(oonaol; is 0.08, which gives us:
Absolute Risk,.re3?ner: = 0.08 - [0.4 * 0.08]
Absolute Risk(lrearnenr) = 0.048 or 4.8%
It is important to remember that relative risk reduction (RRR) may overstate the effectiveness of an V

Block Tim e Rem aining:


T u to r

07 :

39

Wfk

Feedback Suspend End Block

Item: 26 o f 46

11 f M ark

Q. Id : 1176 [549201]

KM28 plus standard therapy?


<>Previous Next Lab Values Notes Calculator

Explanation: User Id: 549201


This question asks you to recognize that the 40% mentioned in the prompt refers to relative risk reduction
(RRR). RRR is defined as the percent reduction in absolute risk between the treatment and control groups.
The formula for RRR is:
RRR = [Absolute Risk(ctntroI. - Absolute Risk,tre3Pners.] / Absolute Risk(cortrot)
We are provided values for Absolute Risk(conool} and RRR in the question stem, so the next step is to solve for
Absolute Risk,rearneT. The formula can thus be rearranged to:
Absolute Risk,.reat1ie^: = Absolute Riskfcon.ro(> - [RRR * Absolute Risk(eonwIJ
In this case the RRR is 0.4 and the Absolute Risk(cwtro0 is 0.08, which gives us:

Absolute Risk(Watliert) = 0.08 - [0.4 * 0.08]


Absolute Risk?re3?Tierr = 0.048 or 4.8%
It is important to remember that relative risk reduction (RRR) may overstate the effectiveness of an
intervention. For example, a RRR of 50% could occur if a drug decreases the incidence of a disease from 2%
to 1 % or from 50% to 25%. Clearly, the latter is of greater clinical significance.
Educational objective:
RRR = [Absolute Risk(C(mor, - Absolute Risk(trearnenrJ / Absolute Risk(con!roI)
Time Spent: 10 seconds Copyright USMLEWorld,LLC. Last updated: [3/30/2014]

Block Tim e Rem aining: 07 : 42


T u to r

Feedback Suspend End Block

Item: 27 o f 46

11 f M a rk

Q. Id : 1209 [549201]

<a
P re v io u s

E>
Next

II
Lab V a lu e s

Bp*-
N o te s C a lc u la to r

A sample of children aged 2-5 years is chosen at random from an outpatient clinic. The number of urinary
tract infection (UTIs) per year is given in the figure below.
-----------
Number of urinary
infection episodes
What is the average number of urinary infection episodes per year in a child in this age group served by the
clinic?
O A. Between 0 and 1
O B. 1
O C. Between 1 and 2
O D. 2
O E. Between 2 and 3

Block Tim e Rem aining: 07 : 48


T u to r

Feedback Suspend End Block

Item: 27 o f 46

11 f M a rk

Q. Id : 1209 [549201]

<a
P re v io u s

E>
Next

II
Lab V a lu e s

Bp*-
N o te s C a lc u la to r

A sample of children aged 2-5 years is chosen at random from an outpatient clinic. The number of urinary
tract infection (UTIs) per year is given in the figure below.
-----------
Number of urinary
infection episodes
What is the average number of urinary infection episodes per year in a child in this age group served by the
clinic?

() A. Between 0 and 1 [65%] o B. 1 [6%] o c. Between 1 and 2 [22%] o D. 2 [3%] o E. Between 2 and 3 [3%]
Explanation: User Id: 549201
To determine the average number of UTIs we sum all the UTIs and divide the result by the sample size. The
total number of UTIs is:
0x50 + 1x30 + 2x10 + 3x10 = 80.
The overall sample size is 100 (50 + 30 + 10 + 10). If we divide the number of UTIs by the sample size we get:
ohm no no

Block Tim e Rem aining: 07


T u to r

: 53

Feedback Suspend End Block

f'M a rk < 3 i l w inssa


Q. Id : 1209 [549201] Previous Next Lab Values Notes Calculator

Item: 27 of 46 SH
01
50

30

0 1 2 3 Number of urinary
infection episodes
What is the average number of urinary infection episodes per year in a child in this age group served by the
clinic?
() A. Between 0 and 1 [65%]

o
o
o
o

B. 1 [6%]
C. Between 1 and 2 [22%]
D. 2 [3%]

E. Between 2 and 3 [3%]


Explanation: User Id: 549201
To determine the average number of UTIs we sum all the UTIs and divide the result by the sample size. The
total number of UTIs is:
0x50 + 1x30 + 2x10 + 3x10 = 80.
The overall sample size is 100 (50 + 30 + 10 + 10). If we divide the number of UTIs by the sample size we get:
80/100 = 0.8
We conclude that a child between 2-5 years-old in this clinic experiences less than 1 UTI per year on average.
Educational Objective:
Understand the measures of center (mean/averages, median and mode). They are common topics on the
USMLE exams.
Time Spent: 13 seconds Copyright USMLEWorldXLC. Last updated: [8/22/2014]

Block Tim e Rem aining:


T u to r

07 : 56

Feedback

Suspend E nd 5o ck

Item: 27 o f 46

11 f M ark

Q. Id : 1209 [549201]

<3
Previous

-----------
Number of urinary
infection episodes
Next

What is the average number of urinary infection episodes per year in a child in this age group served by the
clinic?
Lab Values Notes Calculator

() A. Between 0 and 1 [65 %]

B. 1 [6%]

o
o
o

C. Between 1 and 2 [22%]


D. 2 [3%]

E. Between 2 and 3 [3%]


Explanation: User Id: 549201
To determine the average number of UTIs we sum all the UTIs and divide the result by the sample size. The
total number of UTIs is:
0x50 + 1x30 + 2x10 + 3x10 = 80.
The overall sample size is 100 (50 + 30 + 10 + 10). If we divide the number of UTIs by the sample size we get:
80/100 = 0.8
We conclude that a child between 2-5 years-old in this clinic experiences less than 1 UTI per year on average.
Educational Objective:
Understand the measures of center (mean/averages, median and mode). They are common topics on the
USMLE exams.
Time Spent: 13 seconds Copyright USMLEWorld,LLC. Last updated: [8/22/2014]

Block Tim e Rem aining: 07


T u to r

: 59

Feedback Suspend End Block

Item: 28 o f 46

11 f M a rk

Q. Id : 1189 [549201]

<a
P re v io u s

E>
Next

II
Lab V a lu e s

Bp*-
N o te s C a lc u la to r

Researchers are interested in the association between colorectal carcinoma and non-steroidal
anti-inflammatory drug (NSAID) use. First, they interviewed patients with biopsy-proven colorectal
carcinoma. Then they interviewed the patients' neighbors of similar age and race as controls. The study
design used by the researchers helps to control which of the following potential problems?

O
O
O
O
O

A. Selection bias
B. Recall bias
C. Observer's bias
D. Ascertainment bias
E. Confounding

Block Tim e Rem aining: 08 :


T u to r

09

Feedback Suspend End Block

Item: 28 o f 46

11 f M ark

Q. Id : 1189 [549201]

<a
Previous

E>
Next

Researchers are interested in the association between colorectal carcinoma and non-steroidal
anti-inflammatory drug (NSAID) use. First, they interviewed patients with biopsy-proven colorectal
carcinoma. Then they interviewed the patients' neighbors of similar age and race as controls. The study
design used by the researchers helps to control which of the following potential problems?
Lab Values Notes Calculator
A

O
O
O

A. Selection bias [46%]


B. Recall bias [7%]
C. Observer's bias [6%]

D. Ascertainment bias [6%]


() E. Confounding [34%]
Explanation: User Id: 549201
Confounding refers to the confusion of two supposedly causal variables such that all or part of an observed
effect thought due to one variable is actually due to the other. Matching is one method to control confounding
and is used in case-control studies. The initial step involves selecting variables that could be confounders
(e.g., age, race). Cases and controls are then selected based on the matching variables, such that both
groups have a similar distribution in accordance with the variables.
In this scenario, neighborhood "controls" were matched to "cases" by age and race. Selecting neighbors as
controls has another advantage of matching the cases to controls by variables that are difficult to measure
(e.g., socioeconomic status). Gender and smoking status are other common confounders.
(Choice A) Selection bias is not controlled by matching, because the controls selected may not reflect the
exposure experience of the general population.
(Choices B, C and D) Recall bias, observer's bias, and ascertainment bias result from mislabeling
exposed/unexposed or cases/controls. These are not affected by matching.
Educational Objective:
Matching is used in case-control studies in order to control confounding. Remember: matching variables
should always be the potential confounders of the study (e.g., age, race). Cases and controls are then
selected based on the matchina variables, such that both groups have a similar distribution in accordance

Block Tim e Rem aining: 08 :19


T u to r

Wfk

Feedback Suspend End Block

Item: 28 of 46 1

I^M ark < 1 O i l iR l


Q. Id : 1189 [549201] Previous Next Lab Values Notes Calculator

anti-innammaiory arug [in ^ a iu ; use. t-irst, mey inierviewea pauents wun Diopsy-proven coioreciai
carcinoma. Then they interviewed the patients' neighbors of similar age and race as controls. The study
design used by the researchers helps to control which of the following potential problems?

O
O
O
O

A. Selection bias [46%]


B. Recall bias [7%]
C. Observer's bias [6%]

D. Ascertainment bias [6%]


* () E. Confounding [34%]
Explanation: User Id: 549201
Confounding refers to the confusion of two supposedly causal variables such that all or part of an observed
effect thought due to one variable is actually due to the other. Matching is one method to control confounding
and is used in case-control studies. The initial step involves selecting variables that could be confounders
(e g., age, race). Cases and controls are then selected based on the matching variables, such that both
groups have a similar distribution in accordance with the variables.
In this scenario, neighborhood "controls" were matched to "cases" by age and race. Selecting neighbors as
controls has another advantage of matching the cases to controls by variables that are difficult to measure
(e.g., socioeconomic status). Gender and smoking status are other common confounders.
(Choice A) Selection bias is not controlled by matching, because the controls selected may not reflect the
exposure experience of the general population.
(Choices B, C and D) Recall bias, observer's bias, and ascertainment bias result from mislabeling
exposed/unexposed or cases/controls. These are not affected by matching.
Educational Objective:
Matching is used in case-control studies in order to control confounding. Remember: matching variables
should always be the potential confounders of the study (e.g., age, race). Cases and controls are then
selected based on the matching variables, such that both groups have a similar distribution in accordance
with the variables.
Time Spent: 30 seconds Copyright USMLEWorld,LLC. Last updated: [8/22/2014]

Block Tim e Rem aining: 08


T u to r

Feedback Suspend End Block

Item: 29 o f 46
Q. Id : 1271 [549201]

<a
P re v io u s

11 f M a rk

: 23

qp o

E>
Next

II
Lab V a lu e s

Bp*-
N o te s C a lc u la to r

Clinicians are developing an enzyme-linked immunosorbent assay test for diagnosing rheumatoid arthritis.
The test is designed to detect the presence of serum antibodies against citrullinated proteins. Two test
populations with a differing prevalence of rheumatoid arthritis are selected. The clinicians plan to assess the
test's performance in the 2 populations by comparing a number of diagnostic test parameters. The difference
in disease prevalence is most likely to affect which of the following parameters?

O
O
O
O
O

A. Negative likelihood ratio


B. Positive likelihood ratio
C. Positive predictive value
D. Sensitivity
E. Specificity

Block Tim e Rem aining: 08 :


T u to r

25

Feedback Suspend End Block

Item: 29 o f 46

11 f M ark

Q. Id : 1271 [549201]

<a
Previous

E>
Next

Clinicians are developing an enzyme-linked immunosorbent assay test for diagnosing rheumatoid arthritis.
The test is designed to detect the presence of serum antibodies against citrullinated proteins. Two test
populations with a differing prevalence of rheumatoid arthritis are selected. The clinicians plan to assess the
test's performance in the 2 populations by comparing a number of diagnostic test parameters. The difference
in disease prevalence is most likely to affect which of the following parameters?
Lab Values Notes Calculator
A

O
O

A. Negative likelihood ratio [0%]

O
O

D. Sensitivity [0%]

B. Positive likelihood ratio [0%]


* (> C. Positive predictive value [100%]
E. Specificity [0%]
Explanation: User Id: 549201

Common performance measures for diagnostic tests


Parameter Definition Calculation
Sensitivity
The probability of a
diseased person
testing positive
True positives
ouiisiuYiiy
True positives + False negatives
Specificity
The probability of a
non-diseased person
testing negative
True negatives
opecmciiy True negatives + False positives
Positive The probability that

Si*
True positives
nn\ / _
Block Tim e Rem aining: 08 :
T u to r

35

Io

Feedback Suspend End Block

Item: 29 o f 46

11 f M ark

Q. Id : 1271 [549201]

<a
P re v io u s

E>
Next

Common performance measures for diagnostic tests


Parameter Definition Calculation
Sensitivity
The probability of a
diseased person
testing positive
True positives
True positives + False negatives
Specificity
The probability of a
non-diseased person
testing negative
True negatives
opeciTicity True negatives + False positives
Positive
predictive
value
The probability that
disease is present
given a positive result
True positives
PPV= --------------------------------------------True positives + False positives
Negative
predictive
value
The probability that
disease is absent given
a negative result
True negatives
NPV = --------------------------------------------True negatives + False negatives
Positive
likelihood
ratio
A ratio representing the
likelihood of having the
disease given a

positive result
Sensitivity
LR+ = -------------------1 - Specificity
Negative
likelihood
ratio
A ratio representing the
likelihood of having the
disease given a
negative result
1 - Sensitivity
LR- = -------------------Specificity
Block Tim e Rem aining: 08 : 42
T u to r

il
Lab Values
Feedback
Notes
Suspend
Calculator
AV

End Block

Item
Q. Id:
:

29 of 46

271 [549201] 11 f M ark Previous<


1

NextO Lab Valuesi

NotesW & -' CalculatorH

lik e lih o o d
ra tio
likelihood o f having the
LR+ =

WOI IOIU V ILJ


A

disease given a
positive result 1 - Specificity
N eg ative
lik e lih o o d
ra tio
A ratio representing the
likelihood o f having the
LR- =
1 - Sensitivity
disease given a
negative result Specificity
USMLEWorW. LLC

The positive predictive value (PPV) is the probability that someone who tests positive on a binary diagnostic
test actually has the disease. It is calculated by dividing the number of true-positive results by the total
number of positive results. If the specificity of the test is <100%, there will be false-positive results (ie,
subjects who test positive who do not actually have the disease). Populations having a lower disease
prevalence will have fewer true-positive results. Consequently, false positives will make up a greater
proportion of all positive test results, and the PPV will therefore be lower. As disease prevalence increases,
the PPV will also increase due to a higher proportion of true positives. Similarly, if the sensitivity of the test is
<100%, the negative predictive value will increase as the disease prevalence decreases.
(Choices A and B) Positive and negative likelihood ratios indicate how a respective positive or negative test
result influences the probability of having a disease. Likelihood ratios >1 indicate that the respective test result
is associated with the presence of the disease; likelihood ratios <1 mean that the test result is associated with
the absence of the disease. Because positive and negative likelihood ratios are based on sensitivity and
specificity, they will not be affected by disease prevalence.
(Choices D and E) Sensitivity and specificity are not affected by disease prevalence. This is because

sensitivity is calculated using true positives and false negatives (only people with the disease), and specificity
is calculated using true negatives and false positives (only people without the disease).
Educational objective:
Various parameters are used to evaluate the accuracy and usefulness of diagnostic tests. Positive and
negative predictive values are influenced by disease prevalence; sensitivity, specificity, and likelihood ratios
are not.
Time Spent: 7 seconds Copyright USMLEWorld,LLC. Last updated: [7/28/2014]

Block Tim e Rem aining:


T u to r

08

: 46

qp o

Feedback Suspend End Block

Item: 30 o f 46

11 f M a rk

Q. Id : 1280 [549201]

<a
P re v io u s

E>
Next

II
Lab V a lu e s

Bp*-
N o te s C a lc u la to r

A prospective study evaluates the relationship between regular antioxidant use (vitamin C and vitamin E) and
the risk of stroke in men 40-60 years of age. The results of the study were adjusted to account for any
confounding variables. According to the study results, the men who consumed antioxidants for at least 5
years have a stroke relative risk of 0.75 (p < 0.01), while the men who consumed antioxidants for less than 5
years have a stroke relative risk of 0.95 (p = 0.45). Which of the following factors best explains why the
relative risk of stroke decreases with longer antioxidant use?

O
O
O
O
O

A. Selection bias
B. Lead time bias
C. Latent period
D. Rare disease assumption
E. Observer bias

Block Tim e Rem aining:


T u to r

08

53

Feedback Suspend End Block

Item: 30 o f 46

11 L Mark

Q. Id : 1280 [549201]

<a
P re v io u s

E>
Next

II
Lab Values

A prospective study evaluates the relationship between regular antioxidant use (vitamin C and vitamin E) and
the risk of stroke in men 40-60 years of age. The results of the study were adjusted to account for any
confounding variables. According to the study results, the men who consumed antioxidants for at least 5
years have a stroke relative risk of 0.75 (p < 0.01), while the men who consumed antioxidants for less than 5
years have a stroke relative risk of 0.95 (p = 0.45). Which of the following factors best explains why the
relative risk of stroke decreases with longer antioxidant use?

O
O

A. Selection bias [18%]

D. Rare disease assumption [4%]

B. Lead time bias [39%]


* C. Latent period [31 %]

E. Observer bias [8%]


Explanation: User Id: 549201
This study reveals that a minimum of 5 years of antioxidant use is associated with a 0.75 relative risk for
stroke, a finding that is statistically significant (p < 0.01). The use of antioxidants for less than 5 years
demonstrated a smaller decrease in risk (relative risk of 0.95). However, the p-value of 0.45 for this result
indicates that this small reduction in risk is likely due to chance alone. Thus, the use of antioxidants for less
than 5 years seems to have no appreciable benefit on stroke risk reduction. These results are explained by
assuming that stroke risk reduction through antioxidant use is associated with a long latent period.
Latency is a very important issue to consider when studying disease epidemiology. In most infectious
diseases, the latent period (i.e., the time elapsed from initial exposure to clinically apparent disease) is
relatively short. On the other hand, the pathogenesis of some disease processes (e g., cancer or heart
disease) may demonstrate a very long latent period before clinical manifestations develop. The concept of a
latency period can also be extended to risk factors and risk reducers. Exposure to risk modifiers sometimes
occurs a significant amount of time before the exposure's effect on the disease process is clinically evident.
Additionally, sometimes exposure to risk modifiers must occur continuously over extended periods of time
before disease outcome is affected. In this case, at least five years of continuous exposure to antioxidants
were required to reveal the protective effect of the exposure on the outcome (stroke).

Block Tim e Rem aining:

09:12

Tutor
Feedback
Notes
Suspend
Calculator
A

End B lo ck

Item: 30 o f 46

11 L Mark

Q. Id : 1280 [549201]

<a
P re v io u s

E>
Next

II
Lab V a lu e s

Bp?-Notes C a lc u la to r

assuming that stroke risk reduction through antioxidant use is associated with a long latent period.
Latency is a very important issue to consider when studying disease epidemiology. In most infectious
diseases, the latent period (i.e., the time elapsed from initial exposure to clinically apparent disease) is
relatively short. On the other hand, the pathogenesis of some disease processes (e.g., cancer or heart
disease) may demonstrate a very long latent period before clinical manifestations develop. The concept of a
latency period can also be extended to risk factors and risk reducers. Exposure to risk modifiers sometimes
occurs a significant amount of time before the exposure's effect on the disease process is clinically evident.
Additionally, sometimes exposure to risk modifiers must occur continuously over extended periods of time
before disease outcome is affected. In this case, at least five years of continuous exposure to antioxidants
were required to reveal the protective effect of the exposure on the outcome (stroke).
(Choice A) Selection bias can occur with inappropriate (non-random) selection methods or through selective
attrition of the study participants. This creates a study population that does not accurately represent the
actual population, and can result in erroneous conclusions regarding the exposure-disease relationship.
(Choice B) Lead time can be defined as the time between the initial detection of a disease and a specific
outcome or measured endpoint. Lead time bias can occur when comparing two different tests for the same
disease when one of the tests can accurately detect the disease at an earlier stage. A study comparing
disease survival times may erroneously conclude that using the earlier-detection test prolongs survival; when
in actuality, the increased survival time is due solely to earlier detection of the disease.
(Choice D) The rare disease assumption refers to the fact that the odds ratio approximates the relative risk
when disease prevalence is low.
(Choice E) Observer bias occurs when an observer misclassifies data secondary to individual differences in
interpretation or preconceived expectations regarding the study. It can be reduced by performing a double
blind study (both observers and participants unaware of randomized parameters) and by having multiple
observers encode and verify the recorded data.
Educational objective:
The concept of a latent period can be applied to both disease pathogenesis and exposure to risk modifiers.

The initial steps in pathogenesis and/or exposure to a risk factor sometimes occur years before clinical
manifestations of a disease are evident. Additionally, exposure to risk modifiers may need to be continuous
over a certain period of time before influencing the outcome.
Time Spent: 36 seconds Copyright USMLEWorld,LLC. Last updated: [3/30/2014]

Block Tim e Rem aining:


T u to r

09

16

qp o

Feedback Suspend End Block

Item: 31 o f 46

11 f M a rk

Q. Id : 1284 [549201]

<a
P re v io u s

E>
Next

II
Lab V a lu e s

Bp*-
N o te s C a lc u la to r

A new serologic test for detecting prostate cancer is negative in 95% of patients who do not have the disease.
If the test is used on 8 blood samples taken from patients without prostate cancer, what is the probability of

getting at
ast 1 positive test result?

o
o
o
o
o

A. 1 - (0.05 * 8)
B. 0.05 * 8
c . 0.05s
D. 0.95s
E. 1 - 0.95s

Block Tim e Rem aining:


T u to r

09 :1 9

Feedback Suspend End Block

Item: 31 o f 46

11 ^Mark

Q. Id : 1284 [549201]

<a
Previous

E>
Next

A new serologic test for detecting prostate cancer is negative in 95% of patients who do not have the disease.
If the test is used on 8 blood samples taken from patients without prostate cancer, what is the probability of
getting at least 1 positive test result?
Lab Values Notes Calculator
A

o A. 1 - (0.05 * 8) [19%]
o B. 0.05 * 8 [37%]
o c. 0.05s [11%]
o D. 0.95s [2%]
* () E. 1 -0.95s[31 %]
Explanation: User Id: 549201
This question describes a serologic test that has a specificity of 95%. The scenario states that the test is
used on 8 blood samples from patients without prostate cancer. Each test represents an independent event
with a 0.95 (95%) probability of giving a true-negative result and 0.05 (5%) probability of giving a false-positive
result. To calculate the chance of all 8 tests being negative, use the multiplication rule for independent events:
Probability (all negative) = 0.95s (Choice D)

It is important to know that the total probability is always equal to 1.0 (100%). The probability that at least 1
test turns out positive is thus:
Probability (at least 1 positive) = 1 - Probability (all negative) = 1 - 0.95s
Imagine 8 bowls, each containing 95 white balls and 5 black balls. By randomly taking 1 ball from 1 bowl, you
have a 95% probability of taking a white ball. If you instead take 1 ball from every bowl, the probability that all 8
balls you took are white is 0.95s. The chance of at least 1 of the 8 balls being black is 1 - 0.95s. In addition,
0.05s calculates the probability that all 8 balls you took are black (and of all 8 tests being positive) (Choice C).
(Choice B) The probabilities for a series of independent events are calculated using multiplication, not
addition. For example, assume you pull balls from 20 bowls instead of 8. Adding the probabilities of pulling a
black ball for each trial would give a total probability of 0.05 * 20 = 1.0 (100%). This is incorrect as there is
always a chance of never pulling a black ball no matter how many bowls you take from. In fact, after 20 pulls v

Block Tim e Rem aining:


T u to r

11

: 02

Feedback Suspend End Block

Item: 31 o f 46

11 f Mark

Q. Id : 1284 [549201]

U ' b. U.LIb " 8 |3/% J

O C 0.05s [11%]
O D 0.95s [2%]
</ E. 1 -0.95s [31 %]
<O
Previous Next Lab Values Notes Calculator

Explanation: User Id: 549201


This question describes a serologic test that has a specificity of 95%. The scenario states that the test is
used on 8 blood samples from patients without prostate cancer. Each test represents an independent event
with a 0.95 (95%) probability of giving a true-negative result and 0.05 (5%) probability of giving a false-positive
result. To calculate the chance of all 8 tests being negative, use the multiplication rule for independent events:
Probability (all negative) = 0.95s (Choice D)
It is important to know that the total probability is always equal to 1 .0 (100%). The probability that at least 1
test turns out positive is thus:
Probability (at least 1 positive) = 1 - Probability (all negative) = 1 - 0.95s
Imagine 8 bowls, each containing 95 white balls and 5 black balls. By randomly taking 1 ball from 1 bowl, you
have a 95% probability of taking a white ball. If you instead take 1 ball from every bowl, the probability that all 8
balls you took are white is 0.95s. The chance of at least 1 of the 8 balls being black is 1 - 0.95s. In addition,
0.05s calculates the probability that all 8 balls you took are black (and of all 8 tests being positive) (Choice C).
(Choice B) The probabilities for a series of independent events are calculated using multiplication, not
addition. For example, assume you pull balls from 20 bowls instead of 8. Adding the probabilities of pulling a
black ball for each trial would give a total probability of 0.05 * 20 = 1.0 (100%). This is incorrect as there is
always a chance of never pulling a black ball no matter how many bowls you take from. In fact, after 20 pulls
there is still a 0.9520 = 0.36 (36%) probability of having pulled only white balls.
Educational objective:
If events are independent, the probability that all events will turn out the same is the product of the separate
probabilities for each event. The probability of at least 1 event turning out differently is given as 1 - (probability
of all events being the same).
Time Spent: 202 seconds Copyright USMLEWorld,LLC. Last updated: [7/24/2014]

Block Tim e Rem aining: 11 : 06


T u to r

Feedback Suspend End Block

Item: 32 o f 46
Q. Id : 1274 [549201]

<a
P re v io u s

E>
Next

II
Lab V a lu e s

Bp*-
N o te s C a lc u la to r

11 f M a rk

A prospective cohort study was conducted to assess the role of daily alcohol consumption in the occurrence
of breast carcinoma. The investigators reported a 5-year relative risk of 1.4 for people who consume alcohol
compared to those who do not. The 95% confidence interval was 1.02 to 1.85. Which of the following 'p'
values is most consistent with the results described above?

o
o
o
o
o

A. 0.04
B. 0.06
C. 0.09
D. 0.11
E. 0.20

Block Tim e Rem aining:


T u to r

11 : 09

Feedback Suspend End Block

Item: 32 o f 46

11 f M a rk

Q. Id : 1274 [549201]

<a
P re v io u s

E>
Next

A prospective cohort study was conducted to assess the role of daily alcohol consumption in the occurrence
of breast carcinoma. The investigators reported a 5-year relative risk of 1.4 for people who consume alcohol
compared to those who do not. The 95% confidence interval was 1.02 to 1.85. Which of the following 'p'
values is most consistent with the results described above?
Lab V a lu e s N o te s C a lc u la to r
A

() A. 0.04 [64%] o B. 0.06 [13%] o c. 0.09 [10%] o D. 0.11 [6%] o E. 0.20 [7%]
Explanation: User Id: 549201
Relative risk (RR) is used in cohort studies to determine how strongly a risk factor (ie, exposure) is
associated with an outcome. Relative risk can be expressed as the following equation:
(Mortality rate [or incidence] in exposed group)
(Mortality rate [or incidence] in unexposed group)
If the RR is 1.0 (null value), there is no association between the exposure and the disease. A RR greater than
1.0 indicates that the exposure is associated with increased disease occurrence. A RR less than 1.0 means
that the exposure is associated with decreased disease occurrence.
One inherent weakness of the RR is that it does not account for the possibility that chance alone is
responsible for the results. The 95% confidence interval and p value are two measures of statistical
significance that can help strengthen the findings of a study using RR. To be statistically significant, the
confidence interval must NOT contain the null value of 1.0. When the confidence interval lacks the null value,
this equates to finding that the p value is less than 0.05 and that the association between exposure and
outcome is statistically significant. A p value less than 0.05 indicates there is a less than 5% likelihood that
the results obtained were due to chance alone.
In our theoretical case, the RR is 1.4 and the 95% confidence interval is between 1.02 and 1.85. It can be

Block Tim e Rem aining:


T u to r

11 : 14

Wfk

Feedback Suspend End Block

Item: 32 o f 46

11 f M a rk

Q. Id : 1274 [549201]

O D. 0.11 [6%]
O E. 0.20 [7%]
<O
P re v io u s N e x t Lab V a lu e s N o te s C a lc u la to r

Explanation: User Id: 549201


Relative risk (RR) is used in cohort studies to determine how strongly a risk factor (ie, exposure) is
associated with an outcome. Relative risk can be expressed as the following equation:
(Mortality rate [or incidence] in exposed group)
(Mortality rate [or incidence] in unexposed group)
If the RR is 1.0 (null value), there is no association between the exposure and the disease. A RR greater than
1.0 indicates that the exposure is associated with increased disease occurrence. A RR less than 1.0 means
that the exposure is associated with decreased disease occurrence.
One inherent weakness of the RR is that it does not account for the possibility that chance alone is

responsible for the results. The 95% confidence interval and p value are two measures of statistical
significance that can help strengthen the findings of a study using RR. To be statistically significant, the
confidence interval must NOT contain the null value of 1.0. When the confidence interval lacks the null value,
this equates to finding that the p value is less than 0.05 and that the association between exposure and
outcome is statistically significant. A p value less than 0.05 indicates there is a less than 5% likelihood that
the results obtained were due to chance alone.
In our theoretical case, the RR is 1.4 and the 95% confidence interval is between 1.02 and 1.85. It can be
concluded then that daily alcohol consumption is associated with an increased occurrence of breast
carcinoma and that the findings are statistically significant. The p value would be expected to measure less
than 0.05.
(Choices B, C, D, and E) The other choices contain p values in excess of 0.05.
Educational Objective:
Know how to interpret the confidence interval. Know the relationship between the confidence interval and 'p'
value.
Time Spent: 8 seconds Copyright USMLEWorld,LLC. Last updated: [8/22/2014]

Block Tim e Rem aining:


T u to r

11 : 18

Feedback Suspend End Block

Item: 33 o f 46

11 f M a rk

Q. Id : 1188 [549201]

<a
P re v io u s

E>
Next

II
Lab V a lu e s

Bp*-
N o te s C a lc u la to r

A large prospective study evaluates the relationship between alcohol consumption and breast cancer Four
thousand middle-aged women are enrolled in the study through a random selection of residential addresses.
Daily alcohol consumption and breast cancer incidence are assessed through the use of periodic
questionnaires. Five-year follow-up shows that alcohol consumers are somewhat more likely to develop
breast cancer (relative risk = 1.32, 95% confidence interval = 0.90-1.85). The investigators also report that
800 subjects have been lost to follow-up by the end of the study, the majority of which were moderate to heavy
alcohol consumers. According to this information, which of the following biases is most likely to be present?

O
O
O
O
O

A. Lead time bias


B. Observer bias
C. Recall bias
D. Sampling bias
E. Selection bias

Block Tim e Rem aining: 11 : 21


T u to r

Feedback Suspend End Block

Item: 33 o f 46
Q. Id : 1188 [549201]

<a
P re v io u s

E>
Next

II
Lab V a lu e s

Bp*-
N o te s C a lc u la to r

11 f M a rk

A large prospective study evaluates the relationship between alcohol consumption and breast cancer Four
thousand middle-aged women are enrolled in the study through a random selection of residential addresses.
Daily alcohol consumption and breast cancer incidence are assessed through the use of periodic
questionnaires. Five-year follow-up shows that alcohol consumers are somewhat more likely to develop
breast cancer (relative risk = 1.32, 95% confidence interval = 0.90-1.85). The investigators also report that
800 subjects have been lost to follow-up by the end of the study, the majority of which were moderate to heavy
alcohol consumers. According to this information, which of the following biases is most likely to be present?

O
O
O
O

A. Lead time bias [14%]


B. Observer bias [6%]
C. Recall bias [10%]

D. Sampling bias [36%]


* E. Selection bias [34%]
Explanation: User Id: 549201

Attrition bias
Exposed & unexposed

Block Tim e Rem aining:


T u to r

11 : 3 2

Feedback Suspend End Block

Item: 33 o f 46

11 L M ark

Q. Id : 1188 [549201]

<a
P re v io u s

E>
Next

II
Lab V a lu e s

Bp?-N o te s C a lc u la to r

Loss to follow-up is a problem in prospective studies because it creates the potential for a type of selection
bias known as attrition bias. This form of bias does not occur when the losses happen equally and randomly
between the exposed and unexposed groups, as this simply leads to a smaller study population. However, if
the loss to follow-up occurs disproportionately between the exposed and unexposed groups, attrition bias can
result if the lost subjects differ in their risk of developing the outcome compared to the remaining subjects.
Attrition bias can lead to overestimation or underestimation of the association between the exposure and the
outcome.
In this case, a substantial number of subjects (20%) were lost to follow-up, primarily those with higher levels
of alcohol consumption. If these lost subjects had a higher incidence of breast cancer than those remaining in
the study, then a selective loss of high-risk subjects would have occurred in the exposed group. As a result,
the measure of the association between alcohol use and breast cancer would be underestimated (ie, no
significant association was seen).
(Choice A) Lead-time bias occurs when a screening test diagnoses a disease earlier than it would have
appeared by natural history alone, so that the time from diagnosis until death appears prolonged even though
there might actually be no improvement in survival.
(Choice B) Observer bias can occur in clinical trials when study participants or investigators are aware of
individual treatment assignments. It can be prevented by performing a double-blind study in which neither
participants nor investigators are aware of treatment assignments.
(Choice C) Recall bias results from inaccurate recall of past exposure status and is a potential problem for
retrospective studies such as case-control studies. In this prospective study, exposure status is determined
at the present through periodic questionnaires that assess daily alcohol consumption.
(Choice D) Sampling bias is a type of selection bias that occurs due to nonrandom sampling of a target
population. This causes the characteristics of the study population to differ from those of the target
population, leading to results that may not be generalizable. In this study, participants were selected randomly
based on their residential address.
Educational objective:
In prospective studies, disproportionate loss to follow-up between the exposed and unexposed groups creates
the potential for attrition bias, which is a form of selection bias. As a result, investigators try to achieve high

patient follow-up rates in prospective studies.


Time Spent: 11 seconds Copyright USMLEWorld,LLC. Last updated: [4/2/2014]

Block Tim e Rem aining:


T u to r

11 : 4 4

qp o

Feedback Suspend End Block

Item: 34 o f 46

11 f M a rk

Q. Id : 1173 [549201]

<a
P re v io u s

E>
Next

II
Lab V a lu e s

Bp*-
N o te s C a lc u la to r

Researchers conduct a prospective study that demonstrates an association between alcohol consumption
and transitional bladder carcinoma, with a relative risk (RR) of 1.81 and p-value of 0.03. They then divide the
study subjects into 2 groups, smokers and nonsmokers, and again examine the association between alcohol
consumption and bladder cancer:
Smokers: RR = 0.95; p = 0.87
Nonsmokers: RR = 1.03; p = 0.96
The discrepancy between the overall results and the stratified results is best explained by which of the
following?

O
O
O
O
O
O

A. Effect modification
B. Observer bias
C. Measurement bias
D. Recall bias
E. Confounding
F. Meta-analysis

Block Tim e Rem aining:


T u to r

11 : 4 8

Feedback Suspend End Block

Item: 34 o f 46

11 f M a rk

Q. Id : 1173 [549201]

<a
P re v io u s

E>
Next

II
Lab V a lu e s

Bp*-
N o te s C a lc u la to r

Researchers conduct a prospective study that demonstrates an association between alcohol consumption
and transitional bladder carcinoma, with a relative risk (RR) of 1.81 and p-value of 0.03. They then divide the
study subjects into 2 groups, smokers and nonsmokers, and again examine the association between alcohol
consumption and bladder cancer:
Smokers: RR = 0.95; p = 0.87
Nonsmokers: RR = 1.03; p = 0.96
The discrepancy between the overall results and the stratified results is best explained by which of the
following?

O
O

A. Effect modification [19%]


B. Observer bias [3%]

O
O

C. Measurement bias [8%]

D. Recall bias [2%]


* E. Confounding [63%]

F. Meta-analysis [4%]
Explanation: User Id: 549201

Initial crude analysis confounded by smoking status


Bladder cancer
Yes No
Alcohol
Yes 23 207
use
No 26 444

Block Tim e Rem aining:


T u to r

12 : 07

Feedback Suspend End Block

Item: 34 o f 46

11 ^ M a r k

Q. Id : 1173 [549201]

<a
P re v io u s

E>
Next

II
Lab V a lu e s

Bp?-N o te s C a lc u la to r

Stratified analysis removes confounding effects of smoking


Smokers Non-smokers
Alcohol
use
p-value= 0.87
Alcohol
use
p-value= 0.96
0 USMLEWorld. LLC

This scenario depicts a typical example of confounding. Confounding bias occurs when the
exposure-disease relationship is muddied by the effect of a confounding variable. Confounders are
extraneous factors that correlate with both the exposure and the disease. The initial crude analysis of the
study data suggested that alcohol consumption was associated with bladder carcinoma. However, this
analysis did not account for the effect of smoking, a potential confounder. Smoking is associated with both
bladder carcinoma and significant alcohol consumption (ie, people who drink a lot are more likely to smoke).
As a result, smoking is a potential confounder that may explain some or all of the association observed
between alcohol consumption and bladder cancer.
Confounding effects can be negated by running separate analyses for the smokers and non-smokers (a
technique called stratified analysis). This allows calculation of the true, unconfounded RR that alcohol users
have of developing bladder cancer. In the example above, stratified analysis shows that both the smoking and
nonsmoking groups have an RR close to 1. Their large p-values further suggest that the small difference in
the RR from the null value of 1 is due to chance alone. Thus, the RR of 1.81 found on crude analysis
disappears and there is no true association between alcohol consumption and bladder cancer.
(Choice A) Effect modification results when an external variable positively or negatively impacts the observed

Block Tim e Rem aining:


T u to r

Feedback Suspend End Block

Item: 34 o f 46
Q. Id : 1173 [549201]

<a

11 F M a rk

12 : 13

Wfk

P re v io u s

E>
Next

II
Lab V a lu e s

Bp?-N o te s C a lc u la to r

This scenario depicts a typical example of confounding. Confounding bias occurs when the
exposure-disease relationship is muddied by the effect of a confounding variable. Confounders are
extraneous factors that correlate with both the exposure and the disease. The initial crude analysis of the
study data suggested that alcohol consumption was associated with bladder carcinoma. However, this
analysis did not account for the effect of smoking, a potential confounder. Smoking is associated with both
bladder carcinoma and significant alcohol consumption (ie, people who drink a lot are more likely to smoke).
As a result, smoking is a potential confounder that may explain some or all of the association observed
between alcohol consumption and bladder cancer.
Confounding effects can be negated by running separate analyses for the smokers and non-smokers (a
technique called stratified analysis). This allows calculation of the true, unconfounded RR that alcohol users
have of developing bladder cancer. In the example above, stratified analysis shows that both the smoking and
nonsmoking groups have an RR close to 1. Their large p-values further suggest that the small difference in
the RR from the null value of 1 is due to chance alone. Thus, the RR of 1.81 found on crude analysis
disappears and there is no true association between alcohol consumption and bladder cancer.
(Choice A) Effect modification results when an external variable positively or negatively impacts the observed
effect of a risk factor on disease status. When this occurs, stratified analysis will reveal a significant
difference in risk between the stratified groups. Because there is no significant difference in RR between the
smoking and nonsmoking groups (p-values > 0.05), smoking status does not modify the effect of alcohol
consumption on bladder cancer. Thus, smoking status is a confounder and not an effect modifier.
(Choices B and C) Observer bias and measurement bias distort the strength of association by
misclassifying exposed/unexposed and/or diseased/nondiseased subjects. The scenario does not describe
any particular issues that could affect the classification process.
(Choice D) Recall bias results from inaccurate recall of past exposure by people in the study and applies
mostly to retrospective studies such as case-control studies.
(Choice F) Meta-analysis refers to compiling data from several studies to increase the power of analysis.
Educational objective:
Confounding bias occurs when the exposure-disease relationship is muddied by the effect of an extraneous
factor that has correlations with both the exposure and the disease. Confounding bias can result in the false
association of an exposure with a disease.
Time Spent: 30 seconds Copyright USMLEWorld,LLC. Last updated: [4/12/2014]

Block Tim e Rem aining: 12 : 16


T u to r

qp o

Feedback Suspend End Block

Item: 35 o f 46

11 f M a rk

Q. Id : 1210 [549201]

<a
P re v io u s

E>
Next

II
Lab V a lu e s

Bp*-
N o te s C a lc u la to r

Inflammatory biological markers are clinically useful in a number of different ways, including assessment of
disease activity in conditions such as systemic lupus erythematosus. A new inflammatory marker is being
studied. When the blood level of the new marker is measured along with the C-reactive protein (CRP) level,
the following plot is obtained:
Based on the plot, the correlation coefficient between the two variables is closest to:

o
o
o
o
o

A. + 0.8
B. + 0.2
Oo
D.-0.2
E.-0.8

Block Tim e Rem aining:


T u to r

12 : 27

Feedback Suspend End Block

Item: 35 o f 46

11 f M a rk

Q. Id : 1210 [549201]

<a
P re v io u s

E>
Next

II
Lab V a lu e s

Bp*-
N o te s C a lc u la to r

Inflammatory biological markers are clinically useful in a number of different ways, including assessment of
disease activity in conditions such as systemic lupus erythematosus. A new inflammatory marker is being
studied. When the blood level of the new marker is measured along with the C-reactive protein (CRP) level,
the following plot is obtained:
Based on the plot, the correlation coefficient between the two variables is closest to:
o A. + 0.8 [6%]
o B. + 0.2 [7%]
o c. 0 [9%]
o D. - 0.2 [10%]
* <s> E. - 0.8 [68%]
Explanation: User Id: 549201
Scatter plots are useful for crude analysis of data. Such plots can demonstrate the type of association (i.e.,
linear, non-linear), if one is present. If a linear association is present, the correlation coefficient can be

Block Tim e Rem aining:


T u to r

12 : 41

Wfk

Feedback Suspend End Block

Item: 35 o f 46

11 f M ark

Q. Id : 1210 [549201] Lab V a lu e s

Based on the plot., the correlation coefficient between the two variables is closest to:

o A. + 0.8 [6%]
o B. + 0.2 [7%]
o c. 0 [9%]
o D. - 0.2 [10%]
* <) E. - 0.8 [68%]
Explanation: User Id: 549201
Scatter plots are useful for crude analysis of data. Such plots can demonstrate the type of association (i.e.,
linear, non-linear), if one is present. If a linear association is present, the correlation coefficient can be
calculated to provide a numerical description of the linear association. The correlation coefficient ranges from
-1 to +1 and describes two important characteristics of an association: the strength and the polarity.
A correlation coefficient of 0 indicates that there is no association (i.e., a random distribution). In this case, the
scatter plot shows a linear association between the blood level of the new marker and that of CRP, so we
know that the correlation coefficient is not 0 (Choice C). The scatter plot above shows that an increase in
CRP level is associated with a decrease in the new marker's level. Therefore, you should expect the
correlation coefficient to be negative (Choices A and B). That leaves only the two negative values, -0.2 and
-0.8. Although the scatter plot does not demonstrate a perfect linear arrangement of measurements, the plot

appears to demonstrate a reasonably strong linear association (Choice D). Thus, a correlation coefficient
value of -0.8 is the most appropriate answer among the options given.
It is important to note that the value of the correlation coefficient is invariant to the magnitude of the slope of
the regression line. For instance, assuming a data set with a maximum positive association (i.e., all data
points forming a linear line with a positive slope), the regression line will always have a correlation coefficient
of +1, even if its actual slope is 0.2, 5.5, or any other positive value.
Educational objective:
The correlation coefficient ranges from -1 to +1 and describes two important characteristics of an association:
the strength and the polarity.
Time Spent: 39 seconds Copyright USMLEWorld,LLC. Last updated: [3/7/2014]

Block Tim e Rem aining:


T u to r

12 : 4 5

Feedback Suspend End Block

Item: 36 o f 46

11 f M a rk

Q. Id : 1299 [549201]

<a
P re v io u s

E>
Next

II
Lab V a lu e s

Bp*-
N o te s C a lc u la to r

A researcher is interested in the blood folate level of women who live in a population with a high incidence of
neural tube defects. He takes a random sample of women aged 18-45 (n) and measures their blood folate
levels. He reports the mean, standard deviation (SD) and variance. He also states that he is 95% confident
that the population mean of the blood folate level lies somewhere between 2.4 and 4.6 ng/ml. The last
statement is based on which of the following calculations?
O A. Mean +. 1.96*SD/Vn
O B. Mean+.1.96*SD
O C. Mean 2.58*SD/Vn
O D. Mean +.2.58*SD
O E. Mean SD/r\

Block Tim e Rem aining:


T u to r

12 : 49

Feedback Suspend End Block

Item: 36 o f 46

11 f M ark

Q. Id : 1299 [549201]

<a
P re v io u s

E>
Next

II
Lab V a lu e s

Bp?-N o te s C a lc u la to r

A researcher is interested in the blood folate level of women who live in a population with a high incidence of
neural tube defects. He takes a random sample of women aged 18-45 (n) and measures their blood folate
levels. He reports the mean, standard deviation (SD) and variance. He also states that he is 95% confident
that the population mean of the blood folate level lies somewhere between 2.4 and 4.6 ng/ml. The last
statement is based on which of the following calculations?
/ () A. Mean +_ 1.96*SD/Vn [51 %]

o B. Mean +.1.96*SD [23%]


o c. Mean +. 2.58*SD/Vn [11%]
o D. Mean +.2.58*SD [7%]
o E. Mean + SD/n [7%]
Explanation: User Id: 549201
A 95% confidence interval (Cl) is the range of values in which one can be 95% confident that the true mean of
the underlying population falls. It is a common measure of central tendency reported in epidemiological
studies. In order to calculate the Cl, one needs to know the mean, SD, z-score and sample size.
Standard error of the mean (SEM) is calculated using the formula SEM = SD/\'n. Notice that the sample size
is a part of the calculation. Thus, the Cl will tighten as the sample size increases. The next step is to multiply
the SEM with the corresponding z-score. For 95% Cl the z-score is 1.96. (Remember that 95% of the
observations lie within 2SD from the mean in a normal distribution.) For a 99% Cl the z-score is 2.58.
The final step is to obtain the confidence limits as shown below:
Mean +. 1,96 SD/Vn.
Educational Objective:
Confidence interval for the mean is calculated using the mean, SD, z-score and sample size.
Time Spent: 11 seconds Copyright USMLEWorld,LLC. Last updated: [8/22/2014]

Block Tim e Rem aining:


T u to r

12 : 56

Feedback Suspend End Block

Item: 37 o f 46

11 f M a rk

Q. Id : 1275 [549201]

<a
P re v io u s

E>
Next

II
Lab V a lu e s

Bp*-
N o te s C a lc u la to r

Kidney biopsy samples from 500 diabetic patients were examined by several pathologists. The pathologists
that knew the diabetes status of patients were three times more likely to interpret the sample microscopy
findings as "diabetic nephropathy." Which of the following most likely explains the difference in interpreting the
microscopy results?

O
O
O
O
O

A. Confounding
B. Lead-time bias
C. Recall bias
D. Selection bias
E. Observer bias

Block Tim e Rem aining:


T u to r

12 : 59

Feedback Suspend End Block

Item: 37 o f 46

11 f Mark

Q. Id : 1275 [549201]

<a
Previous

E>
Next

Kidney biopsy samples from 500 diabetic patients were examined by several pathologists. The pathologists
that knew the diabetes status of patients were three times more likely to interpret the sample microscopy
findings as "diabetic nephropathy." Which of the following most likely explains the difference in interpreting the
microscopy results?
Lab Values Notes Calculator
A

A. Confounding [4%]

O
O
O

B. Lead-time bias [3%]


C. Recall bias [6%]

D. Selection bias [7%]


* <> E. Observer bias [80%]
Explanation: User Id: 549201
Observer bias occurs when the investigator's decision is affected by prior knowledge of the exposure status.
In this case, some pathologists were prejudiced by the clinical history because they knew that diabetic
nephropathy is a common finding in patients with diabetes. The pathologists who were blinded to the patients'
diabetes status were not prejudiced by the clinical history and had unbiased interpretations.
(Choice A) Confounding is present when at least part of the exposure-disease relationship can be explained
by another variable (confounder). No information on possible confounders is given in this scenario.
(Choice B) Lead-time bias refers to the apparent prolongation of survival in patients who underwent a
screening test that allowed for earlier diagnosis but did not improve prognosis.
(Choice C) Recall bias results from inaccurate patient recall of past exposure. It is not applicable in this case.
(Choice D) Selection bias can result when subjects are selected for a study or from selective losses during
follow-up.
Educational Objective:
Observer bias occurs when the investigator's decision is affected by prior knowledge of the exposure status.
Time Spent: 11 seconds Copyright USMLEWorld,LLC. Last updated: [8/22/20141

_________________
Block Tim e Rem aining:
T u to r

13 : 07

Feedback Suspend End Block

Item: 38 o f 46

11 f M a rk

Q. Id : 1186 [549201]

<a
P re v io u s

E>
Next

II
Lab V a lu e s

Bp*-
N o te s C a lc u la to r

A large cohort study is conducted to assess the association between smoking and squamous cell carcinoma
of the esophagus among middle-aged Chinese men. During ten years of follow-up. smokers have had five
times the risk of esophageal carcinoma compared to non-smokers (RR = 5.0, 95% Cl = 2.9 - 7.1). According
to the study results, what percent of esophageal carcinoma in smokers can be attributed to smoking?

o
o
o
o
o

A. 25%
B. 50%
C. 70%
D. 80%
E. 90%

Block Tim e Rem aining:


T u to r

Feedback Suspend End Block

Item: 38 o f 46
Q. Id : 1186 [549201]

<a
P re v io u s

E>

11 f M a rk

13 : 14

Next

II
Lab V a lu e s

Bp?-N o te s C a lc u la to r

A large cohort study is conducted to assess the association between smoking and squamous cell carcinoma
of the esophagus among middle-aged Chinese men. During ten years of follow-up. smokers have had five
times the risk of esophageal carcinoma compared to non-smokers (RR = 5.0, 95% Cl = 2.9 - 7.1). According
to the study results, what percent of esophageal carcinoma in smokers can be attributed to smoking?

o A. 25% [13%]
o B. 50% [14%]
o c. 70% [12%]
() D. 80% [46 %]

E. 90% [14%]
Explanation: User Id: 549201
Attributable risk percent (ARP) or etiologic fraction is an important measure of the impact of a risk factor being
studied. ARP represents the excess risk in a population that can be explained by exposure to a particular risk
factor. It is calculated by subtracting the risk in the unexposed population (baseline risk) from the risk in the
exposed population and dividing the result by the risk in the exposed population:
ARP = (risk in exposed - risk in unexposed)/risk in exposed.
An easier way to calculate the ARP is to derive it from the relative risk (RR):
ARP = (RR - 1)/RR.
In this case, ARP = (5.0 - 1)/5.0 = 0.8 (80%). According to the study results, 80% of esophageal carcinoma in
smokers was attributable to smoking.
Educational objective:
ARP represents the excess risk in the exposed population that can be attributed to the risk factor. It can be
easily derived from the relative risk using the following formula: ARP = (RR -1 )/RR.
Time Spent: 26 seconds Copyright USMLEWorld,LLC. Last updated: [7/4/2014]

Block Tim e Rem aining:


T u to r

13 : 24

Feedback Suspend End Block

Item: 39 o f 46

11 L M ark

Q. Id : 1202 [549201]

<a
P re v io u s

E>
Next

II
Lab V a lu e s

Bp*-
N o te s C a lc u la to r

Officials of a large community hospital report an increased incidence of acute myelogenous leukemia (AML)
among children ages 5-12 years old. They observe that some households in the community are exposed to
chemical waste from a nearby factory and worry that exposure to this waste is responsible for the increased
incidence of leukemia. A case-control study is designed to evaluate the hospital officials' claim that exposure
to chemical waste increases the risk for developing AML in childhood. Which of the following populations is
most likely to function as the control group?

O
O
O
O
O

A. Children with exposure to the chemical waste who do not suffer from AML
B. Children without exposure to the chemical waste who do not suffer from AML
C. Children both with and without exposure to the chemical waste who do not suffer from AML
D. Children without exposure to the chemical waste who suffer from AML
E. Children with exposure to the chemical waste who suffer from AML

Block Tim e Rem aining:


T u to r

13 : 28

Feedback Suspend End Block

Item: 39 o f 46

11 L Mark

Q. Id : 1202 [549201]

<a
Previous

E>
Next

Officials of a large community hospital report an increased incidence of acute myelogenous leukemia (AML)
among children ages 5-12 years old. They observe that some households in the community are exposed to
chemical waste from a nearby factory and worry that exposure to this waste is responsible for the increased
incidence of leukemia. A case-control study is designed to evaluate the hospital officials' claim that exposure
to chemical waste increases the risk for developing AML in childhood. Which of the following populations is
most likely to function as the control group?
Lab Values Notes Calculator
A

O
O

A. Children with exposure to the chemical waste who do not suffer from AML [18%]

O
O

D. Children without exposure to the chemical waste who suffer from AML [31 %]

B. Children without exposure to the chemical waste who do not suffer from AML [23%]
* C. Children both with and without exposure to the chemical waste who do not suffer from AML [26%]
E. Children with exposure to the chemical waste who suffer from AML [2%]
Explanation: User Id: 549201
A case-control study is the most appropriate study design for evaluating the hospital officials' claim. This is
because the disease is known (AML), but a retrospective risk factor (chemical waste exposure) needs to be
evaluated. In case-control studies, two groups of subjects are created: cases (subjects with the disease of
interest) and controls (subjects without the disease of interest). In this example, children who suffer from AML
can only be used to form the cases group; they cannot be used as controls (Choices D and E). After the
case and control groups are selected, the frequency of exposure to the retrospective risk factor within both
groups is ascertained. If there is a statistically significant difference between the two groups, then it is
probable that the retrospective risk factor in question is indeed a true risk factor for disease development.
Selecting control subjects according to exposure status is inappropriate because comparing the frequency of
exposure between the case and control groups is an important part of case-control study analysis (Choices A
and B). Optimal selection of control subjects provides an unbiased estimation of exposure frequency
amongst the non-diseased population. Skewing the control group (e.g., through non-random selection) so
that it contains a higher or lower exposure frequency could result in erroneous analysis. Independent
variables not being tested (e.g., age and sex) are often specifically selected to be the same (matched)
between case and control groups, in order to decrease the effects of confounding. However, matching must
be carefully performed so as not to introduce selection bias.

Block Tim e Rem aining:


T u to r

13 : 3 6

Feedback Suspend End Block

Item: 39 o f 46

11 L M ark

Q. Id : 1202 [549201]

most likely to function as the control group?


Lab V a lu e s

O
O

A. Children with exposure to the chemical waste who do not suffer from AML [18%]

O
O

D. Children without exposure to the chemical waste who suffer from AML [31 %]

B. Children without exposure to the chemical waste who do not suffer from AML [23%]
* C. Children both with and without exposure to the chemical waste who do not suffer from AML [26%]
E. Children with exposure to the chemical waste who suffer from AML [2%]
Explanation: User Id: 549201
A case-control study is the most appropriate study design for evaluating the hospital officials' claim. This is
because the disease is known (AML), but a retrospective risk factor (chemical waste exposure) needs to be
evaluated. In case-control studies, two groups of subjects are created: cases (subjects with the disease of
interest) and controls (subjects without the disease of interest). In this example, children who suffer from AML
can only be used to form the cases group; they cannot be used as controls (Choices D and E) After the
case and control groups are selected, the frequency of exposure to the retrospective risk factor within both
groups is ascertained. If there is a statistically significant difference between the two groups, then it is
probable that the retrospective risk factor in question is indeed a true risk factor for disease development.
Selecting control subjects according to exposure status is inappropriate because comparing the frequency of

exposure between the case and control groups is an important part of case-control study analysis (Choices A
and B). Optimal selection of control subjects provides an unbiased estimation of exposure frequency
amongst the non-diseased population. Skewing the control group (e.g., through non-random selection) so
that it contains a higher or lower exposure frequency could result in erroneous analysis. Independent
variables not being tested (e.g., age and sex) are often specifically selected to be the same (matched)
between case and control groups, in order to decrease the effects of confounding. However, matching must
be carefully performed so as not to introduce selection bias.
Educational objective:
Selection of control subjects in case-control studies is intended to provide an accurate estimation of exposure
frequency among the non-diseased general population. Cases and controls are often matched in order to
decrease confounding. However, matching must be carefully performed so as to not introduce selection bias.
Time Spent: 13 seconds Copyright USMLEWorld,LLC. Last updated: [3/30/2014]

Block Tim e Rem aining:


T u to r

13 : 39

Feedback Suspend End Block

Item: 40 o f 46

11 f M a rk

Q. Id : 1201 [549201]

<a
P re v io u s

E>
Next

II
Lab V a lu e s

Bp*-
N o te s C a lc u la to r

A study of 400 patients hospitalized with diabetes mellitus-related complications shows that serum cholesterol
level is a normally distributed variable with a mean of 220 mg/dL and standard deviation of 10 mg/dL. Based
on the study results how many patients do you expect to have serum cholesterol > 240 mg/dL in this study?

Block Tim e Rem aining:


T u to r

13 : 4 4

Feedback Suspend End Block

11 ^Mark
A
A

A study of 400 patients hospitalized with diabetes mellitus-related complications shows that serum cholesterol
level is a normally distributed variable with a mean of 220 mg/dL and standard deviation of 10 mg/dL Based
on the study results how many patients do you expect to have serum cholesterol > 240 mg/dL in this study?
O A. 2 [11 %]
* B. 10 [50%]
O C. 20 [25%]
O D. 64 [9%]
O E. 128 [5%]
Explanation: User Id: 549201
A normal distribution refers to a symmetrical, bell-shaped distribution pattern. The characteristic nature of this
distribution allows one to predict what proportion of observations lie within particular limits from the mean.
The degree of dispersion from the mean is determined by standard deviation. For example, 68% of all the
values lie within 1 standard deviation from the mean. The remaining 32% of the values are therefore outside
of one standard deviation, with 16% of these above and 16% below one standard deviation from the mean. In
addition, 95% of values are within 2 standard deviations from the mean and 99.7 % are within 3 standard
deviations. With a mean of 220 and a standard deviation of 10, the given cutoff point of 240 mg/dL represents
2 standard deviations from the mean. Since 95% of values must be within the 2 standard deviations (200-240
mg/dL), 2.5% of the values must lie below 200 mg/dL and 2.5% above 240 mg/dL. Since there are 400
patients in the study, 10 patients (2.5 %) will have a level above 240 mg/dL. The following slide demonstrates
the concept. The slide below demonstrates the concept.

95%
V

Item: 40 o f 46
Q. Id : 1201 [549201]

11 L M ark

<a
P re v io u s

E>
Next

II
Lab V a lu e s

Bp?-N o te s C a lc u la to r

Explanation: User Id: 549201


A normal distribution refers to a symmetrical, bell-shaped distribution pattern. The characteristic nature of this
distribution allows one to predict what proportion of observations lie within particular limits from the mean.
The degree of dispersion from the mean is determined by standard deviation. For example, 68% of all the
values lie within 1 standard deviation from the mean. The remaining 32% of the values are therefore outside
of one standard deviation, with 16% of these above and 16% below one standard deviation from the mean. In
addition, 95% of values are within 2 standard deviations from the mean and 99.7 % are within 3 standard
deviations. With a mean of 220 and a standard deviation of 10, the given cutoff point of 240 mg/dL represents
2 standard deviations from the mean. Since 95% of values must be within the 2 standard deviations (200-240
mg/dL), 2.5% of the values must lie below 200 mg/dL and 2.5% above 240 mg/dL. Since there are 400
patients in the study, 10 patients (2.5 %) will have a level above 240 mg/dL. The following slide demonstrates
the concept. The slide below demonstrates the concept.

95%
(Choice A) 2 patients is equal to 0.5% of the study population. Only 1 or 2 patients at the most would be
expected to have a level of over 250 mg/dL (3 standard deviations).
(Choice C) 20 patients total (5%) would be expected to fall outside of 2 standard deviations from the mean.
However, half of these will be lower than two standard deviations (<200 mg/dL) and half will be greater than
two standard deviations from the mean (240 mg/dL).
(Choice D) 64 patients (16 % of the study population) will have a value that is one standard deviation above
the mean, but the cutoff value is 2 standard deviations above the mean.
[C h o ic e E l 128 natients /32%1 w ill have a value w hich lie?; nreater than 1 standard deviation aw av from the

Block Tim e Rem aining:


T u to r

13 : 57

qp o

Feedback Suspend End Block

Item: 40 o f 46

11 f M a rk

Q. Id : 1201 [549201]

<a
P re v io u s

E>
Next

II
Lab V a lu e s

Bp?-N o te s C a lc u la to r

2 standard deviations from the mean. Since 95% ot values must be within the 2 standard deviations (2UU-24U
mg/dL), 2.5% of the values must lie below 200 mg/dL and 2.5% above 240 mg/dl_. Since there are 400
patients in the study, 10 patients (2.5 %) will have a level above 240 mg/dL. The following slide demonstrates
the concept. The slide below demonstrates the concept.

95%
(Choice A) 2 patients is equal to 0.5% of the study population. Only 1 or 2 patients at the most would be
expected to have a level of over 250 mg/dL (3 standard deviations).
(Choice C) 20 patients total (5%) would be expected to fall outside of 2 standard deviations from the mean.
However, half of these will be lower than two standard deviations (<200 mg/dL) and half will be greater than
two standard deviations from the mean (240 mg/dL).
(Choice D) 64 patients (16 % of the study population) will have a value that is one standard deviation above
the mean, but the cutoff value is 2 standard deviations above the mean.

(Choice E) 128 patients (32%) will have a value which lies greater than 1 standard deviation away from the
mean, although half of these will be lower and half will be higher than one standard deviation from the mean.
Educational objective:
In a normal (bell-shaped) distribution:
68% - within 1 standard deviation from the mean
95% - within 2 standard deviations from the mean
99.7% - within 3 standard deviations from the mean
Time Spent: 15 seconds Copyright USMLEWorld,LLC. Last updated: [3/6/2014]

Block Tim e Rem aining:


T u to r

14 : 01

qp o

Feedback Suspend End Block

Item: 41 o f 46

11 f M a rk

Q. Id : 1187 [549201]

<a
P re v io u s

E>
Next

II
Lab V a lu e s

Bp*-
N o te s C a lc u la to r

In a large population with little migration, the incidence of diabetes mellitus, type II has been stable at 3 cases
per 1,000 per year for 30 years. The prevalence of this disease increased progressively over the same
period. Which of the following is the most likely explanation of this trend?
O A. High mortality in diabetics
O B. Selective survival
O C. Improved quality of care
O D. Decreased hospitalization rate
O E. Increased diagnostic accuracy

Block Tim e Rem aining:


T u to r
A

14 : 05

Feedback Suspend End Block

Item: 41 of 4 6
1 f M ark < 1 O i l iRl
Q. Id: 1187 [549201] Previous Next Lab Values Notes Calculator
A

In a large population with little migration, the incidence of diabetes mellitus, type II has been stable at 3 cases
per 1,000 per year for 30 years. The prevalence of this disease increased progressively over the same
period. Which of the following is the most likely explanation of this trend?

O
O

A. High mortality in diabetics [2%]

O
O

D. Decreased hospitalization rate [1%]

B. Selective survival [4%]


* C. Improved quality of care [80%]
E. Increased diagnostic accuracy [11%]
Explanation: User Id: 549201
Incidence and prevalence are two important concepts in epidemiology. Incidence is the measure of the
appearance of new cases. Prevalence is the measure of those with the disease in the population at a
particular point in time. The relationship between these two categories can be demonstrated by the following
approximation in a stable population (little migration):
Prevalence = (Incidence) x (Time)
The above vignette describes a disease with a rising prevalence but stable incidence. Such trend can be
attributed to factors that prolong the duration of the disease (e.g., improved quality of care).
(Choice A) A high mortality rate in diabetics would result in a stable or decreased prevalence.
(Choices B and D) Selective survival and a decreased hospitalization rate are not helpful in explaining the
increased prevalence described in this scenario.
(Choice E) Increased diagnostic accuracy affects both the prevalence and incidence of a disease.

Educational Objective:
An increasing prevalence and stable incidence can be attributed to factors which prolong the duration of a
disease (e g., improved quality of care - this scenario is typical for the USMLE).

Block Tim e Rem aining:


T u to r

14 : 14

Feedback

Suspend EndS ock

Item: 42 o f 46

11 f M a rk

Q. Id : 1282 [549201]

<a
P re v io u s

E>
Next

II
Lab V a lu e s

Bp*-
N o te s C a lc u la to r

Blood glucose levels of 800 patients with acute pancreatitis are determined to have a strongly positively
skewed distribution. Which of the following is most consistent with this finding?
O A. Mean is greater than the median
O B. Mean is equal to the mode
O C. Mean is equal to the median
O D. Median is greater than the mean
O E. Mode is greater than the mean

Block Tim e Rem aining:


T u to r

14 : 31

Feedback Suspend End Block

Item: 42 o f 46

11 f M a rk

Q. Id : 1282 [549201]

<a
P re v io u s

E>
Next

II
Lab V a lu e s

Bp?-N o te s C a lc u la to r

Blood glucose levels of 800 patients with acute pancreatitis are determined to have a strongly positively
skewed distribution. Which of the following is most consistent with this finding?
* <) A. Mean is greater than the median [68%]

O
O
O
O

B. Mean is equal to the mode [2%]


C. Mean is equal to the median [4%]
D. Median is greater than the mean [18%]

E. Mode is greater than the mean [8%]


Explanation: User Id: 549201
Although the symmetrical bell-shaped curve is cited as the prototypical distribution curve, most datasets
generated during "real world" statistical analysis have asymmetrical distributions. When a distribution curve is
asymmetric, it is either positively or negatively skewed. With a positive skew, smaller numbers predominate
in the dataset and the long slope of the curve (the "tail") extends in the positive direction. With a negative
skew, larger numbers predominate in the dataset and the long slope of the curve (the "tail") extends in the
negative direction. In a positively skewed distribution, the mean is the most shifted in the positive direction,
followed by the median and then the mode. In similar examples of strongly skewed distributions, the median
is a better measure of central tendency than is the mean.

(Choices B and C) In normal distribution curves where there is no skew, all three measures of central
tendency are precisely equal: mean = median = mode. If a minor skew is present, the three measures are
approximately equal.
(Choices D and E) In a negatively skewed distribution (with "the tail" on the left), the mean is the most shifted
in the negative direction, followed by the median and then the mode.
Educational Objective:
In a positively skewed distribution, the mean is the most shifted in the positive direction, followed by the
median and then the mode.
Time Spent: 39 seconds Copyright USMLEWorld,LLC. Last updated: [8/22/2014]

Block Tim e Rem aining:


T u to r

14 : 39

Feedback Suspend End Block

Item: 43 o f 46

11 f M a rk

Q. Id : 1203 [549201]

<a
P re v io u s

E>
Next

II
Lab V a lu e s

Bp*-
N o te s C a lc u la to r

400 women aged 20-35 coming for routine check-up are asked about their smoking status. 40% of the
women are smokers. Over the next ten years, 25 smokers and 24 non-smokers developed breast cancer.
Which of the following best describes the study design?

O
O
O
O
O

A. Prospective cohort study


B. Retrospective cohort study
C. Case-control study
D. Cross-sectional study
E. Randomized clinical trial

Block Tim e Rem aining:


T u to r

14 : 45

Feedback Suspend End Block

Item: 43 of 46

11 f M a rk

Q. Id : 1203 [549201]

<a
P re v io u s

E>
Next

II
Lab V a lu e s

Bp?-N o te s C a lc u la to r

400 women aged 20-35 coming for routine check-up are asked about their smoking status. 40% of the
women are smokers. Over the next ten years, 25 smokers and 24 non-smokers developed breast cancer.
Which of the following best describes the study design?
* A. Prospective cohort study [83%]

O
O
O

B. Retrospective cohort study [5%]


C. Case-control study [7%]
D. Cross-sectional study [4%]

E. Randomized clinical trial [1%]


Explanation: User Id: 549201
The scenario described above is a good example of prospective cohort study. Initially a group of subjects is
selected (i.e., cohort) and their exposure status is determined (smoker/non-smoker). The cohort is then
followed for a certain period of time and observed for development of the outcome (breast cancer).
Sometimes, the exposure status is determined retrospectively and then patients are tracked from that point of
time, typically using medical records (Choice B).
(Choice C) A case-control study is designed by selecting patients with a particular disease (cases) and
without that disease (controls) and then determining their previous exposure status.
(Choice D) A cross-sectional study is also known as a prevalence study. It is characterized by the
simultaneous measurement of exposure and outcome. It is a snapshot study design that frequently uses
surveys. They are relatively inexpensive and easy to perform.
(Choice E) A randomized clinical trial directly compares two or more treatments. Usually, the subjects are
randomly assigned to an exposure (e g., a medication) or placebo and then followed for the development of
the outcome of interest.
Educational Objective:
Prospective cohort studies are organized by selecting a group of individuals (i.e., cohort), determining their
exposure status, and then following them over time for development of the disease of interest.

Block Tim e Rem aining:


T u to r

14 : 55

Feedback Suspend End Block

Item: 44 of 46

11 f M ark

Q. Id : 1300 [549201] Previous< a NextE > Lab ValuesI

I NotesBp*- Calculator
A rare metabolic disease has severe consequences if left untreated. If detected early after birth the disease
can be treated effectively and all the severe sequelae can be prevented. It is most important for a test to be
used in all newborns to have high:
O A. Sensitivity
O B. Specificity
O C. Positive predictive value
O D. Cutoff value
O E. True negatives
Block Tim e Rem aining:
T u to r

15 : 00

Feedback Suspend End Block

Item: 44 of 46 11 ^Mark
Q. Id : 1300 [549201] Previous< a NextE >

A rare metabolic disease has severe consequences if left untreated. If detected early after birth the disease
can be treated effectively and all the severe sequelae can be prevented. It is most important for a test to be
used in all newborns to have high:
Lab Values Notes Calculator
A

* A. Sensitivity [76%]

O
O
O
O

B. Specificity [12%]
C. Positive predictive value [10%]
D. Cutoff value [1%]

E. True negatives [1%]


Explanation: User Id: 549201
The sensitivity of a test refers to the number of true positives divided by the number of patients with the
disease. Sensitivity represents the ability of a test to rule out those with the disease. The sensitivity of a test
is very important in screening situations like the scenario described above. A test with high sensitivity is one
which identifies most patients with the disease. This means that most sick patients will have a positive test
result. A test that identifies most ill patients (high sensitivity) is essential in scenarios such as ours for two
reasons: 1) the metabolic disease has severe consequences if left untreated; 2) if detected early, the disease
is treatable and the consequences are preventable.
(Choices B and E) Specificity represents the ability of a test to exclude those without the disease. A very
specific test is one which has a low rate of false positives. This means that most healthy patients will have a
negative test result. It is important for a confirmatory test to have a high specificity.
(Choice C) Positive predictive value represents a test's ability to correctly identify those with the disease from
all those who had positive results. Positive predictive value depends on the prevalence of the disease.
(Choice D) Setting a high cutoff value typically produces higher specificity and lower sensitivity.

Educational Objective:
It is important to have high sensitivity in screening tests.
T im e Snent 11 seconds O onvrinht USMI FW nrld I I C 1 ast undated- m /??/?014l V

Block Tim e Rem aining:


T u to r

15 : 07

Feedback Suspend EndQ ock

Item: 45 of 46

11 f M ark

Q. Id : 1286 [549201] Previous< a NextE > Lab ValuesI

I NotesBp*- Calculator
The 70-year mortality trends for five cancers in women in the USA are given in the graph below.
1930 1965 2000 Time
Which of the following curves best corresponds to lung cancer?

o
o

>>
B. B

0p

o
o

dQ
mm

Block Tim e Rem aining:


T u to r

15 : 11

Feedback Suspend End Block

Item: 45 of 46

11 f M ark

Q. Id : 1286 [549201] Previous< a NextE > Lab ValuesI

I NotesBp*- Calculator
The 70-year mortality trends for five cancers in women in the USA are given in the graph below.
1930 1965 2000 Time
Which of the following curves best corresponds to lung cancer?

>>o

'P
O'

B. B [7%]
0 C. C [2%]
* ()
O'
OO
t"Qd

E. E [3%]
Explanation: User Id: 549201
Once uncommon, lung cancer has emerged as the leading cause of cancer mortality in women. Increasing
incidence and mortality from lung cancer in women has been observed over the last four decades due to
increased cigarette smoking.

Block Tim e Rem aining:


T u to r

15 : 20

Feedback Suspend End Block

Item: 4 5 of 4 6

11 f M a rk

Q. Id : 1286 [549201]

<a
P re v io u s

O
N e x t Lab V a lu e s N o te s C a lc u la to r

1930 1965 2000 Time


Which of the following curves best corresponds to lung cancer?

o
o
0s
rm

A. A [10%]
B.

c . C [2%]

* () D. D
r~^J
00
-o
o'

E. E [3%]
Explanation: User Id: 549201
Once uncommon, lung cancer has emerged as the leading cause of cancer mortality in women. Increasing
incidence and mortality from lung cancer in women has been observed over the last four decades due to
increased cigarette smoking.
(Choices A and B) Breast cancer is the most common non-skin cancer among women in the US but breast
cancer mortality is comparatively low. Mortality from breast cancer has stayed relatively stable over time
whereas colon cancer mortality decreased somewhat over the last decades.
(Choice C) Once very common, stomach cancer is now uncommon. Its incidence and mortality decreased
drastically over the first half of the twentieth century.
(Choice E) The mortality trend for ovarian cancer is demonstrated by the green curve.
Educational Objective:
An increase in lung cancer incidence and mortality has been observed in women over the last four decades.
Time Spent: 18 seconds Copyright USMLEWorld,LLC. Last updated: [8/22/2014]

Block Tim e Rem aining:


T u to r

15 : 24

Feedback Suspend End Block

Item: 4 6 of 4 6

11 f M a rk

Q. Id : 1175 [549201]

<>
P re v io u s N e x t
| Next
Lab V a lu e s N o te s C a lc u la to r

A study of drug A versus standard therapy in preventing recurrent pulmonary embolism has just been
released. The absolute risk reduction for drug A versus standard therapy is 4%. If the incidence of pulmonary
embolism with standard therapy is 6% and there are 24 pulmonary emboli in the drug A group, how many total
subjects are there in the drug A group?

o
o
o
o
o
o

A. 2100
B .1800
C .1500
D .1200
E. 900
F. 600

Block Tim e Rem aining:


T u to r

15 : 59

Feedback Suspend End Block

Item: 4 6 of 4 6

11 f M a rk

Q. Id : 1175 [549201]

<a
P re v io u s

E>
Next

II
Lab V a lu e s

Bp*-
N o te s C a lc u la to r

A study of drug A versus standard therapy in preventing recurrent pulmonary embolism has just been
released. The absolute risk reduction for drug A versus standard therapy is 4%. If the incidence of pulmonary
embolism with standard therapy is 6% and there are 24 pulmonary emboli in the drug A group, how many total
subjects are there in the drug A group?
Explanation: User Id: 549201

This question is meant to challenge your knowledge of how absolute risk reductions (ARR) are calculated.
The ARR equals the control event rate [Event Rate^contro|^]minus treatment event rate (Event Rate^treatmentj].
Using our case we can calculate the event rate in the treatment arm.
ARR = Event Rate(con(ro|) - Event Rate(treatment)
4% = 6% - Event Rate(trea(ment)
EventRate(treatment) = 2%
Since the Event Rate(treatment) ecluals tota* number of events divided by the number of subjects in the
treatment arm, we can determine the total number of subject in the treatment arm.
Event Rate(treatment) Events/Total treatment arm subjects
2% = 24/Total treatment arm subjects
Total treatment arm subject = 1200.

Block Tim e Rem aining:


T u to r

16 : 04

Feedback Suspend End Block

Item: 4 6 of 4 6
1 f M a rk < 1 O i l m
Q. Id : 1175 [549201] P re v io u s N e x t Lab V a lu e s N o te s C a lc u la to r

embolism with standard therapy is 6% and there are 24 pulmonary emboli in the drug A group, how many total
subjects are there in the drug A group?

A. 2100 [4%]

B. 1800 [7%]

o c.

1500 [9%]

() D. 1200 [55%] o E. 900 [6%] o F. 600 [19%]


Explanation: User Id: 549201
This question is meant to challenge your knowledge of how absolute risk reductions (ARR) are calculated.
The ARR equals the control event rate [Event Rate^contro^minus treatment event rate (Event
Rate(treatment>lUsing our case we can calculate the event rate in the treatment arm.
ARR = Event Rate(con(ro|) - Event Rate(treatment)
4% = 6% -Event Rate(trea(ment)
Event Rate(treatment) = 2%
Since the Event Rate(treatment) eclua,s the total number of events divided by the number of subjects in the
treatment arm, we can determine the total number of subject in the treatment arm.
Event Rate(treatment) Events/Total treatment arm subjects
2% = 24/Total treatment arm subjects
Total treatment arm subject = 1200.
Educational Objective:
ARR = Event Rate^controj^ - Event Ratetreatment
Time Spent: 9 seconds Copyright USMLEWorld,LLC. Last updated: [8/22/2014]

Block Tim e Rem aining:

16 : 07

Feedback Suspend End Block


23456789
10
T u to r

11
12
13
14
15
16

Item: 1 of 16
Q. Id : 1177 [549201]

I F^Mark

P re v io u s

<a

Lab V a lu e s

The results of study investigating a new diagnostic test in diagnosing acute myocardial infarction are given in
the table below.
Test positive
Test negative
Myocardial
infarction
75
25
No myocardial
infarction

20
80

What is the sensitivity of the new diagnostic test?

o
o
o
o
o
o

A. 25%
B. 37%
C. 55%
D. 70%
E. 75%
F. 80%

Block Tim e Rem aining:

OO : 0 2

Tutor
1

2 11 f M ark
3
4
5

6
7
8
9
10

11
12

13
14
15
16

Item: 1 of 16
Q. Id: 1177 [549201] Previous< a NextE >

The results of study investigating a new diagnostic test in diagnosing acute myocardial infarction are given in
the table below.
Myocardial
infarction
Test positive 75
Test negative 25
No myocardial
infarction

20
80
What is the sensitivity of the new diagnostic test?
Lab Values Notes Calculator
A

Explanation: User Id: 549201


The sensitivity of a test refers to the number of true positives divided by the total number of patients with the
disease. Sensitivity should be high in screening tests in order to pick up all cases of a disease (decrease
false negatives). Using the generic two-by-two square below, sensitivity is represented as a/(a+c)
Disease positive Disease negative
Test positive a
True positive
b
False positive
Test negative c
False negative
d
True negative
This test has a sensitivity of 75/(75+25), or 75%. The false negative ratio is 1-sensitivity. It is not affected by
disease prevalence.
V

Block Tim e Rem aining: oo : 06 I


T u to r

Feedback Suspend End Block

Item: 1 of 16

2 Q. Id : 1177 [549201]
34

F^Mark Previous<

NextE

>

Lab Valuesi

NotesW&-' CalculatorH

Myocardial No myocardial A
5 infarction infarction
6 Test positive 75 20
7 Test negative 25 80
9 What is the sensitivity of the new diagnostic test?
Explanation: User Id: 549201
The sensitivity of a test refers to the number of true positives divided by the total number of patients with the
disease. Sensitivity should be high in screening tests in order to pick up all cases of a disease (decrease
false negatives). Using the generic two-by-two square below, sensitivity is represented as a/(a+c)
Disease positive Disease negative
Test positive a
True positive
b
False positive
Test negative c
False negative
d
True negative
This test has a sensitivity of 75/(75+25), or 75%. The false negative ratio is 1-sensitivity. It is not affected by
disease prevalence.
Educational Objective:
Sensitivity equals a/(a+c). Screening tests are designed to have high sensitivities.
Time Spent: 10 seconds Copyright USMLEWorld,LLC. Last updated: [8/22/2014]

Block Tim e Rem aining: oo :


T u to r

09

qp

Feedback Suspend End Block

12

3
4
5

6
7

8
9
10

11
12

13
14
15
16

Item: 2 of 16 D-M ark O O I I


Q. Id : 1279 [549201] Previous Next Lab Values

A new estrogen receptor agonist is being evaluated for the treatment of postmenopausal symptoms. A
prospective study shows that the drug increases the risk of deep vein thrombosis (DVT) in treated women
who smoke compared to untreated women who smoke, with a relative risk (RR) of 1.70 and P value of .01. In
nonsmokers, no increased risk of DVT is evident with use of the drug (RR=0.96; P=.68). Which of the
following describes this phenomenon?

O
O
O

A. Confounding [51 %]
B. Selection bias [10%]

C. Latent period [4%]


* () D. Effect modification [32%]

E. Observer bias [3%]


Explanation: User Id: 549201
Effect modification occurs when the effect of a main exposure on an outcome is modified by another variable.
In this scenario, the effect of the new estrogen receptor agonist on the incidence of DVT is modified by
smoking status. Smokers taking the medication have an increased risk of developing DVT, while nonsmokers
taking the medication do not. There are other well-known examples of effect modification, such as the

elihood that asbestos exposure will result in lung cancer (a phenomenon


significantly impacted by smoking
status).
(Choices A, B, and E) Effect modification is not a bias. It is not due to flaws in the design or analysis phases
of the study. When such flaws do occur, they can result in confounding, selection bias, or observer bias.
Effect modification is a natural phenomenon that should be described, not corrected.
It is most easily confused with confounding, because both can be identified by dividing the whole cohort into
subgroups (stratified analysis). Imagine that smoking is a confounding factor that, by itself, is associated with
a higher risk of DVT. If more smokers are taking the study drug, it might appear that the drug causes DVT
(e.g., crude RR=1.5; P<_05). But when stratified analysis is performed by analyzing smokers and
nonsmokers separately, the drug is no longer associated with increased risk of DVT:
Smokers: RR=1.01; P is not significant
Nonsmokers: RR=1.01; P is not significant

Block Tim e Rem aining: oo : 19


T u to r

Feedback
Notes
Suspend
Calculator
A

End Block
1 ____ n -4 /> a f

_____
Item: 2 of 16
-5

Q. Id : 1279 [549201] 1 f'M ark Prpuiniic< J o * 1 Vali l


456789
10

11
12
13
14
15
16

O E. Observer bias [3%]


Explanation: User Id: 549201
Effect modification occurs when the effect of a main exposure on an outcome is modified by another variable.
In this scenario, the effect of the new estrogen receptor agonist on the incidence of DVT is modified by
smoking status. Smokers taking the medication have an increased risk of developing DVT, while nonsmokers
taking the medication do not. There are other well-known examples of effect modification, such as the
likelihood that asbestos exposure will result in lung cancer (a phenomenon significantly impacted by smoking
status).
(Choices A, B, and E) Effect modification is not a bias. It is not due to flaws in the design or analysis phases
of the study. When such flaws do occur, they can result in confounding, selection bias, or observer bias.
Effect modification is a natural phenomenon that should be described, not corrected.
It is most easily confused with confounding, because both can be identified by dividing the whole cohort into
subgroups (stratified analysis). Imagine that smoking is a confounding factor that, by itself, is associated with
a higher risk of DVT. If more smokers are taking the study drug, it might appear that the drug causes DVT
(e.g., crude RR=1.5; P<.05). But when stratified analysis is performed by analyzing smokers and
nonsmokers separately, the drug is no longer associated with increased risk of DVT:
Smokers: RR=1.01; P is not significant
Nonsmokers: RR=1.01; P is not significant
In this estrogen receptor agonist study, stratified analysis shows that smokers are more susceptible to the
drug causing DVT compared to nonsmokers. But it is not smoking itself causing DVT because you compared
smokers taking the drug to smokers not taking the drug.
(Choice C) The latent period is the time required for an exposure to begin having an effect. There is no
information on how latency was handled in this study.
Educational objective:
Effect modification is present when the effect of the main exposure on the outcome is modified by the
presence of another variable. Effect modification is not a bias.
Time Spent: 13 seconds

Block Tim e Rem aining:


Copyright USMLEWorld,LLC. Last updated: [3/14/2014]
OO : 2 2
Tutor

11 f M a rk
16789

10

11
12

13
14
15
16

Item: 3 of 16
Q. Id : 1276 [549201] Previous< a NextE > Lab Values

II

Researchers are studying the relationship between essential hypertension and a common mutation in the
structure of a sodium channel protein. A study population is randomly selected. Blood samples are obtained
for leukocyte genotyping, and arterial blood pressure is measured using ambulatory blood pressure
monitoring. The researchers concluded that the mutation in the structure of sodium channel protein is
associated with hypertension. Which of the following choices identifies the study design used by the
investigators?

O
O
O

A. Prospective cohort study [13%]


B. Retrospective cohort study [7%]

C. Case-control study [11%]


* () D. Cross-sectional study [55%]

E. Randomized clinical trial [14%]


Explanation: User Id: 549201
A cross-sectional study (also known as a prevalence study) simultaneously measures exposure and
outcome. The cross-sectional study has a "snapshot" design that is frequently used in surveys, mostly
because it is inexpensive and easy to perform. The major limitation of this study design is that a temporal
relationship between exposure and outcome is not always clear. In this case, demonstrating a temporal
relationship was straightforward because the possession of a specific genotype clearly precedes
hypertension.
(Choices A and B) Prospective and retrospective cohort studies are organized by selecting a group of
individuals (ie, a cohort), determining their exposure status, and then following them forward in time or from a
point in the past to the present for development of the disease of interest.
(Choice C) A case-control study is designed by selecting individuals with a particular disease (cases),
individuals without that disease (controls), and then evaluating previous exposure status.
(Choice E) A randomized clinical trial directly compares two or more treatments. Usually, the subjects are
randomly assigned to experience a specific exposure (eg, a medication) or no exposure (eg, placebo), and
are then followed for the outcome of interest (eg, disease).

Block Tim e Rem aining:

OO : 29

T u to r

Feedback
Notes
Suspend
Calculator
A

End B lo ck
11 f M a rk

12
B456789
10

11
12
13
14
15
16

Item: 3 of 16
Q. Id : 1276 [549201]

monnoring. i ne researcners concluded inai ine mutation in tne structure ot sooium cnannei protein is
associated with hypertension. Which of the following choices identifies the study design used by the
investigators?
Lab V a lu e s

O
O
O

A. Prospective cohort study [13%]


B. Retrospective cohort study [7%]
C. Case-control study [11 %]

^ () D. Cross-sectional study [55%]

E. Randomized clinical trial [14%]


Explanation: User Id: 549201
A cross-sectional study (also known as a prevalence study) simultaneously measures exposure and
outcome. The cross-sectional study has a "snapshot" design that is frequently used in surveys, mostly
because it is inexpensive and easy to perform. The major limitation of this study design is that a temporal
relationship between exposure and outcome is not always clear. In this case, demonstrating a temporal
relationship was straightforward because the possession of a specific genotype clearly precedes
hypertension.
(Choices A and B) Prospective and retrospective cohort studies are organized by selecting a group of
individuals (ie, a cohort), determining their exposure status, and then following them forward in time or from a
point in the past to the present for development of the disease of interest.
(Choice C) A case-control study is designed by selecting individuals with a particular disease (cases),
individuals without that disease (controls), and then evaluating previous exposure status.
(Choice E) A randomized clinical trial directly compares two or more treatments. Usually, the subjects are
randomly assigned to experience a specific exposure (eg, a medication) or no exposure (eg, placebo), and
are then followed for the outcome of interest (eg, disease).
Educational Objective:
In a cross-sectional study, exposure and outcome are measured simultaneously at a particular point of time
("snapshot study"). In other study designs, a certain time period separates the exposure from the outcome.
Time Spent: 7 seconds Copyright USMLEWorld,LLC. Last updated: [8/22/2014]

Block Tim e Rem aining: oo : 32


Feedback Suspend End B lo ck
2 11 f M a rk
3
5
6
7
8
9
10
11
12
13
14
15
16

Item: 4 of 16
Q. Id : 1184 [549201]

<a
P re v io u s Lab V a lu e s

A study was conducted to assess the association between simvastatin therapy and serum fibrinogen levels in
patients who undergo percutaneous coronary interventions. Serum fibrinogen levels are recorded as either
high or normal based on a specific cutoff. The number of patients matching each classification criteria is
given in the table below:
Fibrinogen high Fibrinogen normal
Simvastatin therapy 43 67 110
No simvastatin therapy 32 58 90
75 125 200
Which of the following is the best statistical method for assessing the results of this study?

O
O
O
O
O

A. Two-sample t test
B. Two-sample z test
C. Chi-square test
D. Analysis of variance
E. Meta-analysis

Block Tim e Rem aining:

OO : 3 5

Tutor

Item: 4 of 16

11 f M ark
Q. Id: 1184 [549201] Previous< a NextE >

A study was conducted to assess the association between simvastatin therapy and serum fibrinogen levels in
patients who undergo percutaneous coronary interventions. Serum fibrinogen levels are recorded as either
high or normal based on a specific cutoff. The number of patients matching each classification criteria is
given in the table below:
Fibrinogen high Fibrinogen normal
Simvastatin therapy 43 67 110

No simvastatin therapy 32 58 90
75 125 200
Lab Values Notes Calculator
A

Which of the following is the best statistical method for assessing the results of this study?

O
O

A. Two-sample t test [31 %]

O
O

D. Analysis of variance [6%]

B. Two-sample z test [4%]


* () C. Chi-square test [56%]
E. Meta-analysis [3%]
Explanation: User Id: 549201
The chi-square test for independence is used to test the association between 2 categorical variables. In this
example, patients are divided into 2 groups (simvastatin therapy or no simvastatin therapy) and the number of
patients in each group that experience the outcome (high or normal serum fibrinogen levels) is recorded in a 2
x 2 table. Then a chi-square analysis is performed to see if there is a statistical association between
simvastatin status and fibrinogen status. If there is a large difference in outcome proportions between the 2
groups, the null hypothesis is rejected and an association between the exposure and the outcome is assumed
to be present.
(Choices A and B) The two-sample z test and two-sample t test are used to compare 2 group means, not
categorical variables. The two-sample t test would be appropriate in this example if fibrinogen levels were
recorded as numerical values (ie, in mg/dL) instead of categorically. v

Block Tim e Rem aining: oo : 41


T u to r

Feedback Suspend End Block

Item: 4 of 16

11 f M ark

Q. Id: 1184 [549201] Previous< a NextE > Lab ValuesI

vvmcn

ot

me Toiiowing is ine oest statistical metnoa tor assessing tne results or tnis stuay

O
O

A. Two-sample t test [31 %]

O
O

D. Analysis of variance [6%]

B. Two-sample z test [4%]


* <) C. Chi-square test [56%]
E. Meta-analysis [3%]
Explanation: User Id: 549201
The chi-square test for independence is used to test the association between 2 categorical variables. In this
example, patients are divided into 2 groups (simvastatin therapy or no simvastatin therapy) and the number of
patients in each group that experience the outcome (high or normal serum fibrinogen levels) is recorded in a 2
x 2 table. Then a chi-square analysis is performed to see if there is a statistical association between
simvastatin status and fibrinogen status. If there is a large difference in outcome proportions between the 2
groups, the null hypothesis is rejected and an association between the exposure and the outcome is assumed
to be present.
(Choices A and B) The two-sample z test and two-sample t test are used to compare 2 group means, not
categorical variables. The two-sample t test would be appropriate in this example if fibrinogen levels were
recorded as numerical values (ie, in mg/dL) instead of categorically.
(Choice D) Analysis of variance (ANOVA) is used to compare the means of 2 or more groups. For example,
an ANOVA analysis could be used to assess for differences in mean blood pressure among 3 sample
populations grouped by exercise status (eg, never exercise, exercise occasionally, or exercise frequently).
(Choice E) Meta-analysis is an epidemiologic method of pooling the data from several studies to conduct an
analysis having a relatively larger statistical power than that of the individual studies.
Educational objective:
The chi-square test for independence is used to test the association between 2 categorical variables. In the
case of an exposure status and a binomial outcome, patients are divided into 2 groups based on exposure,
and the number of patients that experience each outcome is recorded in a 2 x 2 table.
Time Spent: 12 seconds Copyright USMLEWorld,LLC.

Block Tim e Rem aining: oo : 44


T u to r

Last updated: [4/12/2014]


Feedback
Notes
Suspend
Calculator
A

End Block

Item: 5 of 16

11 f M a rk
Q. Id : 1303 [549201] Lab V a lu e s

67 89
10

11
12
13
14
15
16

A study of postmenopausal women shows that the relative risk (RR) of myocardial infarction in patients taking
hormone replacement therapy (HRT) is 1.32 (p = 0.03) compared to women who do not receive HRT.
Researchers conducting another study believe that there is no harmful effect of HRT since the RR is 1.30 and
p = 0.07. Which of the following seems to be a problem in the second study?
O A. Placebo effect
O B. Poor blinding
O C. Researcher expectancy
O D. Sample size
O E. Berkson's bias
Block Tim e Rem aining: oo : 47
Feedback Suspend End B lo ck
11 f Mark
12389
10
11

12

13
14
15
16

Item: 5 of 16
Q. Id : 1303 [549201] Previous< a NextE > Lab ValuesI

A study of postmenopausal women shows that the relative risk (RR) of myocardial infarction in patients taking
hormone replacement therapy (HRT) is 1.32 (p = 0.03) compared to women who do not receive HRT.
Researchers conducting another study believe that there is no harmful effect of HRT since the RR is 1.30 and
p = 0.07. Which of the following seems to be a problem in the second study?

O
O
O

A. Placebo effect [3%]


B. Poor blinding [5%]

C. Researcher expectancy [30%]


* D. Sample size [48%]

E. Berkson's bias [14%]


Explanation: User Id: 549201
The information presented in this scenario suggests that the two studies had similar results in terms of RR
but reached disparate conclusions due to different p values. The p value in the first study reached statistical
significance (p < 0.05) whereas it did not in the second study. The most likely explanation is a difference in
the sample size between the studies. The small sample size in the second study made it underpowered to
detect a difference in outcome between HRT treated and untreated patients. Statistically, the power of a study
refers to its ability to detect a difference between groups when a difference truly exists. Since the RRs in both
studies are similar it is unlikely that design flaws (e.g. poor blinding, researcher expectancy) are present in the
second study (Choices B and C)
(Choice A) Placebo effect refers to patient's expectations affecting an outcome.
(Choice E) Berkson's bias refers to selection bias that can be created by selecting hospitalized patients as
the control group.
Educational Objective:
The power of a study increases proportionally with the sample size. Thus, the larger a sample the greater the
ability to detect a difference when one truly exists.
Time Spent: 9 seconds Copyright USMLEWorld,LLC.
Block Tim e Rem aining: oo : 52
T u to r

Last updated: [8/22/2014]


Feedback
Notes
Suspend
Calculator
End Block

11 f M a rk

12345
789
10

11
12
13
14
15
16

Item: 6 of 16
Q. Id : 1278 [549201]

A 34-year-old male is admitted to the hospital with acute chest pain. An ECG obtained in the ER shows ST
segment elevation in leads II, III and avF. A sample of blood is taken from the patient and the plasma
homocysteine level is measured three times using a new test. The results are 12.2 pmol/L, 13.5 pmol/L and
15.0 pmol/L. These results suggest that concerns should be raised about which of the following with the new
test?
Lab V a lu e s

O
O
O
O
O

A. Reliability
B. Validity
C. Accuracy
D. Sensitivity
E. Specificity

Block Tim e Rem aining: oo : 54


Feedback Suspend End B lo ck

11 ^Mark
123456
7

8
9
10

11
12

13
14
15
16

Item: 6 of 16
Q. Id : 1278 [549201] Previous< a NextE > Lab ValuesI

A 34-year-old male is admitted to the hospital with acute chest pain. An ECG obtained in the ER shows ST
segment elevation in leads II, III and avF. A sample of blood is taken from the patient and the plasma
homocysteine level is measured three times using a new test. The results are 12.2 pmol/L, 13.5 pmol/L and
15.0 pmol/L. These results suggest that concerns should be raised about which of the following with the new
test?
* A. Reliability [62%]
O B. Validity [5%]
O C. Accuracy [28%]
O D. Sensitivity [3%]
O E. Specificity [1 %]
Explanation: User Id: 549201
This example deals with test-retest reliability. A reliable test is reproducible; it gives similar or very close
results on repeat measurements. In the clinical laboratory, reliability is typically quantified in terms of the
coefficient of variation (CV). The CV is the standard deviation of the set of repeated measurements divided by
their mean, and is generally expressed as a percentage. The CV is useful in the clinical laboratory setting
because it can be used to compare precision between tests that have different normal values and/or different
units.
In this example, repeat measurements of the same sample yielded markedly different results; therefore, the
new test is not very reliable. The coefficient of variation should be calculated for this test and compared to the
coefficient of variation of other available tests that measure plasma homocysteine.
(Choices B and C) Validity (accuracy) is defined as the test's ability to measure what it is supposed to
measure. To evaluate the validity of a new test, its results should be compared to those obtained with the
"gold standard" test on the same individual. In this case, since there were no test results obtained using the
"gold standard," the validity or accuracy of the test cannot be determined. Note that a test can be highly
reliable but invalid.
(Choices D and E) Sensitivity is defined as a test's ability to identify the true presence of disease, whereas

<>nprifiritv is: Hpfinprl ps: p tout's; philitv to irlpntifv thp trnp a h ^ p n r p of H i^ p o ^ p T h p rpliphilitv o f p fp<>t ______

Block Tim e Rem aining: 0 1 : oo


Feedback
Notes
Suspend
Calculator
T u to r

End Block
11 f Mark
123456
7

8
9
10

11
12

13
14
15
16

Item: 6 of 16
Q. Id : 1278 [549201]

* A. Reliability [62%]
O B. Validity [5%]
O C. Accuracy [28%]
O D. Sensitivity [3%]
O E. Specificity [1%]
<>Previous Next Lab Valuesi

Explanation: User Id: 549201


This example deals with test-retest reliability. A reliable test is reproducible; it gives similar or very close
results on repeat measurements. In the clinical laboratory, reliability is typically quantified in terms of the
coefficient of variation (CV). The CV is the standard deviation of the set of repeated measurements divided by
their mean, and is generally expressed as a percentage. The CV is useful in the clinical laboratory setting
because it can be used to compare precision between tests that have different normal values and/or different
units.
In this example, repeat measurements of the same sample yielded markedly different results; therefore, the
new test is not very reliable. The coefficient of variation should be calculated for this test and compared to the
coefficient of variation of other available tests that measure plasma homocysteine.
(Choices B and C) Validity (accuracy) is defined as the test's ability to measure what it is supposed to
measure. To evaluate the validity of a new test, its results should be compared to those obtained with the
"gold standard" test on the same individual. In this case, since there were no test results obtained using the
"gold standard," the validity or accuracy of the test cannot be determined. Note that a test can be highly
reliable but invalid.
(Choices D and E) Sensitivity is defined as a test's ability to identify the true presence of disease, whereas
specificity is defined as a test's ability to identify the true absence of disease. The reliability of a test is
unrelated to its sensitivity or specificity.
Educational Objective:
A reliable test is reproducible in that it gives similar results on repeat measurements. Reliability is maximal
when random error is minimal.
Time Spent: 8 seconds Copyright USMLEWorld,LLC. Last updated: [8/22/2014]

Block Tim e Rem aining: o i : 04


Feedback
Notes
Suspend
Calculator
T u to r

End Block
11 f M a rk

12345689
10

11
12
13
14
15
16

Item: 7 of 16

Q. Id : 1205 [549201]

A study is conducted to assess the effect of beta-blocker therapy in patients with acute myocardial infarction
(Ml). The study found that 20 patients out of 90 who took a beta-blocker during the week prior to a Ml
developed a major arrhythmia during hospitalization. The study also determined that 30 patients out of 70 who
did not take any beta-blocker developed a major arrhythmia. What is the odds ratio for developing major
arrhythmia in patients taking beta-blockers versus patients not taking beta-blockers?
Lab V a lu e s

O A. (20x40)/(70x3Q)

O
O
O
O

B. (20x70)/(30x40)
C. (20x30)/(70x40)
D. (20/50)/(70/110)
E. (20/90)/(30/70)

Block Tim e Rem aining: o i : 08


Feedback Suspend End B lo ck
11 f M a rk

12345689
10

11
12
13
14
15
16

Item: 7 of 16
Q. Id : 1205 [549201]

<a
P re v io u s Lab V a lu e s C a lc u la to r

A study is conducted to assess the effect of beta-blocker therapy in patients with acute myocardial infarction
(Ml). The study found that 20 patients out of 90 who took a beta-blocker during the week prior to a Ml
developed a major arrhythmia during hospitalization. The study also determined that 30 patients out of 70 who
did not take any beta-blocker developed a major arrhythmia. What is the odds ratio for developing major
arrhythmia in patients taking beta-blockers versus patients not taking beta-blockers?
* A. (20x40)/(70x3Q) [42%]

O
O
O
O

B. (20x70)/(30x40) [8%]
C. (20x30)/(70x40) [8%]
D. (20/50)/(70/110) [3%]

E. (20/90)/(30/70) [39%]
Explanation: User Id: 549201
It is important to know how to calculate some basic measures of effect using varying data presentations. The
first step in making these calculations is to create a 2x2 table as shown below.
Major
arrhythmia No arrhythmia
Beta-blocker 20(a) 70 (b) 90
No beta-blocker 30 (c) 40 (d) 70
50 110 160
Odds ratio is calculated using the following formula: OR = ad/bc. In our case OR = 20x40/70x30 = 0.38
(Choice E) Relative risk is calculated using the following formula: RR = [a/(a+b)j / [c/(c+d)]
Educational Objective:
Odds ratio calculation is commonly asked. Be sure you can construct 2x2 table and calculate odds ratio.
Time Spent: 8 seconds Copyright USMLEWorld,LLC. Last updated: [3/16/2014]

Block Tim e Rem aining: o i :


T u to r

Feedback Suspend End Block

11 ^Mark
12345
6
7
8
9
10

11
12

13

13

14
15
16

Item: 8 of 16
Q. Id : 1272 [549201] Previous Next < > - Lab Values Notes Calculator

Investigators are designing a randomized control trial to test the hypothesis that drug B will decrease the
mortality associated with acute ST elevation myocardial infarction compared to standard of care. In order to
ensure that they will not miss a difference between drug B and standard of care if one truly exists, they want to
maximize:
O A. Type I error
O B. Type II error
O C. Alpha (a)
O D. Beta (0)
O E. 1-0
Block Tim e Rem aining: 0 1 : 1 6 I P O
T u to r

Feedback Suspend End Block

11 ^Mark
12345
6
7
8
9
10

11
12

13
14
15
16

Item: 8 of 16
Q. Id : 1272 [549201] Previous Next < > -

Investigators are designing a randomized control trial to test the hypothesis that drug B will decrease the
mortality associated with acute ST elevation myocardial infarction compared to standard of care. In order to
ensure that they will not miss a difference between drug B and standard of care if one truly exists, they want to
maximize:
Lab Values Notes Calculator
A

O
O
O
O

A. Type I error [10%]


B. Type II error [12%]
C. Alpha (a) [8%]
D. Beta (0) [8%]

* <> E. 1-0 [62%]


Explanation: User Id: 549201
Statistical power, or 1- (3, is the probability of rejecting the null hypothesis when it is truly false. That is, it is the
probability of finding a true relationship. Power depends upon sample size and the difference in outcome
between the two groups being tested. In this study the researchers want to find a difference between drug B
and standard of care if one exists; they want to maximize power.
(Choice A) Type I error occurs when researchers reject the null hypothesis when the null hypothesis is really
true. That is, a study finds a statistically significant difference between two groups when one does not truly
exist. An example would be if a study concluded that hard candy improves heart failure mortality when, in fact,
it does not. Alpha (a) is the maximum probability of making a type I error a researcher is willing to accept. It
corresponds with the p value, or the probability of making a type I error. The a is typically set at p = 0.05,
meaning that researchers accept a 5% possibility that a difference perceived as true is actually due to chance.
(Choice D) Type II error occurs when researchers fail to reject the null hypothesis when it is truly false. It
causes investigators to miss true relationships. An example of type II error would be a study finding that
aspirin does not affect platelet function when, in fact, it does. Beta ((3) is the probability of committing a type II
error. Thus, if (3 is set at 0.2, the power (Choice E) will be 80%; there will be an 80% chance rejecting a null
hypothesis when it is truly false.
Mathematically these concepts are represented below:
V

Block Tim e Rem aining: o i : 20


Feedback Suspend End Block
11 f Mark
12345
T u to r

6
7
8
9
10

11
12

13
14
15
16

Item: 8 of 16
Q. Id : 1272 [549201] Previous Next < > - Lab Values Notes

Explanation: User Id: 549201


Statistical power, or 1- p, is the probability of rejecting the null hypothesis when it is truly false. That is, it is the
probability of finding a true relationship. Power depends upon sample size and the difference in outcome
between the two groups being tested. In this study the researchers want to find a difference between drug B
and standard of care if one exists; they want to maximize power.
(Choice A) Type I error occurs when researchers reject the null hypothesis when the null hypothesis is really
true. That is, a study finds a statistically significant difference between two groups when one does not truly
exist. An example would be if a study concluded that hard candy improves heart failure mortality when, in fact,
it does not. Alpha (a) is the maximum probability of making a type I error a researcher is willing to accept. It
corresponds with the p value, or the probability of making a type I error. The a is typically set at p = 0.05,
meaning that researchers accept a 5% possibility that a difference perceived as true is actually due to chance.
(Choice D) Type II error occurs when researchers fail to reject the null hypothesis when it is truly false. It
causes investigators to miss true relationships. An example of type II error would be a study finding that
aspirin does not affect platelet function when, in fact, it does. Beta (p) is the probability of committing a type II
error. Thus, if (3 is set at 0.2, the power (Choice E) will be 80%; there will be an 80% chance rejecting a null
hypothesis when it is truly false.
Mathematically these concepts are represented below:

Disease positive Disease negative


Test positive a
True positive
b
False positive
Test negative c
False negative
d
True negative
Type I error: b/(b+d)
Type II error: c/(a+c)
Educational Objective:
Power (1- P) is the probability of rejecting a null hypothesis when it is truly false. It is typically set at 80% and
depends upon sample size and difference between outcomes.
Time Spent: 7 seconds Copyright USMLEWorld,LLC. Last updated: [8/22/2014]

Block Tim e Rem aining: o i :


Feedback Suspend
Calculator
T u to r
A

End B lo ck
11 f M a rk

12345678
10

11
12
13
14
15
16

Item: 9 of 16
Q. Id : 1281 [549201]

<a
P re v io u s Lab V a lu e s

23

qp

A 65-year-old Caucasian male is hospitalized with chest pain and hypotension. An intra-arterial line is placed
for direct blood pressure monitoring. Consecutive readings of his systolic blood pressure are 75. 110, 90. 80.
75, and 110 mm Hg. Which of the following is the median systolic blood pressure?

Block Tim e Rem aining:

01:27

Tutor
11 f M a rk

12345678

10

11
12
13
14
15
16

Item: 9 of 16
Q. Id : 1281 [549201]

<a
P re v io u s Lab V a lu e s

A 65-year-old Caucasian male is hospitalized with chest pain and hypotension. An intra-arterial line is placed
for direct blood pressure monitoring. Consecutive readings of his systolic blood pressure are 75. 110, 90. 80.
75, and 110 mm Hg. Which of the following is the median systolic blood pressure?
00
o
0s

A.

* () B. 85 [76%] o c. 90 [16%] o D. 100 [1%] o E. 110 [2%]


Explanation: User Id: 549201
It is important to understand the difference between the median, mean, and mode, three measures of central
tendency. The median of an ordered dataset is the number that separates the right half of the data from the
left half. Note that the dataset MUST be ordered before the median is determined. If the number of
observations in the dataset is even, finding the median requires adding the middle two values together and
dividing by two (see below).

Median
75
75 80 90

110
110

Systolic Blood Pressure


Therefore, the ordered dataset in this case is (75, 75, 80, 90, 110, 110). Because there are an even number
of observations present, the median is (80+90)/2 = 85.
(Choices A and D) The values 80 and 100 are not measures of the center of the dataset.

Block Tim e Rem aining:

O l : 34

Tutor
11 f M a rk

12345678

10

11
12
13
14
15
16

Item: 9 of 16
Q. Id : 1281 [549201]

O E. 110 [2%]
Explanation: User Id: 549201
It is important to understand the difference between the median, mean, and mode, three measures of central
tendency. The median of an ordered dataset is the number that separates the right half of the data from the
left half. Note that the dataset MUST be ordered before the median is determined. If the number of
observations in the dataset is even, finding the median requires adding the middle two values together and
dividing by two (see below).

Median
75 110

75 80 90 110
Lab V a lu e s

Systolic Blood Pressure


Therefore, the ordered dataset in this case is (75, 75, 80, 90, 110, 110). Because there are an even number
of observations present, the median is (80+90)/2 = 85.
(Choices A and D) The values 80 and 100 are not measures of the center of the dataset.
(Choice C) To find the mean of a dataset, add all of the observations and divide that sum by the number of
observations. In this case, the mean is (75+75+80+90+110+110)/6 = 90.
(Choice E) Another measure of the center of a dataset is the mode. Finding the mode is easy, because the
mode is the most frequent value of a dataset. In our scenario, the dataset is "bimodal" because 75 and 110
are each listed twice.
Educational objective:
The median is the value that is located in the precise center of an ordered dataset. It divides the right half of
the data from the left half.
Time Spent: 9 seconds

Block Tim e Rem aining:


Copyright USMLEWorld,LLC. Last updated: [8/22/2014]
Feedback Suspend End B lo ck
11 f M a rk

1234567
8
9

EE
11

12
13
14
15
16

Item: 10 of 16
Q. Id : 1765 [549201]

A new portable cholesterol measuring device determines a patient's blood cholesterol level to be 200 mg/dl_
on three separate measurements of the same blood sample. Using the gold standard measurement method,
the same sample is found to have a cholesterol level of 260 mg/dl_. The new measuring device is:
Lab V a lu e s

O
O
O
O
O

A. Accurate but not precise


B. Precise but not reliable
C. Sensitive but not specific
D. Reliable but not accurate
E. Sensitive but not valid

Block Tim e Rem aining: 0 1 : 40


Feedback Suspend End B lo ck
11 f M ark
1234567

8
9
10

11
12

13
14
15
16

Item: 10 of 16
Q. Id : 1765 [549201] Previous< a NextE > Lab Values

II

A new portable cholesterol measuring device determines a patient's blood cholesterol level to be 200 mg/dl_
on three separate measurements of the same blood sample. Using the gold standard measurement method,
the same sample is found to have a cholesterol level of 260 mg/dL The new measuring device is:

O
O
O

A. Accurate but not precise [12%]


B. Precise but not reliable [21 %]

C. Sensitive but not specific [5%]


* D. Reliable but not accurate [59%]

E. Sensitive but not valid [3%]


Explanation: User Id: 549201

Reliability (or) Precision

*
Accurate
A reliable tool is one that consistently provides the same value when measuring a fixed entity. An accurate
tool is one that gives a measurement close to the actual value (as reflected in the gold standard
measurement).
(Choice A) A tool with a high level of precision produces measurements with a low level of variability when
used to assess a fixed entity. Precision and reliability are essentially equivalent terms. The measuring device
described above is precise but not accurate.

Block Tim e Rem aining: o i : 48


Feedback
Notes
Suspend
Calculator
T u to r

End Block

Item: 10 of 16

11 f M a rk

Q. Id : 1765 [549201] Lab V a lu e s

E. Sensitive but not valid [3%]


Explanation: User Id: 549201
11
12
13
14
15
16

Reliability (or} Precision Accurate


A reliable tool is one that consistently provides the same value when measuring a fixed entity. An accurate
tool is one that gives a measurement close to the actual value (as reflected in the gold standard
measurement).
(Choice A) A tool with a high level of precision produces measurements with a low level of variability when
used to assess a fixed entity. Precision and reliability are essentially equivalent terms. The measuring device
described above is precise but not accurate.
(Choice B) The measuring device described above is both precise and reliable.
(Choices C and E) Sensitivity and specificity refer to the ability of a diagnostic test to discriminate between
individuals who do and do not have a defined clinical condition. Validity is the extent to which a measurement
actually measures what it claims to measure.
Educational Objective:
The reliability of a measurement technique refers to its reproducibility. The accuracy of a measurement
technique is the degree to which the average measurement value matches that of the gold standard technique.
Time Spent: 10 seconds Copyright USMLEWorld,LLC. Last updated: [8/22/2014]

Block Tim e Rem aining: o i : 50


Feedback Suspend End B lo ck
11 f M a rk

123456789
10

ED
12
13
14
15
16

Item: 11 of 16
Q. Id : 1229 [549201]

<a
P re v io u s Lab V a lu e s

A study investigating a new test for diagnosing acute myocardial infarction (AMI) has just been released. The
sensitivity of the test is 75% and the specificity is 80%. The study contains 200 patients who actually had an
AMI and 400 who actually did not, as determined by the diagnostic gold standard. How many false negatives
are in the study?

Block Tim e Rem aining:


Tutor
11 f M a rk

01:53

123456789
10

ED
12
13
14
15
16

Item: 11 of 16
Q. Id : 1229 [549201]

<a
P re v io u s Lab V a lu e s

A study investigating a new test for diagnosing acute myocardial infarction (AMI) has just been released. The
sensitivity of the test is 75% and the specificity is 80%. The study contains 200 patients who actually had an
AMI and 400 who actually did not, as determined by the diagnostic gold standard. How many false negatives
are in the study?
() A. 50 [60%] o B. 80 [19%] o c. 120 [9%] o D. 150 [7%] o E. 320 [4%]
Explanation: User Id: 549201
False negatives describe people who actually have a disease but test negative. In the table below they are
represented by square c.

Disease positive Disease negative


Test positive a
True positive
b
False positive a+b
Test negative c
False negative
d
True negative c+d
a+c b+d
Sensitivity = a/(a+c)
Specificity = d/(b+d)
USMLEWorid, LLC

Block Tim e Rem aining:

02:05

Tutor

11 ^Mark
123456789
10

ED
12
13
14
15
16

Item: 11 of 16
Q. Id : 1229 [549201]

<a
P re v io u s Lab V a lu e s

The number of false negatives can be calculated using the test's sensitivity and total number of patients with
AML The sensitivity of a test refers to the number of true positives divided by the total number of patients with
the disease (a / [a + c]). In this case, the sensitivity is 75% and the total number of patients with AMI (a + c)
equals 200. Using this information, the number of true positives (a) can be calculated:
Sensitivity = a / (a + c)
0.75 = a l 200
a = 150
Since the total number of patients with the disease (a + c) equals 200 and the number of true positives (a)
equals 150, the number of false negatives (c) is 50.
For the sake of thoroughness, the two-by-two table can be completed as follows. Specificity refers to the
number of true negatives divided by all people without the disease (d / [b + d]). Repeating the same exercise
as above gives the completed table:
Specificity = d / (b + d)
0.8 = d / 400
d = 320

b = 400 - 320 = 80
AMI No AMI
Test positive 150 80 230
Test negative 50 320 370
200 400
The number of true positives and false negatives can be more quickly calculated using the following equations:
True positives = (Sensitivity) * (Number of patients actually with the disease)
False negatives = (1 - Sensitivity) * (Number of patients actually with the disease)
Educational objective:
Sensitivity is the number of true positives divided by the total number of subjects actually with the disease.
True positives = (Sensitivity) * (Number of patients actually with the disease)
False negatives = (1 - Sensitivity) * (Number of patients actually with the disease)
Time Snent 8 second?;

Block Tim e Rem aining:


Onnvrinht tJSMI FWnrld I I C
02 : 10
Tutor

I ast undated f4/12/20141


11 f M ark
123456789
10
11
12
13
14
15
16

Item: 12 of 16
Q. Id : 1232 [549201]

t> i

l EP-m
Previous N| n ^ J Lab Values Notes Calculator
Your patient with chest pain wants to know if he is having a heart attack. You tell him that his result of test A, a
new test for diagnosing myocardial infarction (Ml), is positive. The study results for test A versus people who
have true myocardial infarction are given in table below.
Myocardial No myocardial
infarction infarction
Test positive 200 50
Test negative 120 80
Based upon test A, what is the probability that your patient is experiencing myocardial infarction?

o
o
o
o
o

A. 90%
B. 80%
C. 60%
D. 50%
E. 40%

Block Tim e Rem aining: 02 : 13 q p O


T u to r

Feedback Suspend End Block

11 f M a rk

123456789
10

11

m
13
14
15
16

Item: 12 of 16
Q. Id : 1232 [549201]

<a
P re v io u s Lab V a lu e s

Your patient with chest pain wants to know if he is having a heart attack. You tell him that his result of test A, a
new test for diagnosing myocardial infarction (Ml), is positive. The study results for test A versus people who
have true myocardial infarction are given in table below.

Myocardial No myocardial
infarction infarction
Test positive 200 50
Test negative 120 80
Based upon test A, what is the probability that your patient is experiencing myocardial infarction?

A. 90% [2%]
<) B. 80% [77%]

o
o
o

C. 60% [15%]
D. 50% [2%]

E. 40% [3%]
Explanation: User Id: 549201
The positive predictive value (PPV) of a test refers to the number of people with a disease who test positive
divided by the total number of people who test positive. It gives the probability that someone with a positive
test will truly have a disease. Using a two-by-two table the PPV equals a/(a+b).
Generic

Disease
positive
Disease
neqative
Test
positive
a
True
positive
b
False
positive
a+b
Test
negative
c
False
negative
d
True
negative
c+d
ct-i-r h 4.H

Block Tim e Rem aining:


Tutor

l= >
Test A
Ml Wo Ml
Test A
positive
200 50 250
Test A
negative
120 80 200
320 130
11 f M ark
123456789

10
11

0 2 : 19

12

13
14
15
16

Item: 12 of 16
Q. Id : 1232 [549201] Previous< a NextE > Lab Values

II

Based upon test A, what is the probability that your patient is experiencing myocardial infarction?

A. 90% [2%]
<) B. 80% [77%]

0
0
0

C. 60% [15%]
D. 50% [2%]

E. 40% [3%]
Explanation: User Id: 549201
The positive predictive value (PPV) of a test refers to the number of people with a disease who test positive
divided by the total number of people who test positive. It gives the probability that someone with a positive
test will truly have a disease. Using a two-by-two table the PPV equals a/(a+b).

Generic
Disease
positive
Disease
neqative
Test
positive
a
True
positive
b
False
positive
a+b
Test
negative
c
False
negative
d
True
negative
c+d
a+c b+d
Test A
Ml Wo Ml
Test A
positive
200 50 250
Test A
negative
120 80 200
320 130
Unlike specificity and sensitivity, PPV varies by the prevalence of disease. If the disease prevalence

increased the PPV will increase; similarly, PPV will decrease with decreasing prevalence. In our case the
PPV = 200/(200+50), or 80%
Educational Objective:
Positive predictive value represents the probability of truly having a disease for a given positive test result. It
increases with increasing disease prevalence and decreases with decreasing disease prevalence.
Time Spent: 7 seconds Copyright USMLEWorld,LLC.
Block Tim e Rem aining: 02 : 23
T u to r

Last updated: [8/22/2014]


Feedback
Notes
Suspend
Calculator
A

End Block
11 f M a rk

123456789
10
11
12
KQ
14
15
16

Item: 13 of 16
Q. Id : 1283 [549201]

Plasma homocysteine levels are measured in patients with acute coronary syndrome who are treated at a
large community hospital. The mean plasma homocysteine level in this group is determined to be 11.1
pmol/L with a standard deviation of 1.2 pmol/L. In a separate group of patients hospitalized on the general
ward in the same hospital, the mean plasma level is 9.5 pmol/L and the standard deviation is 1.3 pmol/L.
Which of the following statistical methods should be used to compare the mean homocysteine levels of these
2 groups of patients?
Lab V a lu e s

O
O
O
O
O

A. Two-sample t test
B. Linear regression
C. Correlation coefficient
D. Chi-square test
E. Meta-analysis

Block Tim e Rem aining: 02 : 26


Feedback Suspend End B lo ck
11 f M ark
123456789

10
11
16

Item: 13 of 16
Q. Id : 1283 [549201]

<a
Previous

E>
Next

II
Lab Values

Plasma homocysteine levels are measured in patients with acute coronary syndrome who are treated at a
large community hospital. The mean plasma homocysteine level in this group is determined to be 11.1
pmol/L with a standard deviation of 1.2 pmol/L. In a separate group of patients hospitalized on the general
ward in the same hospital, the mean plasma level is 9.5 pmol/L and the standard deviation is 1.3 pmol/L.
Which of the following statistical methods should be used to compare the mean homocysteine levels of these
2 groups of patients?
* A. Two-sample t test [69%]
O B. Linear regression [1 %\
O C. Correlation coefficient [7%]
O D. Chi-square test [20%]
O E. Meta-analysis [3%]

Explanation: User Id: 549201

Two-sample t test
Null hypothesis Alternate hypothesis
Both samples drawn from
the same population
(Ml = M2)
Both samples drawn from
different populations
(Ml * M2)
Block Tim e Rem aining: 02 : 36
Feedback
Notes
Suspend
Calculator
T u to r

End B lo ck
11 f M a rk

123456789
10

11
12
KQ
14
15
16

Item: 13 of 16
Q. Id : 1283 [549201]

Null hypothesis
Both samples drawn from
the same population
(Ml = M2)
P re v io u s Lab V a lu e s

Alternate hypothesis
Both samples drawn from
different populations
(pi 5* m2 )
Small apparent difference between the means of the samples
null hypothesis not rejected
Large apparent differences between the means of the samples
null hypothesis rejected
Block Tim e Rem aining:

02:41

Tutor

11 ^Mark
123456789
10

11
12
KQ
14
15
16

Item: 13 of 16
Q. Id : 1283 [549201]

<a
P re v io u s Lab V a lu e s

USMLEW orld, LLC

The two-sample t test is commonly employed to determine if the means of 2 populations are equal. Several
statistical approaches can be used, but the basic numerical requirements needed to perform this test are the
2 mean values, the sample variances (eg, standard deviations), and the sample sizes. The t statistic is then
calculated, from which the p value can be determined. If p < 0.05, the null hypothesis (which assumes that
there is no difference between 2 groups) is rejected and the 2 means are assumed to be statistically different.
(Choice B) Linear regression is used to model the linear relationship between a dependent variable and an

independent variable. For example, linear regression could be used to determine the relationship (described
in terms of a trend line) between the number of cigarettes smoked per day and the number of yearly
hospitalizations in COPD patients.
(Choice C) The correlation coefficient is a measure of the strength and direction of a linear relationship
between 2 variables. For example, a study may report a correlation coefficient describing the association
between estrogen levels and breast cancer risk in postmenopausal women. It is different from linear
regression in that a single number is reported describing the strength and magnitude of the association.
(Choice D) The chi-square test is most appropriate for use with categorical data. It can be used to evaluate
whether the expected frequency of an occurrence is consistent with the observed frequency of that
occurrence (goodness of fit"). For instance, a study evaluating Mendelian inheritance of red and green seed
colors would use a chi-square test to compare the observed and expected proportions of each seed type.
(Choice E) Meta-analysis is an epidemiologic method of analyzing pooled data from several studies, thereby
increasing the statistical power beyond that of the individual studies.
Educational objective:
The two-sample t test is a statistical method commonly employed to compare the means of 2 groups of
subjects.
Time Spent: 9 seconds

Block Tim e Rem aining:


Copyright USMLEWorld,LLC. Last updated: [4/12/2014]
02:44
Tutor
11 f M a rk

10

O
12 O
11

13

14 o

*
15

o
16

o
Item: 14 of 16
Q. Id : 1301 [549201]

<a
P re v io u s Lab V a lu e s

A study to assess spironolactone's efficacy in patients with heart failure is performed. 450 patients receive
either spironolactone or placebo for two years. Neither the patients nor physicians are aware of who takes the
drug or placebo. The study setup described above is most effective in preventing:

Block Tim e Rem aining:

02:48

Tutor
11 f Mark
123456789
10
11

12
13

14
15
16

Item: 14 of 16

II

Q. Id : 1301 [549201] Previous< a NextE > Lab Values

A study to assess spironolactone's efficacy in patients with heart failure is performed. 450 patients receive
either spironolactone or placebo for two years. Neither the patients nor physicians are aware of who takes the
drug or placebo. The study setup described above is most effective in preventing:

O
O

A. Beta error [3%]

O
O

D. Effect modification [8%]

B. Recall bias [3%]


* () C. Observer bias [72%]
E. Selection bias [13%]

Explanation: User Id: 549201


Blinding technique is commonly used in clinical trials. The blinding can involve patients exclusively or both
patients and physicians (double blinding). The main purpose of blinding is to prevent patient or researcher
expectancy from interfering with the determination of an outcome. For example, a researcher's belief in a
positive outcome in treated patients can potentially result in observer bias.
(Choice A) Beta error refers to a conclusion that there is no difference between the groups studied when a
difference truly exists. Beta error is a random error, not a systematic error (i.e., bias).
(Choice B) Recall bias results from the inaccurate recall of past exposure by subjects. It applies mostly to
case-control studies.
(Choice D) Effect modification is not a bias and should not be controlled.
(Choice E) Selection bias results from the manner in which people are selected for the study, or from the
selective losses from follow-up.
Educational Objective:
The main purpose of blinding is to prevent patient or researcher expectancy from interfering with an outcome.
Time Spent: 9 seconds Copyright USMLEWorld,LLC. Last updated: [8/22/2014]
Block Tim e Rem aining: 02 : 53
T u to r

Feedback
Notes
Suspend
Calculator
End B lo ck
11 f M a rk

123456789
10

11
12
13
14

m
16

Item: 15 of 16
Q. Id : 1174 [549201]

<a
P re v io u s Lab V a lu e s

Physician researchers are evaluating the efficacy of a new lipid-lowering drug being marketed as Superstatin.
The drug manufacturer claims that the drug is more effective than existing hypolipidemic agents in preventing
myocardial infarctions. Results of a 5-year randomized control trial are shown below.
Number of patients treated Number of patients treated with
with Superstatin control medication
Myocardial infarction 10 25
No myocardial infarction 990 975
Compared to the control medication, how many patients need to be treated with Superstatin to prevent one
additional myocardial infarction?

o
o
o
o
o
o
o

A. 1
B. 2
C. 2.5
D. 15
E. 40
F. 67
G. 100

Block Tim e Rem aining:


Tutor
11 f M ark
123456789
10
11

12
13

Item: 15 of 16
Q. Id : 1174 [549201]

02:56

* F. 67 [67%]
O G. 100 [0%]
<>
Previous Next Lab Valuesi

Explanation: User Id: 549201


C o m m o n m e a s u re s o f th e ra p e u tic e ffic a c y
Term D efinition C alculation
A bsolute R isk
R eduction
(ARR)
Percentage indicating
the actual difference in
event rate between
control and
treatment groups.
ARR = Control Rate - Treatment Rate
Relative Risk
R eduction
(RRR)
Percentage indicating
relative reduction in the
treatment event rate
compared to the control
group.
RRR = ARR/Control Rate
Relative Risk
(RR)
Ratio of the probability
of an event occurring in
the treatment group
compared to the
control group.
RR = Treatment Rate/Control Rate
Num ber
Needed to Treat
(NNT)
Number of individuals
that need to be treated
to prevent a negative
outcome in one patient.
NNT = 1/ARR

Block Tim e Rem aining: 03 :13


Feedback
Notes
Suspend
Calculator
T u to r

End Block
11 f M a rk

123456789
10

11
12
13
14

m
16

Item: 15 of 16
Q. Id : 1174 [549201]

<a
P re v io u s Lab V a lu e s

R eduction
(RRR)
treatment event rate
compared to the control
group.

RRR = ARR/Control Rate


Relative Risk
(RR)
Ratio of the probability
of an event occurring in
the treatment group
compared to the
control group.
RR = Treatment Rate/Control Rate
Num ber
Needed to Treat
(NNT)
Number of individuals
that need to be treated
to prevent a negative
outcome in one patient.
NNT = 1/ARR
USMLEWorkJ, LLC

The number needed to treat (NNT) represents the number of patients that need to be treated with a
medication in order to prevent an additional negative outcome. NTT is calculated by dividing 1 by the absolute
risk reduction (ARR). In this example, ARR is calculated by subtracting the event rate in the treatment group
(10/1000= 0.01 or 1%) from the event rate in the control group (25/1000 = 0.025 or 2.5%). Performing the
calculation (0.025 - 0.01) results in an ARR value of 0.015 or 1.5%. Finally, dividing 1 by 0.015 gives an NNT
of 66.6. Rounding up the result to the nearest whole number, 67 patients need to be treated with Superstatin
to prevent an additional myocardial infarction.
A similar concept to NNT is the number needed to harm (NNH), which usually applies to medication side
effects. In both cases, lower values of NNT or NNH imply that an intervention is more beneficial (NNT) or
harmful (NNH). For instance, if a third hypolipidemic medication had a NNT of 40, it would be more beneficial
than Superstatin as only 40 patients would need to be treated to prevent an additional myocardial infarction.
Educational objective:
The number needed to treat (NNT) is the number of patients that need to be treated with a medication to avoid
a negative outcome in one patient. NTT is calculated by dividing 1 by the absolute risk reduction. Lower NTT
values represent more beneficial treatments.
Time Spent: 8 seconds

Block Tim e Rem aining:


Copyright USMLEWorld,LLC. Last updated: [6/24/2014]
03:15
Tutor
11 f M a rk

123456789
10

11
12
13
14
15

Item: 16 of 16
Q. Id : 1204 [549201]

<a
P re v io u s Lab V a lu e s

A study determines that the mean blood cholesterol level in 200 non-diabetic hospitalized patients is 195
mg/dL and in 180 diabetic hospitalized patients is 210 mg/dL. The probability that the observed difference is
due to a chance is reported to be 5%. There is also a 20% probability of concluding that there is no difference
in blood cholesterol level when there is one in reality. What is the power of the study?

o
o
o
o
o

A. 0.05
B. 0.20
C. 0.50
D. 0.80
E. 0.95

Block Tim e Rem aining:


Tutor
11 f Mark

03:18

123456789
10
11

12
13
14
15
16

Item: 16 of 16

II

Q. Id : 1204 [549201] Previous< a NextE > Lab Values

A study determines that the mean blood cholesterol level in 200 non-diabetic hospitalized patients is 195
mg/dL and in 180 diabetic hospitalized patients is 210 mg/dL. The probability that the observed difference is
due to a chance is reported to be 5%. There is also a 20% probability of concluding that there is no difference
in blood cholesterol level when there is one in reality. What is the power of the study?
Explanation: User Id: 549201
Power of a study indicates the probability of seeing a difference when one truly exists. In our example, power
describes the probability of detecting a difference in blood cholesterol level between diabetics and nondiabetics if
there is a real difference. Power is reciprocally related to type II error (P), i.e., stating that there is
no difference between groups when one truly exists (Choice B). Mathematically, power is defined:
Power = 1 - p
Power of a study is positively correlated with the sample size. Type I error (a) describes the probability of
seeing a difference when there is no difference in reality (Choice A).
Educational Objective:
Power of a study indicates the probability of seeing a difference when there is one; Power = 1 - p.
Time Spent: 9 seconds Copyright USMLEWorld,LLC. Last updated: [8/22/2014]

Block Tim e Rem aining:


T u to r

Feedback
Notes
Suspend
Calculator
End B lo ck

03 : 25

Anda mungkin juga menyukai